glannon, pro res

¡Supera tus tareas y exámenes ahora con Quizwiz!

QUESTION 5. Let's fix this. An attorney is a fifth-year associate at a large firm, hoping to make partner in the next two or three years. She supervises the first-year associates at the firm. She learns that the most recently hired associate recently shredded some evidence that would have undermined a client's case, and then told the judge and opposing party that the missing documents had been in a briefcase that went missing when a burglar broke into the associate's car. The attorney knows this is not true and discusses it with the senior litigation partner, who finds the story amusing. Neither reports the associate's deception to the judge or opposing party. Which of the following statements is true regarding this situation?

A.Both the attorney and the litigation partner are subject to discipline for not taking action to correct the associate's false statements and misconduct.

QUESTION 5. Contacting the warehouse worker. While preparing for a trial over workplace discrimination, the plaintiff's attorney contacts an entry-level, night-shift worker in the company's off-site warehouse, who supposedly told the frequent, shocking sex jokes that led to the "hostile environment" claim by female workers that became the subject of the pending lawsuit. The attorney did this without the permission of the company's lawyer, even though he suspects the company's lawyer would have allowed it if he had asked. The warehouse worker has no supervisory authority, has never been to the corporate offices, has no authority to bind the company, and is now on suspension (unpaid leave) while the company does its own internal investigation of the allegations. Was the communication by the plaintiff's attorney proper?

A.No, the attorney may not communicate with the constituent of a represented organization (opposing party) whose act or omission in connection with the matter may be imputed to the organization for purposes of civil liability.

QUESTION 11. Duties in the cloud. The attorney has switched to cloud computing, meaning that their firm pays a monthly fee to store all their spreadsheets and documents in an Internet-based database or archive. This protects client information and case documents from being lost whenever a computer at the firm crashes; the cloud service automatically creates an online backup for every file. According to the Rules of Professional Conduct, which of the following is true?

A.The attorney and his firm have an affirmative duty to make reasonable efforts to ensure that the cloud service is secure against computer hacking or other invasive access to clients' confidential information.

QUESTION 2. Not my brother's keeper. An attorney is a second-year associate at a law firm with no supervisory responsibilities. He learns that another second-year associate is working on a case in which the client is suing a company that the other associate used to represent at his previous firm, and the attorney suspects it is a substantially related matter. The firm has done nothing to screen the other associate from the matter. No one ever discusses it with the attorney, and the attorney does not know all the facts of the situation. Later, the client sues the firm for malpractice due to the conflict of interest and reports the matter to the state disciplinary authority. Which of the following is true regarding the attorney's involvement in the situation?

A.The attorney does not have disciplinary liability for the conduct of the other associate, because he is neither a partner nor in a supervisory position and did not participate in the violations directly.

QUESTION 9. Nuremberg defense. An attorney is an associate in a litigation firm representing plaintiffs. In her current case, her supervising partner instructs her to assert that the defendant had an affirmative statutory duty to protect the plaintiff's interests, even though the attorney can find no statute to support this assertion. The attorney has brought this to the attention of her supervising partner, who rebuked her for questioning his authority and insisted that she do as he said. He assures her that the defendants will settle before trial anyway, so the bogus claim merely gives some psychological leverage during settlement negotiations, and it cannot do any harm. Moreover, the partner says that the attorney may not last long at the firm if she cannot follow instructions, which could have been a threat of termination. At a preliminary hearing, however, the judge confronts the attorney about the unsupportable claim, and she concedes that no statutory duty exists. The judge is irate and considers reporting the attorney to the state bar disciplinary authority. Which of the following is correct regarding the attorney's situation?

A.The attorney is responsible for asserting a frivolous claim, even though her supervising partner insisted that she do it and threatened her with termination.

QUESTION 3. Respondeat superior. An attorney is a fifth-year associate at a large firm and is responsible for supervising the work of a first-year associate. The attorney, however, now spends most of his time in Singapore, trying to open a satellite office for the firm there to service one of its major corporate clients. He has not inquired into the associate's compliance with the Rules of Professional Conduct in over eighteen months, as they mostly communicate by e-mail regarding pending cases and assignments. To the best of his knowledge, though, the attorney believes the associate is following the rules, and he knows that the associate has attended two Legal Ethics CLE courses in the last year. Unbeknownst to the attorney, the new associate has been overbilling hours and has been neglecting certain client matters. Which of the following is true regarding the attorney's situation?

A.The attorney is subject to discipline as a lawyer having direct supervisory authority over another lawyer who failed to make reasonable efforts to ensure that the other lawyer conforms to the Rules of Professional Conduct, even though there was no direction, ratification, or knowledge of the violation.

QUESTION 4. Protocols in place. An attorney is a partner in a medium-size firm. Another partner at the firm, the managing partner, is responsible for implementing policies and procedures to detect and resolve conflicts of interest, to account for client funds and property, to identify dates by which actions must occur in pending matters, and to ensure that inexperienced lawyers receive proper supervision. The managing partner, however, now spends most of his time in Singapore, trying to open a satellite office for the firm there to service one of its major corporate clients. The managing partner is rarely at the home office and has completely neglected the implementation of ethical policies in the firm, so that minimal safeguards or procedures are in place. One of the new associates has committed several serious violations of professional responsibilities in the last few months, including an egregious conflict-of-interest problem and several missed deadlines for filing responsive pleadings. The attorney knew nothing about the violations and was not directly supervising the associate, and tries not to meddle in any of the managing partner's responsibilities, including the implementation of ethical policies and procedures. Which of the following is correct?

A.The attorney is subject to discipline as a partner in the firm for failing to make reasonable efforts to ensure that the firm has in effect measures giving reasonable assurance that all lawyers in the firm conform to the Rules of Professional Conduct.

QUESTION 7. Just following orders. The attorney is an associate at a small firm, and her supervising partner instructs her to draft pleadings in a case for a client. The supervising partner knows that the statute of limitations has already run on the claim, and that the client had little or no factual evidence to support the claim in any case. The partner believes the opposing party will want to settle the claim quickly for a modest sum and will not bother to investigate issues such as the statute of limitations or the factual support for either side. The attorney follows the partner's instructions and drafts the pleadings, without checking the statute of limitations for this specific claim or conducting her own investigation into the facts of the case. Opposing counsel, however, is upset over the frivolous claim and reports the attorney to the state bar. Which of the following is correct regarding the attorney's situation?

A.The attorney should not be responsible for asserting a frivolous claim, and the fact that she was just following orders could support her defense that she was unaware that the claim was frivolous.

QUESTION 26. The lawyer-CPA. An attorney specializes in estate planning. Besides being a lawyer, she is a certified public accountant (CPA). One of her clients hires her to prepare a will and handle the planning for a complex estate, which will involve creating two charitable trusts and other maneuvers for avoiding hefty estate taxes. The estate planning in this case involves some transfers to create the trusts in the current calendar year, which will be reportable on the current year tax returns. The client asks the attorney to prepare her tax returns for the current year, given that the attorney is handling all the estate planning, and already has all the documentation about the finances and assets of client. The attorney agrees to prepare the returns as a CPA, and she creates a separate retainer agreement with the client for the preparation of the tax returns, one that complies with all IRS requirements for tax preparers and stipulates this retainer shall be for accounting work, not legal services. Five years later, the same client runs for Congress, and during a contentious campaign, a reporter asks the attorney how much the client paid in taxes in the year that the attorney prepared the tax returns. The attorney answers the question in detail. The client complains that this constitutes a breach of lawyer confidentiality, but the attorney defends her actions by explaining that the amount of taxes paid that year was information derived solely from her work as a CPA, under a separate retainer with due disclosures, and not as a lawyer. Who is correct here—the client, or the attorney?

A.The client is correct because the circumstances were such that the non-legal accounting services were not distinct from the legal services attorney was providing at the time.

QUESTION 10. Regulation by the legislature. A state legislature enacted a statute governing the licensing of attorneys and discipline for practitioners. The preamble to the statute asserted "field preemption" over the regulation of lawyers in that jurisdiction, thereby abolishing all prior rules and codes of the state bar. An attorney came under discipline under the new law and contested the legal validity of the enactment itself. What should be the result?

A.The court will hold the law invalid because the judiciary has inherent power to regulate the attorneys who practice in its courts.

QUESTION 4. Criticizing the jury. A judge presided over a controversial, high-profile jury trial of a county clerk who engaged in civil disobedience and refused to issue marriage licenses to gay couples, even after receiving a court order to do so. The judge thought that the law and the evidence against the defendant was overwhelming, as the clerk freely admitted that she had willfully flouted the law and disregarded the court order. Even so, the jury decided to engage in nullification and acquitted the county clerk of all charges. In shock, the judge chastised the jury for not honoring their oath to uphold the law, and for allowing their political views influence their verdict in the case. Under the Code of Judicial Conduct, which of the following is true?

A.The judge could be subject to discipline for criticizing the jurors for their decision.

QUESTION 10. Relevant factors. A partner gives an associate the typed notes from a previous client interview conducted by the partner, and the associate has the task of drafting a complaint for a personal injury lawsuit based on the allegations in the notes. The associate has no direct contact with the client, and does not really have any way to verify whether the notes represent everything discussed in the interview (the notes are not a transcript) or whether the allegations are factually accurate, truthful, or tell the complete story. The associate completes the task as assigned, drafting the pleadings based on the notes. The associate then submits the drafted complaint to the partner for review. Later, the pleadings turn out to be frivolous, based on complete falsehoods. Which of the following is true regarding the associate attorney's role in drafting the complaint?

A.The lack of opportunity for the associate to investigate or verify facts on her own will be a relevant factor for the state disciplinary authority in deciding whether to discipline the associate.

QUESTION 6. Dumping on the associates. An attorney works at the state Public Defender office. Due to their insufficient funding and the overwhelming number of indigent defendants in her city, her caseload is so great that she cannot do adequate investigation into any of her client's cases or conduct legal research about possible defenses. Nevertheless, 95 percent of the cases end in plea bargains without going to trial, so the attorney tells herself that her neglect of case development makes no difference. Her supervisor at the Public Defender office is aware of the unreasonable caseloads of all the attorneys who work there, but the supervisor wants the attorneys to increase their caseloads in order to provide representation to more indigent defendants, even if that means doing minimal work on each case. Which of the following is true regarding the ethical situation facing the attorney and her supervisor?

A.The supervisor could be subject to discipline for not ensuring that a subordinate attorney can manage her workload, even if that means not assigning the lawyer any more cases for now.

QUESTION 8. Reasonable resolution. An attorney is a new associate at a law firm, and the managing partner assigns her a new case, in which the firm will represent two co-plaintiffs in a personal injury case. The attorney is concerned that a conflict of interest could arise between the two plaintiffs and suggests that the firm should represent only one of them. When she discusses this with the managing partner, the managing partner disagrees, because the interests of the two plaintiffs seem perfectly aligned, and they can have each sign an informed consent form waiving the conflict up front. Both admit the question is a close one in terms of the ethical rules for conflicts of interest, but the managing partner insists that they proceed. Which of the following is true regarding this situation?

A.The supervisor's reasonable resolution of the question should protect the subordinate professionally if the resolution subsequently faces a challenge.

QUESTION 12. Have it your way. An attorney represents a sophisticated business client in a litigation matter. The attorney wants to hire an outside non-lawyer investigator/paraprofessional to help find and develop evidence and witnesses for the case. The client agrees but wants the attorney to hire a particular outside company with whom the client has close business dealings and a long history. The attorney would normally have used a different firm that is more familiar to him. Which of the following is correct, according to the Model Rules and the accompanying comments?

A.Where the client directs the selection of a specific non-lawyer service provider outside the firm, the lawyer ordinarily should agree with the client concerning the allocation of responsibility for monitoring as between the client and the lawyer.

QUESTION 1. Responsibility versus liability. An attorney is a partner in a newer law firm that has no effective measures in place to ensure that lawyers in the firm conform to the Rules of Professional Conduct. An associate at the firm violates the rules, and the state bar investigates the policies and procedures in place at the firm. The state disciplinary authority has determined that the first attorney is subject to discipline for his failure to take reasonable measures to ensure conformity with the rules. Because of this determination and the subsequent sanction, which of the following is true?

A.Whether a lawyer may be liable civilly or criminally for another lawyer's conduct is a question of law beyond the scope of the rules; the determination of a violation does not automatically mean that the partner attorney would be civilly or criminally liable.

QUESTION 2. A mutually beneficial arrangement. An attorney made an informal agreement with a physician that they would refer clients to each other when the situation seemed appropriate. They did not pay each other any money for referrals, but the relationship was explicitly reciprocal—the attorney referred patients who needed medical examinations to the physician, and when the physician had patients needing legal representation, he referred them to the attorney. The relationship was not explicitly exclusive—each was free to refer clients to others—but it happened that neither had similar reciprocal relationships with anyone else. They always informed their clients when making such referrals that they had a reciprocal relationship. Is such an arrangement proper?

A.Yes, a lawyer may agree to refer clients to another lawyer or a non-lawyer professional, in return for the undertaking of that person to refer clients or customers to the lawyer, as long as clients are aware and the relationship is not exclusive.

QUESTION 15. A deeply divided law school faculty. A law school suffers from deep divisions among its faculty. One group of the faculty dislikes the Dean and wants to force his resignation with a vote of no confidence and pressure on the Board of Trustees. The other group is loyal to the Dean and resents their disloyal colleagues, whom they consider unprofessional. The controversy surrounding the law school's Dean overlaps with faculty divisions over hiring practices, tenure, and whether the school should try to emulate top-tier law schools in order to boost their national rankings, or if they should focus instead exclusively on pedagogy and preparing the students for the practice of law after graduation. The divisions are so great that each faction has threatened to quit or take other drastic action that would imperil the school's existence, if their side does not prevail. The Board of Trustees obtains an agreement from both factions on the faculty that they will hire an attorney to function as a third-party neutral to attempt to broker a compromise between the factions on the faculty. The attorney is an alumnus of the law school and offers to serve in this capacity without charging legal fees. He claims that he is not representing the Board, the Dean, or either side of the balkanized faculty. He begins to schedule private conferences with each faculty group, the Dean, and the Board, as well as meetings attended by representatives from each faction of the faculty to have deliberations and consider possible compromises. The attorney also insists that he is not an arbitrator or mediator because no litigation over the dispute is pending or even contemplated at this point. Is it proper for the attorney to serve in this capacity?

A.Yes, because a lawyer can serve as a third-party neutral when the lawyer assists two or more persons who are not clients of the lawyer to reach a resolution of a dispute or other matter that has arisen between them.

QUESTION 4. The big spender. An attorney pays $1,000 per month for a billboard advertisement for his firm, $2,000 per month for a few radio commercials, $3,000 per month for Internet advertising, and $4,000 per month for newspaper and magazine advertisements. The total amount for advertising is $10,000. At the same time, the attorney's average total income from legal fees is $15,000 per month. Is it permissible for the attorney to spend such sums on advertising?

A.Yes, because a lawyer may advertise services through written, recorded, or electronic communication, including public media, and may pay the reasonable costs of such advertisements or communications.

QUESTION 1. Across state lines. An attorney represented a seller in a business transaction involving industrial equipment. When the deal was complete, the purchaser sent the attorney a check for $7,000, the agreed-upon purchase price, with a letter directing the attorney to forward the money to his client (the seller). The attorney notified his client immediately that the check had come in. The client was traveling at the time, and asked the attorney to hold the funds until he returned from his trip. The attorney had only recently moved to this jurisdiction and opened a new firm, did not yet have a client trust account at any banks in the area, so he deposited the check in the client trust account in the neighboring state, where he had practiced until recently. He told the client that the funds would be in a separate client trust account, and explained that it would be out of state, and the client consented. As soon as the check cleared, the attorney wrote a check to the client for the full amount from the client trust account, which the client picked up in person. Did the attorney act properly in this case?

A.Yes, because a lawyer may deposit client funds in an out-of-state client trust account if the client gives informed consent to this arrangement.

QUESTION 6. Disinheriting the daughter. An attorney agreed to prepare a will for a client, a wealthy widow with three grown children. An earlier will divided her estate equally between her children, but the client now wants to modify the will to disinherit her only daughter, who disobeyed the client's wishes by marrying outside their nationality. The daughter is also a lawyer and is married to a lawyer, and the estate is substantial. The client's two sons are both working as manual laborers and they struggle financially. In the past, there had been some tension between the brothers and their sister, although the relationships seem to be cordial now. The attorney believes that disinheriting the daughter will ensure that the daughter and her husband will contest the will after the client's death and will rupture the tenuous relationship between the siblings. The client did not ask for the attorney's advice about disinheriting the daughter, she just insisted on it. The attorney initiated a debate about it, explaining that he believed it could be against the client's best interests and would cause unnecessary acrimony between her children. Was it proper for the attorney to initiate such advice when the client did not ask for it?

A.Yes, because a lawyer may offer unsolicited advice to a client when doing so appears to be in the client's interest.

QUESTION 5. All in the family. An attorney calls his friend, a close personal acquaintance, who was recently arrested for driving while intoxicated. The attorney advises that he saw the friend's arrest on the local police news and offers to represent his friend for the attorney's usual fee for handling such cases. The friend hires the attorney to represent him on the case. Are the attorney's actions proper?

A.Yes, because attorneys can solicit professional employment from family members, close personal friends, and persons with whom the attorney had a previous professional relationship.

QUESTION 3. Don't lecture me. An attorney represents a client, who lost his criminal appeals and is now serving a life sentence in a federal penitentiary. The client confesses to the attorney that he (the client) committed a murder for which a jury incorrectly convicted another (innocent) man. The client says he is happy that someone else took the fall for that crime and that he will never tell anyone. The attorney lectures the client about the morality of this situation, allowing an innocent man to face life imprisonment or even capital punishment for a crime that the client committed, and pleads with the client to reveal the truth. Was it proper for the attorney to bring morality into his consultation with the client, and to sermonize on this point for a few moments?

A.Yes, because in rendering advice, a lawyer may refer not only to law but to other considerations such as moral factors.

QUESTION 4. Money laundering. An attorney knew about another lawyer's involvement in an illegal money laundering enterprise, although the money laundering was unrelated to the other lawyer's law practice or representation of clients. Eventually, when federal law enforcement officials bring criminal charges against the other lawyer, who is part of another firm, the first attorney's awareness of the situation becomes evident. Could the attorney who knew of the wrongdoing and ignored it be subject to discipline?

A.Yes, because it is a violation of the Rules of Professional Conduct to fail to report serious fraud or criminal activity by another lawyer.

QUESTION 5. European vacation. An attorney received a small settlement check from a client for the opposing party on Christmas Eve, late in the afternoon. All the support staff at the firm had already gone home for the day, due to the impending holiday, and the firm was to stay closed until January 2. The attorney was rushing to catch a flight to Europe, where she planned to spend the holidays with her family. On January 2, when the attorney returned and reopened the office, she promptly notified the opposing party that the check had arrived. Could the attorney be subject to discipline for her actions in this case?

A.Yes, because she did not notify the opposing party soon enough.

QUESTION 10. Certified by whom? An attorney is properly certified as an immigration law specialist by a state bar organization that provides official certifications. In her advertisements, the attorney describes herself as a "Certified Specialist in Immigration Law" without identifying the certifying organization. The attorney also mentions that she speaks Spanish and Portuguese (besides English), and that her fees are very affordable. Could the attorney be subject to discipline for making such statements in her advertisements?

A.Yes, because she failed to identify the certifying organization.

QUESTION 7. The victim's story. An attorney represented a client in a misdemeanor criminal matter involving minor vandalism. The attorney interviewed the victim, who incurred the property damage, hoping to learn more about the value of the damage and how frequently vandalism occurs in that neighborhood. The property owner explained to the attorney that the client had been demanding "protection money" from him and other business owners in the neighborhood for a long time, and that the vandalism followed his refusal to continue paying the protection money. The amount involved was substantial, and the attorney realized that the client could face much more serious charges for extortion. The attorney never discussed this with the client, and the client gladly accepted a plea bargain offer for a few months' probation on the misdemeanor vandalism charge. Several years later, the client died in a car accident, and the property owner became a business-world celebrity when he published a book about how businesses transform neighborhoods. A reporter eventually found the attorney and interviewed him about the vandalism incident, several years prior, that had damaged the property owner's building at the time. The attorney explained that the incident was in fact part of a larger extortion operation and that the business owner had handled the matter nobly. Should the attorney be subject to discipline for this disclosure?

A.Yes, because the confidentiality rule applies not only to matters communicated in confidence by the client but also to all information relating to the representation, whatever its source.

QUESTION 22. Living across the state line. An attorney lives in the border town of Nashua, New Hampshire, which is a forty-five-minute drive from Boston, Massachusetts. The attorney took the Massachusetts bar exam and passed it, and now seeks admission to the bar in that state, as she has a job offer from a firm in Boston and plans to commute there every day from her home in New Hampshire. The state bar of Massachusetts has a rule that lawyers must be residents of the state to obtain a license to practice law there on a regular basis, so it declines her application to the bar. When the attorney challenges this decision in federal court, will she prevail?

A.Yes, because the residency requirement violates the Privileges and Immunities Clause of the U.S. Constitution.

QUESTION 1. The former employee. In anticipation of trial, a plaintiff's attorney contacts several former employees of the defendant corporation and interviews them about the day-to-day operations of the company and the chain of command for addressing complaints. The attorney does this without permission from the defendant's lawyer. Was this proper?

A.Yes, consent of the organization's lawyer is not a requirement for communication with a former constituent of the organization that is a represented opposing party.

QUESTION 8. Background research. A newly appointed judge finds herself assigned to a court in a rural agricultural area, so the court has many cases related to farm ownership and foreclosures, government regulation and subsidies for agriculture, and so on. The judge has lived her entire life in a major urban center, and she is unfamiliar with this area of law and the economic realities faced by individuals and firms in the farming industry. To learn more background for upcoming cases that are not yet assigned, the new judge spends some of her weekends and evenings reading articles and blogs online. Does this background research violate the CJC?

C.No, because judges may do background reading to become better informed.

QUESTION 14. Profit sharing for paralegals. An attorney practices personal injury law, representing plaintiffs on a contingent fee basis. The attorney employs a paralegal to assist with preparing documents for litigation. The paralegal's salary arrangement is 10 percent of the firm's total net revenue each year. In years when the attorney wins several large cases, the paralegal receives higher wages, and in years when the attorney has no big wins, the paralegal receives almost nothing. The paralegal does not bring clients to the firm and does not participate in judgments about which clients to represent, or about how to handle the cases. Is the attorney subject to discipline for this arrangement?

B.No, because non-lawyers may participate in a firm compensation plan based on overall profit sharing.

QUESTION 10. Choose the violation. An attorney accepted a position on the state Commission on Judicial Conduct, a division of the judicial branch that investigates complaints of misconduct by judges and brings disciplinary actions when a violation of the Code of Judicial Conduct has occurred. The attorney was surprised by the wide range of rules that regulate the conduct of judges, as well as the variety of complaints that the Commission had to investigate each year. Which of the following activities violated the Code of Judicial Conduct?

B.A judge's clerk did Internet research about alternative methods for toxic mold remediation in homes in a case in which one of the parties alleged, among other things, that the opposing party failed to mitigate damage from toxic mold. The clerk wrote a memorandum for the judge about her research, and the judge did not disclose to the parties or their counsel that the clerk had done this research.

QUESTION 13. The law student intern. An attorney works for a mid-size law firm that employs two or three law students every year as summer associates. The manager of the student associates assigns one of them to work on the attorney's pending antitrust case, in addition to assignments for other lawyers at the firm. While researching an important issue in the case, the summer associate discovered an older Supreme Court decision that was unfavorable to their client. The summer associate decided not to tell anyone about the case, as the opposing party seemed to have overlooked it in their briefs. The attorney was not aware of any of this. The hearing went well for their side, and the associate never said a word about "burying" that Supreme Court case he had found. After winning at the hearing, the attorney complimented the summer associate for his fine work and rewarded him by treating him to lunch at an expensive restaurant. The judge's clerks, however, found the case, and the judge queried the lawyers about how they could have missed it. Opposing counsel admitted he had been negligent in doing legal research on the matter, and the summer associate then recounted how he hid the case from the attorney. Is the attorney now subject to discipline for what the summer associate did?

B.No, because the attorney did not know about the associate's conduct at the time it occurred, or while submitting briefs, or even when the hearing began.

QUESTION 1. Soliciting campaign contributions. As part of its CJC, a state has adopted a rule prohibiting judicial candidates from personally soliciting campaign contributions. Disregarding this rule, a candidate running for a judicial seat in that state signed and sent a mass-mailing requesting that the recipients give financial support to her campaign. She also posted the letter online. The state bar brought a disciplinary action against the candidate for violating the no-solicitation rule. The candidate responded by claiming that the prohibition violated her First Amendment rights to free speech. When she seeks judicial review of her claim, how should the court rule?

B.The court will hold that the First Amendment allows the state to prohibit in-person solicitations by judges as well as campaign fund-raising letters.

QUESTION 7. Fact-checking on the Internet. A traffic accident led to litigation. At trial, one of the witnesses explained that dense tree foliage blocked the visibility of drivers from seeing traffic that was turning into their lane from a side street. After court adjourned for the day, the judge checked the accident scene on Google Earth and discovered that the entire area had retail and commercial development along both sides of the road. Few, if any, trees remained in the area. Did the judge violate the Code of Judicial Conduct?

B.The judge's Internet search violated CJC Rule 2.9(c) because this is an adjudicative fact.

QUESTION 2. My old friend, the opposing party. An attorney represents the defendant in litigation over a car accident. The plaintiff, who was driving the other car, was a childhood friend and neighbor of the attorney—they keep in touch. As the defendant's attorney has known the plaintiff since childhood, he calls the plaintiff, who has retained counsel as well, to see if they can resolve the case without going to trial. Is the attorney subject to discipline for calling his lifelong friend?

B.Yes, as a lawyer shall not communicate about the subject of the representation with a person the lawyer knows to be represented by another lawyer in the matter, unless the lawyer has the consent of the other lawyer or is authorized to do so by law or a court order.

QUESTION 6. Tie us over for the weekend. A client retains an attorney to handle a criminal matter. The client delivers a retainer check to the attorney on Friday afternoon. The retainer check will only cover the work the attorney anticipates he will begin and complete the following Monday. Because the following Monday is a banking holiday, if the attorney deposits the retainer check into his client trust account on Friday afternoon, he will not have access to the funds until Tuesday. The attorney deposits the retainer check into his business checking account and pays himself on Friday before the firm closes with those funds. Is the attorney subject to discipline?

B.Yes, attorneys shall deposit amounts paid in advance into a client trust account and the attorney shall not withdraw the funds until fees are earned or expenses are incurred.

QUESTION 20. The people want to know. An attorney represented a large corporation as a defendant in a toxic tort action. The matter had received little media attention and the corporate officers who retained the attorney emphasized the need to be discreet as long as possible, so that the pending litigation would have a minimal effect on stock prices. The representation necessitated that the attorney interview some of the employees involved in the incident that gave rise to the litigation, including some of the lowest-level unskilled laborers. A few of these individuals, as well as their co-workers whom the lawyer did not interview, asked the lawyer for details about what was happening with the case. The lawyer felt that they had a right to know about the case as it could affect the company, and their jobs, so he explained who the plaintiffs were, how strong the evidence appeared to be on each side, and the potential liability the company was facing. Could the attorney be subject to discipline for sharing this information with the company employees?

B.Yes, because a lawyer may not disclose to company employees any information relating to the representation except for disclosures explicitly or impliedly authorized by the organizational client in order to carry out the representation.

QUESTION 8. It's the little things. An attorney faced criminal sanctions for having over two thousand unpaid traffic and parking tickets, and several instances of failure to appear for jury duty. Could the attorney be subject to professional discipline for these minor offenses?

B.Yes, because a pattern of repeated offenses, even ones of minor significance when considered separately, can indicate indifference to legal obligation.

QUESTION 8. Who is behind this report? A client wants to sell a parcel of commercial real estate, and he hired an attorney to represent him in the matter. As part of the representation, the client asked his attorney to prepare a thoroughly researched opinion memorandum concerning the title of the property, for the information of a prospective purchaser and the purchaser's prospective lender. The attorney gave the title opinion to the client, who gave it to the prospective purchaser, who in turn submitted it to the prospective lender. The prospective lender received and reviewed the attorney's title opinion but was not aware that the lawyer who prepared the title opinion represented the seller of the property rather than the buyer. Could the attorney be subject to discipline for failing to disclose explicitly in the memorandum what party he represents and that he has a duty of loyalty and confidentiality to the seller?

B.Yes, because the title opinion should identify the person by whom the lawyer is retained, and should make this clear not only to the client under examination, but also to others to whom the results are to be made available.

QUESTION 13. Too much information. In anticipation of a round of settlement negotiations over a business partnership breakup, a client authorized his lawyer to disclose that the client was having personal financial troubles, but added that the lawyer should "leave it at that—don't elaborate too much." The lawyer was to attend the settlement conference without the client. At the settlement conference, when the other parties pressed the lawyer about why his client seemed so inflexible about a settlement amount for dissolving the partnership, the lawyer said that his client was having personal financial problems. Counsel for one of the other partners asked, "Like what? Perhaps the other partners could do something to help, and it would make it easier to resolve the partnership breakup." The lawyer then explained that everyone in the room must keep the following information completely confidential and went on to explain that his client was on the verge of bankruptcy due to a gambling problem. He also explained, in a hushed tone, that the client had even assigned his equity share in the partnership to a business rival of the partnership to pay off a personal loan. The others were shocked, with a mixture of sympathy for their partner's gambling problem, and alarm at the implications of their main market rival owning a significant share of the existing partnership. One of the other attorneys, however, checked the terms of the original partnership agreement, and informed the rest that equity interests in the company were unassignable without a majority vote of the other partners, making the assignment legally void. Could the lawyer be subject to discipline for the disclosures he made at the settlement conference?

B.Yes, because he went beyond what the client had authorized him to disclose, thus breaching his duty of confidentiality to the client.

QUESTION 2. A clean start in another state. An attorney obtained admission to the bar in New York and practiced there for two years. She worked for Big Firm, which has offices in five states and a few locations overseas. After her two years in the New York office, the firm transferred her to its office in San Diego, California. The attorney then applied for admission to the California bar under a reciprocity arrangement, and the state bar admitted her without making her re-take the bar exam. After practicing in California for three years, somehow the New York state bar learned that the attorney had made false statements on her original bar application about misdemeanor arrests during college. The New York bar informed the California state disciplinary authority about this problem, and the California state bar commenced disciplinary proceedings against the attorney in California. Can the attorney be subject to discipline in California for false statements made on a bar application in another state?

B.Yes, because if a person makes a material false statement in connection with an application for admission, it may be relevant in a subsequent admission application or disciplinary proceeding elsewhere.

QUESTION 19. Corporate counsel and the tortfeasor employee. An attorney represents a corporation. One of the corporation's delivery trucks, driven by a corporation employee, had a tragic accident with a school bus full of children, and many children died. The delivery truck driver suffered severe injuries, but survived, and spent three weeks recovering in the hospital. In preparation for the wrongful death lawsuits by the deceased children's families, the corporation's attorney visited the truck driver in the hospital and interviewed him about the accident. The attorney did not explain that he was not representing the driver, or that the driver should retain his own lawyer. The unsophisticated driver may have assumed that his employer's lawyer was also looking out for his (the driver's) interests. The driver made some incriminating admissions to the lawyer about being slightly intoxicated at the time of the accident and having been careless while driving. He also admitted that at the time of the accident, he had taken the corporate delivery truck off its assigned route to attend to some personal business for about twenty minutes. Could the attorney be subject to discipline in this case?

B.Yes, because in dealing with an organization's employees, the lawyer should explain the identity of the client when the lawyer should reasonably know that the organization's interests are adverse to those of the employee with whom the lawyer is dealing.

QUESTION 3. Honest ads. An attorney advertised in a local newspaper. His advertisement reads, "I never charge more than $200 per hour for any type of legal work, and for simple legal problems such as uncontested divorces or name changes, I charge even less." The attorney once had a particularly complicated, tedious case in another jurisdiction for which he charged $250 per hour, but he does not expect such a case to arise in the future, though his fee would be higher if it did. The attorney's advertisement fails to state that some other lawyers in the community charge much lower fees. The advertisement includes a pencil drawing of an unrealistically handsome, but generic-looking judge sitting behind the bench in a courtroom with a gavel in his hand. Could the attorney be subject to discipline for this advertisement?

B.Yes, because it is not true that he never charges more than $200 per hour.

QUESTION 2. Third-party billing. An attorney uses an outside billing service to track client billing and to send bills to clients each month. The attorney submits computerized timesheets to the billing company at the end of each workday showing how much time he spent on which tasks for which clients. The billing company calculates the monthly totals and sends detailed bills to clients on the attorney's behalf. The clients are not aware that the attorney uses an outside billing service until they receive their bills. Has the attorney violated his ethical duties to his clients?

B.Yes, because submitting the client names, time worked, and tasks involved constitutes a disclosure of confidential information for which clients must provide informed consent beforehand.

QUESTION 6. Supporting the cause. An attorney is active within a new political movement and she has represented several members of the movement, who faced arrest or criminal charges for protesting and picketing. The attorney learns that police have arrested one of the prominent leaders of the movement for trespassing on private property during a protest, but that the movement leader is already out on bail. In response, the attorney calls the leader and offers to represent him in his case free of charge, explaining that she has experience representing other members of the movement in similar cases. The leader agrees to have the attorney represent him on a pro bono basis. The attorney wants to represent the leader because she admires him, but also because she believes it will generate terrific publicity for the firm's practice. Was it proper for the attorney to make this telephone solicitation?

B.Yes, because the attorney did not charge for providing these legal services.

QUESTION 17. Trouble with the Feds on the horizon. An attorney worked for a corporation as in-house counsel. The attorney discovered that the Chief Financial Officer falsified the corporation's quarterly earnings report to prop up the firm's share price, as the CFO's compensation is partly in stock options. The attorney knows that these misrepresented earnings appeared in the filings to the Securities and Exchange Commission and will eventually result in severe regulatory fines or civil liability for the corporation. The attorney thus reasonably believes that the violation is likely to result in substantial injury to the organization. The attorney confronted the CFO, but this proved unfruitful, and then he proceeded up the corporate chain of command, eventually going to the Chief Executive Officer and the Board of Directors. The officers and directors refused to address the problem because they thought it would send their stock prices into a freefall and make the corporation vulnerable to a hostile takeover from corporate raiders. Would it now be proper for the attorney to become a whistleblower and reveal the problem to the relevant government authorities?

B.Yes, because the attorney has exhausted all other reasonable avenues to address the problem internally, so the lawyer may reveal information relating to the representation whether Rule 1.6 permits such disclosure or not.

QUESTION 2. The buffer. An attorney has a busy transactional practice and frequently must handle client funds, either for making commercial purchases, sales, leases, dispute settlements, or other transfers. The attorney faithfully deposits client money in a separate trust account and does not commingle the funds with his own, except that he deposits enough of his own money in the account to cover the monthly bank service charges. He also put $1,000 in the account when he opened it and left it there, as a buffer in case there were any accounting errors, so the clients would never experience inconvenience due to the account being inadvertently overdrawn. The attorney keeps complete, accurate records of all deposits and withdrawals for seven full years, after which he destroys the records to preserve client confidentiality. Is the attorney acting improperly?

B.Yes, because the attorney should not have deposited the $1,000 buffer from his own funds in the account.

QUESTION 18. The disbarred lawyer-agent. A firm specializing in sports law represented several professional athletes as clients. The state disciplinary authorities suspended and eventually disbarred one of the associates at the firm, but the disciplinary action did not implicate the rest of the firm (the lawyer's misconduct had occurred completely outside the scope of his duties there). The firm retained the disbarred lawyer as a sports agent for some of the athletes who were clients of the firm. The disbarred lawyer would draft contracts for the athletes and negotiate deals for the firm's clients with their sports teams or with companies seeking the athlete's product endorsement. Could the partners at the firm be subject to discipline for facilitating the disbarred lawyer in the unauthorized practice of law?

B.Yes, because the disbarred lawyer is engaging in the unauthorized practice of law by drafting and negotiating contracts for the firm's clients.

QUESTION 9. Stories over drinks with friends. An attorney represented a client, who was a defendant in a criminal prosecution. The client's trial ended in a conviction and a life sentence. After all possible appeals were complete, the attorney's representation of the client ended. The attorney sent the client a letter, which the client received in prison, explaining that his representation was now ending and providing a detailed accounting of all billing matters. No outstanding bills remained. Several years later, the attorney met with some former law school classmates at an alumni event, and they swapped stories over drinks about some of their cases over the years. The attorney mentioned the client, but only by first name, and explained how the guilty verdict felt like a failure on his part even though he knew the client was guilty because the client's friends and family members had all witnessed the crime and told the attorney privately what they had seen. Could the attorney be subject to discipline for disclosing confidential client information?

B.Yes, because the duty of confidentiality continues after the client-lawyer relationship has terminated.

QUESTION 11. Seller beware. A client intends to purchase a parcel of real estate and retained an attorney to analyze the seller's title to the property. The attorney requests information from the seller regarding the seller's original acquisition of the property and obtains additional information from the local tax assessors and title registry. The attorney concludes that the seller does not have clear title to the property and informs the seller of this opinion when the seller asks him about it. The seller forbids the attorney to disclose the information to the prospective purchaser of the property and insists that he showed the attorney his documents about the original acquisition of the parcel with the understanding that the attorney would not say anything unfavorable. May the attorney inform the prospective purchaser of his opinion about the title?

B.Yes, because the seller does not have a client-lawyer relationship with the attorney.

QUESTION 3. Accusing the Judges. A district attorney in a small city was on a losing streak, and she decided it must be the local judges who were at fault. She used her social media accounts to attack some of judges before whom she would regularly appear, for refusing to order reimbursement of the district attorney's costs of investigating cases and bringing them to trial. She also criticized the local judiciary for the longstanding backlog of criminal cases awaiting trial. At one point she even alleged that some local judges had connections with a drug cartel. These were all unsubstantiated claims, and the accusations were false. The named judges did not in fact have backlogs on their own dockets, and no one from the district attorney's office had submitted any formal requests for reimbursement of investigatory costs. The district attorney eventually faced disciplinary proceedings over this social media campaign against the judges, and her defense was that such statements had First Amendment protection and reflected her personal beliefs. Should the district attorney be subject to discipline for her public statements?

B.Yes, because the statements were false, and the district attorney did not have an objectively reasonable belief, at least for an attorney, that these statements were accurate and true.

QUESTION 12. Just go with it. An attorney represents a client, who wants to sell his business. A prospective purchaser has required from the client an evaluation of the business' solvency, detailing its current liabilities, potential liabilities, revenue, and assets. The client provides the attorney with documents pertaining to each of these issues, and explains to the attorney in confidence that he has often understated the earnings of the business to avoid paying taxes on the business profits. Now he is concerned that the prospective purchaser will undervalue the profitability of the business and refuse to pay an appropriate price to purchase it. He asks the attorney to adjust the earnings figures upward by 25 percent, the same amount by which the client falsely lowered them in the corporate records, to portray the business accurately to the potential purchaser. The attorney finds this objectionable and prepares a report based on what the records say regarding the earnings and gives the evaluation directly to the purchaser. When the client learns about this, he explains to the prospective purchaser over the phone what happened. Despite the low reported earnings, the purchaser pays the client's asking price for the business, because of the client's truthful representations over the phone. Could the attorney be subject to discipline for his conduct in this matter?

B.Yes, because under no circumstances is the lawyer permitted to knowingly make a false statement of material fact or law in providing an evaluation for a third party.

QUESTION 8. Fighting over the fees. A client met with an attorney for a free consultation, and explained that she had met with two other lawyers for consultations and that she planned to hire one of the three to provide the legal services necessary to set up her professional business. The attorney needed to make a good impression on the client, so he mentioned a few prominent accountants and physicians in town whom the attorney had represented and helped with incorporating their partnerships or practice groups. These former clients had never explicitly authorized the attorney to disclose his representation of them in these matters. The client hired the attorney, and the attorney provided the legal services necessary to set up her business. Unfortunately, a dispute arose between the client and the attorney over the fees, and this fee dispute turned into litigation between the attorney and the client. In order to support his claims and defenses in the fee dispute, the attorney had to disclose to the tribunal exactly what he did for the client and the complexity of the issues involved, which necessarily involved the disclosure of confidential information. Was it proper for the attorney to disclose this confidential information about the client merely to prevail in a fee dispute?

B.Yes, the lawyer may reveal information relating to the representation to establish a claim in a fee dispute between the lawyer and the client.

QUESTION 25. Non-compete agreements for associates. An attorney owns his own firm in a small town and hires an associate as a junior lawyer to help with the growing caseload. The employment agreement stipulates that the associate cannot practice law in that small town after leaving the attorney's firm. Which of the following is true regarding this arrangement?

C.Both the attorney and the associate are subject to discipline for such an agreement.

QUESTION 1. Emergency room patients. An attorney is dating a woman whose sister works as a nurse in a hospital emergency room. The attorney gives the nurse, his girlfriend's sister, a stack of his business cards and law firm brochures, and offers to pay her $200 for any clients who hire him because of her referrals, with the understanding that she will not refer patients to any other lawyers. The nurse recommends several patients per month to the attorney for representation in personal injury claims, and one or two per month actually hire the attorney to represent them. Is such an arrangement proper?

C.No, because a lawyer shall not give anything of value to a person for recommending the lawyer's services, with certain exceptions not applicable here.

QUESTION 5. Asking for advice. An attorney is representing a client who is a celebrity. The attorney is confused about whether he may publicly disclose information that he learned in confidence from his client if the information is already a matter of public record, and his research indicates there is a split of authority on this question. The attorney calls another lawyer who specializes in lawyer malpractice and disciplinary matters to seek advice. The other lawyer agrees to provide an opinion and to keep the conversation a secret. The attorney tries to use a hypothetical to explain the problem, but given the client's national reputation and celebrity status, the other lawyer knows immediately who the client is, and can easily surmise the nature of the confidential information. Is the attorney subject to discipline for disclosing confidential information about his client?

C.No, a lawyer may reveal information relating to the representation of a client to the extent the lawyer reasonably believes necessary to secure legal advice about the lawyer's compliance with the Rules of Professional Conduct, even without client authorization.

QUESTION 2. Don't worry. . . . A client repeatedly calls an attorney to discuss her pending divorce case. The client wants above-guideline child support, alimony, and a large percentage of the estate, even though the parties have only been married two years. The attorney has continuously given his honest opinion about what he believes the client is eligible to receive, and what he believes she may receive in the divorce based on his experience. The client has recently become angry with the attorney because she is unhappy with his opinion. She has even asked, "Are you working for me or my husband?" In an effort to keep the client happy, the attorney begins to tell the client what he believes she is eligible to receive when she asks, but simply states "the court will decide" when the client asks the attorney what he believes she will receive. Are the attorney's actions proper?

C.No, an attorney should give his honest opinion about the case when asked, even if the opinion is unsatisfactory to the client.

QUESTION 13. The former mediator. An attorney, who often serves as a court-appointed mediator, received an appointment to mediate the divorce case between a husband and wife. The case settled in mediation and the divorce became final soon after. A year later, the husband sought to retain the attorney to represent him in a modification suit against his ex-wife. The attorney accepted the case and sent a letter to the ex-wife advising her that her ex-husband had retained him to represent him in a modification suit. Are the attorney's actions proper?

C.No, an attorney who previously served as a third-party neutral is required to obtain informed consent, confirmed in writing, from all parties to the proceeding prior to representing a party in a suit connected to the previous matter.

QUESTION 16. Working with Washington lawyers. An attorney in a state that has adopted the Model Rules in their current form enters into a fee-sharing agreement with a lawyer admitted in Washington, D.C., which permits fee sharing with non-lawyers and multidisciplinary practices. They collaborate on a case and divide the fees as agreed. The attorney from the Model Rules state is aware that the other attorney will share his part of the fees with non-lawyers in the D.C. office; in fact, the D.C. lawyer's firm has accountants who hold an ownership share in that firm. Is the non-D.C. attorney subject to discipline for indirectly sharing legal fees with non-lawyers, given that he practices in a state that forbids fee sharing with non-lawyers?

C.No, as long as the first attorney shares fees only with another attorney, it does not matter if the other attorney shares fees with non-lawyers as permitted by his home jurisdiction.

QUESTION 9. Punished in both states. An attorney had a license to practice law in two jurisdictions—his home state where he lived and had his main office, and a neighboring state where he represented several clients each year. The attorney committed serious professional misconduct in his home state and received a public reprimand from the state disciplinary authorities. All the conduct took place in his home state, the client resided in the state, and the representation took place entirely within his home state. The lawyer's conduct would have violated the rules in either of the jurisdictions where he had a license to practice law, because it involved commingling client funds with his own money, and the states had nearly identical rules concerning this activity. After the attorney received a public reprimand in his home state, where the misconduct occurred, the state bar disciplinary authority in the neighboring state (where he also practiced) then commenced disciplinary proceedings against him as well. In the end, the neighboring state bar suspended his license for six months in that state, a much more severe sanction than the public reprimand he received in his home state, where the misconduct actually occurred. The attorney claims that the neighboring state bar has no jurisdiction over conduct that occurred entirely outside of the state. He also objects that the second punishment raises double jeopardy concerns. Is the attorney correct?

C.No, because a lawyer may be subject to the disciplinary authority of two jurisdictions for the same conduct, and may receive different sanctions in each state.

QUESTION 9. I won't do it. On his website, an attorney explains that he handles most areas of personal injury law, and then displays in large, bold letters: "I DO NOT REPRESENT CLIENTS IN CRIMINAL MATTERS OR DIVORCE MATTERS—PLEASE FIND ANOTHER LAWYER IF YOU ARE FACING CRIMINAL CHARGES OR NEED TO LEAVE YOUR SPOUSE." Is it improper for a lawyer to make such a statement in his website or advertising materials?

C.No, because a lawyer may communicate the fact that the lawyer does or does not practice in particular fields of law.

QUESTION 6. Under orders. An attorney represents a client before an Administrative Law Judge in a regulatory enforcement matter. The Administrative Law Judge orders the attorney to disclose whether the client was informed by counsel about the regulatory requirements in question before the violation occurred. The client forbids the attorney to answer the question. The attorney initially objects, but the Administrative Law Judge insists. Could the attorney be subject to discipline for disclosing such confidential client information to the Administrative Law Judge?

C.No, because a lawyer may comply with an order to reveal information relating to the representation of a client by a court or by another tribunal or government entity claiming authority pursuant to other law to compel the disclosure.

QUESTION 5. Just a hunch. An attorney suspects that another lawyer in his firm has violated the Rules of Professional Conduct in a rather serious matter, but has no first-hand knowledge of the situation—his suspicion rests on the fact that the other lawyer seems to be acting paranoid and evasive, and a number of strange coincidences have occurred in his cases. Does the attorney who suspects something seriously wrong is afoot have a duty to report the other lawyer to the state bar disciplinary authority?

C.No, because he does not have actual knowledge of the violation.

QUESTION 3. Discussing a case with the partners. An attorney is a partner in a seven-lawyer firm. A client retained the attorney to handle his workers' compensation matter. The attorney did discuss with the client that he would normally disclose to the other partners in the firm some of the details about his cases and clients, and the client expressly forbade the attorney from telling anyone in his firm anything about his case. Nevertheless, at the weekly meeting of the partners, as everyone discussed their pending cases, the attorney explained the client's case and solicited input from the partners. One partner had an ingenious suggestion that was helpful to the client's case. The attorney mentioned to the client in their next phone call that one of his partners had suggested a strategy that could turn the case in the client's favor. The client was upset that the attorney had discussed the case with anyone else. Was it proper for the attorney to discuss the case with the others at the firm?

C.No, because lawyers in a firm may not disclose to each other information relating to a client of the firm if the client has instructed that particular information be confined to specified lawyers.

QUESTION 7. After the storm. After a hurricane damaged hundreds of homes in a southeastern state, an attorney, who practices in that state, sent letters to a dozen homeowners in the affected area offering to represent them in their insurance claims arising out of the storm damage. Each letter was handwritten and personalized, and the attorney addressed each envelope by hand so that recipients would perceive it as a personal letter and would be more likely to open it and read it. Nothing in the letters was coercive or threatening. Were the attorney's actions improper?

C.No, because such communications can be mailed to inform the public about the need for legal services, and about the qualifications of available lawyers and law firms, without subjecting the public to live person-to-person persuasion that may overwhelm a person's judgment.

QUESTION 8. New work, old clients. An attorney specializes in employment law, especially employer-provided benefits, as well as healthcare law. After Congress passes sweeping legislative reforms for the regulation of employer-sponsored healthcare plans, the attorney called her former business clients offering to help them sort through the changes in employee benefit plans that the new laws would require. These individuals had not initiated the contact, nor had they indicated that they did not wanted to be contacted by the attorney. Could the attorney be subject to discipline for calling these former business clients?

C.No, because the attorney called only to former clients.

QUESTION 21. Exclusively federal practice. An attorney has a firm in a state in which the attorney lacks a license to practice law. The attorney's legal work, however, consists entirely of representing local inventors before the United States Patent and Trademark Office in Washington, D.C., either by correspondence or by traveling to appear there in patent proceedings. A relevant federal statute states that non-lawyers may represent patent applicants before the USPTO. The attorney does no other legal work for clients—if clients need representation for family law matters, employment matters, incorporating businesses, or personal injury suits, the attorney refers them to outside counsel. All the attorney's clients, however, are located in the state where the firm has its office, and the attorney is unlicensed there. Is the attorney subject to discipline?

C.No, because the attorney is providing services authorized by federal law, which preempts state licensing requirements.

QUESTION 3. When the client disputes the fee. An attorney represented a client in a contention litigation matter, at the end of which the attorney received a settlement check for an agreed-upon amount from the opposing party ($100,000). The client had agreed to the amount but was unsatisfied and blamed the lawyer for the disappointing settlement amount. The attorney called the client to inform her that the check had arrived, and explained that he would forward the amount minus his fees and the expenses, which constituted half of the amount (the jury consultants and experts in the case had turned out to be very expensive). The client was furious and said that the expenses should have been included in the attorney's contingent fee, and that the attorney was not entitled to the original contingent fee in any case because the case had never gone to a verdict and had settled for a mediocre amount. Pursuant to their retainer agreement, the client and attorney agreed to schedule arbitration over the disputed fees and expenses as soon as possible, which realistically would be three or four months later. In the meantime, the attorney kept the money in the client trust account until they could resolve the dispute. Was this proper?

C.No, because the attorney should have paid the client $50,000 immediately and held only the remainder until the dispute was resolved.

QUESTION 1. What's in a name? A government entity provides grants to a legal aid office that represents indigent individuals. The government entity requires reporting of the names of clients, brief factual summaries, and the type of representation involved for all matters where the government entity's funds provided the financial support for the representation. This information helps ensure that the funding is going to its intended purposes. An attorney works for the legal aid office. Most of his clients are uneducated and unsophisticated, so he does not explain to them that he must disclose their information. Is it proper for the attorney to make the required disclosures to the funding agency without obtaining the clients' informed consent?

C.No, because the client names, basic facts, and types of cases are confidential information, and require client authorization for disclosure.

QUESTION 14. The good deal. A husband and wife are attending court-ordered mediation with an attorney, who is serving as the neutral mediator. The husband has retained counsel, but the wife has not. During mediation, the wife asks the mediator for his advice, and asks whether he believes that the husband's offer is a "good deal" for her. The attorney explains that his position as mediator only allows him to facilitate the negotiating process. The wife continues to seek the attorney's advice about the settlement proposals the husband makes. The attorney finally tells the wife what she is getting is a decent percentage of the estate and that he believes it to be a "good deal" for her. The attorney also informs the wife again that he does not represent her and that anything he says is merely general information, not legal advice. Are the attorney's actions proper?

C.No, the lawyer should decline to advise her, and instead explain more carefully the difference between the lawyer's role as a third-party neutral and a lawyer's role as one who represents a client.

QUESTION 2. The major campaign contributor. In a state with elected judges, a certain candidate (the challenger) for the state supreme court sought to unseat an incumbent justice. The president of a coal company in that state set up a nonprofit organization to advocate for the removal of the incumbent justice, and contributed millions of dollars to this entity, which ran an extensive advertising campaign criticizing the incumbent justice. Even though the president did not give this money directly to the challenger, the campaign was effective, and the challenger won the election and became a state supreme court justice. The millions contributed to the nonprofit by the coal company president exceeded the total amount of all other campaign contributions in that election. Shortly thereafter, a lawsuit involving the coal company, which had been working its way through the courts, came up for review by that state supreme court. The justice who had won the election joined a 3-2 majority ruling in favor of the coal company in a case worth more than $50 million. The other party had petitioned for recusal by the newly seated justice, given that the coal company's executive had contributed millions of dollars to help the justice win the election, but the justice did not recuse himself, explaining in a lengthy memorandum why he thought he could render an unbiased decision in the case. Was it proper for the new justice to participate in the decision in this case?

C.No, because the coal company's disproportionate campaign contributions created an impermissible appearance of bias, even if the justice was in fact unbiased.

QUESTION 7. Whether to appeal. An attorney represented a client in tort litigation against a pharmaceutical company over injuries allegedly resulting from one of the company's drugs. During a pretrial hearing about the admissibility of certain evidence, the court ruled against the attorney and ordered that the evidence was inadmissible at trial. The attorney then contacted a reporter from a prominent newspaper and gave him a lengthy interview explaining the case, discussing the upcoming trial, and giving the reporter the very evidence that the court had held should be inadmissible at the trial. The newspaper ran the story on the same day that jury selection began for the trial. Opposing counsel moved to disqualify the attorney due to misconduct in the matter, that is, the public disclosure of the inadmissible material in an attempt to taint the jury pool. The court agreed to disqualify the attorney on the eve of the trial. Another firm was already representing the client as co-counsel, so that firm agreed to continue with the trial work alone. The attorney filed an interlocutory appeal, which he lost at the appellate court and appealed to the Supreme Court. Delaying the trial with this interlocutory appeal was clearly against the client's interest, but it was necessary for the attorney to continue to handle this big case. Is it proper for the attorney to recommend to the client that they appeal his disqualification, if it is not clearly in the client's interest to do so?

C.No, because the decision to appeal should turn entirely on the client's interest.

QUESTION 12. The letter to the physician. An attorney represents Conglomerate Corporation. One of the company's employees died in a bizarre accident at the worksite, and Conglomerate asked the attorney to investigate the cause and the company's potential liability. As part of her investigation, the attorney wrote a letter to a physician he knew personally. The attorney's letter set out the circumstances of the employee's death and requested the physician's professional opinion as to the cause. The letter also explained that the attorney was preparing for a "possible claim" by the employee's estate for worker-compensation benefits, and potential wrongful death damages. When litigation ensued, the deceased's estate later requested a copy of the letter that the attorney sent to the physician. Must the attorney disclose the letter?

C.No, because the letter is non-discoverable work product.

QUESTION 22. The dreaded conversation. An attorney served as in-house counsel for a corporation, and uncovered illegal actions taken by a particular senior manager (not the Chief Executive Officer or any comparable officer or director, but an individual with decision-making authority and several direct subordinates in the organization). The senior manager had a reputation for being arrogant and unreasonable, and he and the attorney had clashed on several occasions and were barely on speaking terms. At the same time, the senior manager was exceptional in his area of expertise and was an asset to the company despite his unpleasant demeanor. The attorney summoned the nerve to confront the senior manager about the problem as graciously as possible, and the senior manager's initial response was to be dismissive, saying that he was unaware of any laws or regulations that he might have violated. The attorney walked away from the conversation discouraged and planned to take the matter up with the corporate officers, and potentially the Board of Directors. Before doing so, he reconsidered and returned to the manager, and patiently explained to him the relevant laws and regulations that the manager had violated. The senior manager begrudgingly accepted the attorney's advice and took all necessary measures to rectify the wrongdoing and prevent any long-term repercussions. The senior manager also insulted the attorney, called him incompetent for not bringing up the matter earlier, and suggested that the attorney's incompetence was due to the attorney's ethnic background. Could the attorney be subject to discipline for not referring the matter of the illegal actions to a higher authority in the corporation?

C.No, because the manager took the lawyer's advice.

QUESTION 9. What's missing here? An attorney represents a client before a government agency that enforces securities regulations. As part of the representation, the attorney must prepare an opinion concerning the legality of the securities registered for sale under the securities laws, for submission to the government agency, which requires such reporting. The client authorizes the attorney to prepare the written opinion but insists that the attorney exclude any mention of a particular business loss the client's company incurred recently, in order to avoid upsetting the shareholders. To preserve the client's confidential information, the attorney prepares the written opinion without the information the client asked him to withhold. The report does not mention that it excludes some unfavorable information. The attorney prepares the written opinion and gives it to the client, who submits it to the agency. Is it proper for the attorney to follow the client's instructions in preparing this report?

C.No, because when a lawyer's report categorically excludes certain issues or sources, then the lawyer must describe in the report any such limitations that are material to the evaluation in the report.

QUESTION 3. Straight to the top. While preparing for a trial over workplace discrimination, the plaintiff's attorney contacts the owner and chief executive officer (CEO) of the defendant corporation and interviews her about the day-to-day operations of the company and the chain of command for addressing personnel complaints. The owner/CEO is not personally involved in the matter of the pending litigation—she actually never met the plaintiff warehouse worker who claims to be the victim of workplace discrimination, she is not on the witness list to testify at trial, and nobody has suggested that she was responsible for the wrongdoing. Even so, she has the power to settle the case or stipulate to a judgment amount, so the plaintiff's attorney talks to her directly. The attorney does this without permission from the corporation's lawyer, whom the company's general counsel hired; general counsel is an employee three steps below the CEO in the organizational chart. Was this communication by the plaintiff's attorney proper?

C.No, the rules prohibit a lawyer from communicating with the constituent of a represented organization (opposing party) who has authority to obligate the organization with respect to the matter.

QUESTION 21. Going up the chain of command. An attorney is in-house counsel for a large international corporation and has daily contact with higher-level executives and managers. One day, a senior executive mentions casually to the attorney that he has offered lucrative stock options, worth millions of dollars, to a foreign government official who has agreed to give the firm an exclusive contract to provide certain goods and services to the foreign state. The executive seems to think this is normal and good for the company, but the attorney believes it constitutes bribery of foreign officials, which would violate the Foreign Corrupt Practices Act, and could subject the company to enormous fines and penalties. The attorney explains her concerns to the executive, including her concern that he could face personal criminal charges in addition to bringing liability on the corporation, and she reminds him that she represents the corporation, not him personally. The executive is dismissive of her concerns, even though she approaches him several times about the matter. How must the attorney proceed?

C.She should approach the executive's immediate corporate superior, advising those next up the chain of authority to stop the transaction and take appropriate actions against the executive involved.

QUESTION 5. The mutual fund investments. A judge has investments in a Felicity Mutual Fund, a large mutual fund that invests in "blue chip" stocks of well-established, large corporations that offer lower risk than other stocks. A case arises on the judge's docket involving Intec, a large company whose stocks are currently among those in the Felicity Mutual Fund's portfolio. Lawyers for the opposing party file a motion for the judge to recuse or disqualify herself because of these investments. Which of the following describes what the Code of Judicial Conduct would require of the judge in this circumstance?

C.The judge does not need to disqualify herself, because the stock holdings in Intec are indirect investments made through a mutual fund.

QUESTION 6. It starts with a phone call. A business owner hires an attorney hoping to enforce a non-compete agreement against a former employee at their technology firm. According to the client, a rumor started going around just this past week that the former employee either had started his own consulting practice nearby or was working for a nearby competitor. Each scenario, if true, could violate the non-compete agreement. The employee left the client's company on bad terms about three weeks ago. The client provides a copy of the non-compete agreement, and speculates that the former employee may have forgotten about the agreement (which he signed fifteen years ago), and may even be oblivious to the fact that he is violating it. The attorney decides that the first step is to call the former employee and ask him whether he has found another job yet or has started his own business. The attorney assumed that the former employee would not have retained counsel yet to challenge the non-compete agreement, given the client's comments about him, and the brief time since the events had unfolded. The former employee answers the phone, and after the attorney identifies himself as counsel for the business client, the former employee explains that he has started his own rival company, and that he believes the non-compete agreement is invalid under state law. When the attorney asks why it would be invalid, the former employee says that his own lawyer told him that recent changes in state law make the previous agreement void, and that they plan to challenge the agreement in court. The attorney asks him to have his own counsel contact him so that they can discuss potential settlement for the dispute. Has the attorney acted properly?

C.Yes, as the prohibition on communications with a represented person only applies in circumstances where the lawyer knows that the person is in fact represented in the matter to be discussed, and this means that the lawyer has actual knowledge of the fact of the representation.

QUESTION 7. Simple battery. A criminal court found that an attorney had engaged in domestic violence against his partner, and convicted the attorney of misdemeanor-level battery, for which he served a six-month term of probation. Could the attorney be subject to professional discipline as well?

C.Yes, because crimes of violence indicate a lack of the character traits required for law practice.

QUESTION 17. The marketing guru and his contract. The attorney hires a nationally known Internet-marketing specialist, a tech guru, to help develop the firm's reputation and attract new clients. The Internet specialist has made millions on previous tech startups, while the attorney is not well known and has been practicing for only two years. The tech guru demands certain terms in the contract that require the attorney to confer with the tech guru about accepting clients that were former clients of the tech guru, to avoid conflicts of interest. The attorney must also clear any litigation positions, approaches, or strategies that pertain to intellectual property or Internet marketing liability with the tech guru, to avoid positions that would jeopardize the guru's other business. Is the attorney subject to discipline for this arrangement?

C.Yes, because in this case, a non-lawyer has a contractual right to direct or control the professional judgment of the lawyer.

QUESTION 15. Profit sharing and votes for paralegals. An attorney practices personal injury law, representing plaintiffs on a contingent fee basis. The attorney employs a paralegal to assist with preparing documents for litigation. The paralegal's salary arrangement is 10 percent of the firm's total net revenue each year. In years when the attorney wins several large cases, the paralegal receives higher wages, and in years when the attorney has no big wins, the paralegal receives almost nothing. The paralegal does not bring clients to the firm, but does participate in judgments about which clients to represent, how to structure contingent fee arrangements, and how much to seek in damages after a verdict, as these matters directly affect the paralegal's income as well as the attorney's. Could the attorney be subject to discipline for this arrangement?

C.Yes, because this paralegal is sharing profits with the attorney, and is able to influence the professional judgment of the lawyer under this arrangement.

QUESTION 20. Recent graduates awaiting their bar results. An attorney hires three new associates upon their graduation from law school in a neighboring state. The associates passed the bar in the neighboring state, but they are still unlicensed in the attorney's state. The associates confine their work to conducting research, reviewing documents, and attending meetings with witnesses in support of the attorney, who is responsible for all the litigation. The research done by the associates, however, is far beyond the capabilities of a paralegal or a typical law student associate. Is the attorney subject to discipline for this arrangement?

D.No, because the associates merely conduct delegated work under the attorney's supervision, for which the attorney is ultimately responsible.

QUESTION 6. The television commercial. A local furniture store decided to make a low-budget television commercial telling customers that the store was having its best clearance sale ever. The owner of the store lived next door to a state trial judge, who had held his judicial seat for twenty years and was well-known in the community. At the request of the furniture store owner, the judge agreed to appear in his judicial robe in the television commercial, recommending the store and its owner to the viewers. Could the judge be subject to discipline for this commercial, under the Code of Judicial Conduct?

D.Yes, because it violated the Code of Judicial Conduct for a judge to appear in judicial robes in business advertising.

QUESTION 16. Clash of the titans. An attorney worked for a corporation as its in-house counsel. Hostility breaks out between the Chief Executive Officer (CEO) and the Chief Financial Officer (CFO), with each threatening to sue the other over allegations of slander, libel, trespass to chattel, and so on. Does this personal clash between top managers present the attorney with a conflict of interest?

D.No, because a lawyer employed by an organization represents the organization acting through its duly authorized constituents, so the lawyer represents neither of these officers individually.

QUESTION 9. Court records and archives. During the sentencing phase of a criminal trial, the judge grew concerned that the prosecutor expressed uncertainty about the number of the defendant's prior convictions. The judge instructed her clerk to search the court's electronic docket and archives to determine the defendant's criminal case history, and to expand the search to other jurisdictions if their records were accessible from the clerk's computer. The clerk discovered several other judicial proceedings involving the defendant, and carefully noted how many of these had resulted in convictions. The clerk also noted pending cases awaiting sentencing that might prompt the judge to wait a little longer before rendering a decision, so that the judge could consider those cases as well in her own sentencing. Finally, the clerk noted some cases with sealed records, which the clerk was able to access and review and report to the judge. The judge intended to follow appropriate procedures for taking judicial notice of any of these facts that might bear on the defendant's sentence. Was the clerk's search proper, according to the Code of Judicial Conduct?

D.It was appropriate for the clerk to review the court dockets and archives for the defendant's other proceedings but not to review the court records that were under seal.

QUESTION 5. Risky business. A client hires an attorney to help with the legal documents necessary to liquidate most of his investments so that he can use the cash to fund a new business venture. The client explains that he plans to quit his regular job and start a new career working from home as a "day trader," buying and selling stocks online every day in hopes of making large profits. The client has no experience or training in finance or investments, but he attended a seminar that featured testimonials from others who claimed to have made millions as day traders. The attorney thinks this is a foolish idea, but the client does not ask the attorney for his advice. Does the attorney have an ethical duty to caution the client against his seemingly reckless decision?

D.No, because a lawyer is not expected to give advice until asked by the client.

QUESTION 6. Mandatory reporting. An attorney works at a large firm and sees almost daily violations or potential violations of the Rules of Professional Conduct, though nearly all of them are minor and cause no harm or injury to the clients, third parties, or anyone else. For example, some lawyers represent co-defendants in cases where conflicts could arise at some point in the litigation, though the cases always seem to settle before any such scenarios develop. In other instances, certain lawyers seem to do minimal research on their cases or sometimes neglect client matters for weeks at a time, but again there has not been a case that was particularly serious. Does the attorney have a duty to report these violations to the state disciplinary authority?

D.No, because a lawyer must report only those offenses that a self-regulating profession must vigorously endeavor to prevent; if a lawyer were obliged to report every violation of the rules, the failure to report any violation would itself be a professional offense.

QUESTION 4. My client is a suspicious character. A client hired an attorney to represent him in a simple real estate matter. When the attorney asked some standard questions about the financial arrangements for the sale and purchase of the property, the client was somewhat evasive on a few points, but provided the information necessary to complete the legal work for the transaction. The attorney also heard from a friend that the client frequently cavorted with prostitutes. The attorney finds the client suspicious and has many unanswered questions, but none surrounding the transaction that occasioned the representation. Does the attorney have an ethical duty to inquire into the affairs of a suspicious client?

D.No, because a lawyer ordinarily has no duty to initiate investigation of a client's affairs or to give advice that the client has indicated is unwanted.

QUESTION 11. The campaign volunteer. An attorney volunteered for a judge's reelection campaign because he hoped to receive court appointments. He drives the judge from campaign stop to campaign stop without receiving any compensation for his time or effort. The judge wins re-election, and then shows his gratitude to the attorney by frequently appointing him to represent indigent defendants at the state's expense. The appointments turn out to be lucrative and to generate substantial fees for the attorney. Could the attorney be subject to discipline for soliciting funds for a judge with such self-interested motives?

D.No, because for purposes of this rule, the term "political contribution" does not include uncompensated services.

QUESTION 1. That was then, this is now. An attorney has been practicing for five years, but on her application to the bar five years earlier, she had stated that she had attended a particular private high school, when in fact she had attended a public high school. An unhappy client recently filed a grievance against the attorney, which was frivolous, but the state disciplinary authority had to conduct a routine, preliminary inquiry into the matter in order to make a determination that the complaint merited dismissal. The disciplinary board member assigned to the case had attended the elite private high school from which the attorney claimed to have graduated, and made a mental note of the attorney's high school when he did a cursory review of her bar admission files. He thought it was strange that he had never seen or heard her name at any alumni or reunion functions, as they had supposedly graduated the same year and the classes were small. On a hunch, the board member checked the alumni lists for the school and discovered that the attorney had lied on her application to the bar five years earlier. When asked about this issue, the attorney said she could not be subject to discipline now for the misstatement she made several years ago, and that the board lacked jurisdiction because it was unrelated to the current grievance complaint. Is she correct?

D.No, because if a person makes a material false statement in connection with an application for admission, it may be the basis for subsequent disciplinary action if the person is admitted.

QUESTION 1. Protecting the client's feelings. Halfway through a trial, an attorney can tell that his client is going to lose. The opposing party successfully impeached the attorney's only favorable witness, and the judge has already told the parties that he plans to follow the state's model jury instructions for this type of case, which effectively preclude the legal theory that the attorney had made the centerpiece of his case. During a lunchtime break, the client turns to the attorney and tearfully asks if they still have any chance of winning. The attorney does not want to make her cry and feels very awkward about the situation, so to spare her feelings, he assures the client that they still have a good chance of prevailing. The attorney is representing the client on a contingent fee basis, so he knows it will not cost the client any more in legal fees to finish the trial. At the same time, there is still an open settlement offer on the table from the other party, albeit a very small, unsatisfying settlement, which the client could accept at any time if she wants to terminate the litigation. Is it proper for the attorney to feign confidence to protect his client's feelings?

D.No, because in representing a client, a lawyer shall render candid advice.

QUESTION 4. Client dispute with a third-party creditor. An attorney received from the opposing party $150,000 as a settlement for the attorney's client. Before the attorney could disburse the funds to the client, a third-party judgment creditor with a court-ordered lien against the client contacts the lawyer demanding disgorgement of the client's funds to satisfy the amount of the judgment, from a matter in which the lawyer did not represent the client. The client instructs the attorney to give the money to the client immediately and not to give anything to the third-party judgment creditor. Preliminary inquiries suggest that the third-party judgment creditor has a valid court order to execute on the client's assets. The attorney did as the client instructed him to do, disbursing the funds promptly to the client, and informed the judgment creditor to take up the matter with the client directly. Did the attorney act properly?

D.No, because in this type of situation, the lawyer must refuse to surrender the property to the client until the claims are resolved.

QUESTION 18. Now that we're already in trouble. A large corporation was under investigation by a government regulatory agency over possible violations of securities law. The corporation hired an attorney to represent it in the matter and authorized the attorney to make a full internal investigation to discover the merits of the accusations. The attorney discovered that a high-level manager had falsified quarterly earnings reports, a clear violation of the law that could expose the corporation to devastating sanctions and civil liability. The attorney confronted the officer involved, but this proved unfruitful, and then he proceeded up the corporate chain of command, eventually going to the Chief Executive Officer and the Board of Directors. The officers and directors refused to address the problem because they thought it would send their stock prices into a freefall and make the corporation vulnerable to a hostile takeover from corporate raiders. Would it now be proper for the attorney to become a whistleblower and reveal the problem to the relevant government authorities?

D.No, because the attorney has a duty of confidentiality to the corporation, and the corporation hired the attorney to defend the organization against a claim arising out of an alleged violation of law.

QUESTION 11. Responsibility in hiring. A small firm employs several associates who work under the supervision of the partners, as well as three clerical staff. The most recently hired associate has a complicated situation with his license to practice law. The associate graduated from an accredited law school, successfully passed the state bar exam, and applied for admission to the bar, believing he had met all the eligibility requirements. He had no criminal record or history of academic misconduct, or any other problems meeting the traditional character and fitness requirements. The state bar approved his application and he attended his swearing-in ceremony. The state legislature, however, had recently passed a statute creating the option of a legislative veto for lawyers seeking admission to practice law in the state. The sponsors of the enactment had stated that its purpose was to prevent the grown children of illegal immigrants from becoming lawyers, even though the bar applicant might be a United States citizen "just because they happened to be born here." The associate was born in Arizona one month after his parents had moved there illegally from a country in Central America. A staff member of the relevant legislative committee flagged the associate's name from a list of recent bar licensees, along with three others in his situation. During a special session of the legislature, the state legislature exercised the equivalent of a legislative veto, narrowly passing a special act that permanently disbarred the associate and the others for the sole reason that their parents were illegal aliens. The associate received official notice of his disbarment from the Office of Legislative Counsel, not from the state bar. This occurred one week after the associate's swearing-in ceremony by the state bar, and two days after he started working at the firm. A notice of the disbarments appeared in the next issue of the state bar journal, but most of the firm was unaware of the situation, except for one managing partner in whom the associate had confided. Could the partners at the firm be subject to discipline for employing the associate as an attorney, despite challenging any such discipline in court?

D.No, because the courts have inherent power to regulate the legal profession, and the legislature's action could not survive a court challenge.

QUESTION 11. The internal inquiries by corporate counsel. Executives at a large pharmaceutical manufacturer discovered that one of the company's foreign subsidiaries had paid bribes to local government officials to obtain lucrative government contracts. During the internal corporate investigation of the matter, the corporation's directors asked their in-house General Counsel to send written inquiries to dozens of the corporation's foreign managers about whether similar payments or bribes were occurring elsewhere. After reviewing the responses to these inquiries and following up with phone calls and meetings, the corporate directors self-reported any questionable transactions to the Internal Revenue Service (IRS). The IRS then demanded to see all the original written responses to these internal inquiries. The corporate directors and General Counsel refused, claiming that the information was privileged. Should the government (the IRS) be able to compel production of these documents with a court subpoena?

D.No, because the information was provided at the request of the corporate directors to the in-house attorney, and the information was work-related, and was necessary for obtaining legal advice.

QUESTION 12. Certified long ago. An attorney entered into an exclusive reciprocal arrangement with a local advertiser, in which the lawyer agreed to advertise her firm solely through that advertising agency, for normal market rates, and the advertising agency agreed not to take any other law firms in the region as clients. The agency ran newspaper and billboard ads based on information supplied by the lawyer, which included a claim that the lawyer was a certified specialist in immigration law, as certified by the state bar association itself. While the certification was legally valid, the lawyer had not handled a single immigration case since obtaining the certification some time ago, and has not kept abreast of major changes in immigration law in the meantime. Given the extensiveness and complexity of recent changes in the law, the attorney would no longer be able to provide competent representation in immigration matters without extensive research and study. Last week, the lawyer happened to drive by two cars on the shoulder of the road that had been in a minor collision. The attorney pulled over, got out of her car, and approached one of the drivers who were waiting for a tow truck. Handing the driver her business card, the attorney offered to represent her in any litigation over the accident, and assured her that she would charge a fair rate, and the driver gladly took it and said she might have seen one of the lawyer's advertisements. The attorney was not aware at the time that the driver was an immigrant from Europe who was in the country on a temporary work visa. The driver visited the attorney a few days later for an initial consultation, but decided not to retain the lawyer for representation because the driver's insurer had already settled the matter. Was it proper for the lawyer to offer to represent the driver in this way?

D.No, because the lawyer approached the driver in person at the scene of the accident and offered to represent her.

QUESTION 4. A chance encounter between adversaries. The plaintiff and the defendant in a lawsuit run into each other in the supermarket and start discussing their case without their lawyers being present. Both have been shocked at the mounting litigation costs, and at how long the case has gone on. The plaintiff volunteers to withdraw his case if the defendant will withdraw his counterclaims and pay whatever filing fees are involved in such a voluntary dismissal. Later, when each party reports this to their respective lawyers, the plaintiff's lawyer is very upset. The plaintiff mentioned that the defendant said his own attorney (defense counsel) had helped give him the idea by asking at their first consultation, "Why haven't you and the plaintiff simply resolved this on your own, without resorting to litigation?" The plaintiff's lawyer reports the defendant's attorney for misconduct, claiming that opposing counsel merely used his client as an agent to communicate with the plaintiff without the latter's lawyer present. Is the defendant's attorney subject to discipline, based on these facts?

D.No, because the parties to a matter may communicate directly with each other without their lawyers being present or consenting to the conversation.

QUESTION 3. Attending the lawyer's party. A judge attended a social event—a birthday party—with around thirty other individuals at the home of a lawyer who had been a longtime acquaintance. The lawyer who hosted party in his home had a pending case before the judge that was awaiting a pretrial procedural ruling. Apart from exchanging brief greetings when the judge first arrived, the lawyer and the judge did not engage in any substantial conversation during the party, and neither the judge nor the lawyer discussed the pending case itself with anyone at the party. A month later, the judge ruled in favor of the party represented by the lawyer. Was it improper for the judge to attend the social event, given that it occurred at the home of a lawyer who had a case pending before the judge?

D.No, because there were no ex parte communications about the pending matter, and it was a social event attended by thirty people.

QUESTION 10. They will find out eventually. A government agency contacts an attorney, who works as in-house counsel for Corporation, and requests a report about some of Corporation's activities that come under the agency's regulatory jurisdiction. As the attorney begins to investigate the matter to prepare the report, he learns that the information requested by the agency will subject Corporation to significant regulatory enforcement sanctions, and if the information became public, would adversely affect Corporation's share price. At this point, the agency has not issued a subpoena and compliance with the request is voluntary, although the agency could compel the disclosure eventually. The managers and directors of Corporation instruct the attorney not to submit the report until the agency issues a subpoena, to buy some time to mitigate their regulatory violations. May the attorney prepare the report and submit it to the agency at this time?

D.No, because when a lawyer knows or reasonably should know that the evaluation is likely to affect the client's interests materially and adversely, the lawyer shall not provide the evaluation unless the client gives informed consent.

QUESTION 4. Test results. A client was with three friends in a car when a police officer stopped the vehicle. During the stop, the police officer found cocaine and marijuana in the vehicle. The prosecutor charged the client for possession of a controlled substance. The prosecutor did not charge anyone for possessing marijuana, though it was illegal to possess such a substance in the jurisdiction where the vehicle was stopped. The attorney knows the client uses marijuana. The client took a drug test at the attorney's recommendation. The drug test shows the client negative for cocaine but positive for marijuana. The attorney wants to use the drug test to show it was unlikely that the cocaine found in the car belonged to the client. Nevertheless, providing the drug test to the prosecutor would reveal that the client tested positive for marijuana and might lead to charges based on the marijuana found in the vehicle at the time of the stop. The attorney asks his client if he can show the prosecutor the drug test as evidence that the client did not use cocaine, but does not mention the risk of additional charges. The client tells the attorney he can share the results with the prosecutor. Did the attorney act properly?

D.No, the client must give informed consent and the attorney did not make the client aware of the risks and reasonable alternatives.

QUESTION 10. The prospective client. An attorney has been practicing law for two years, and has represented some law school graduates in their appeals before the bar when the Board of Law Examiners had denied the applications for licenses on character or fitness grounds. A former law school classmate who was a first-year student when the attorney was a third-year student visits the attorney in his office. The former classmate was on law review and graduated near the top of the class, but now he expresses concern about the character portion of the bar application. "I need you to represent me before the Board of Law Examiners," the former classmate said. The attorney asks the classmate to explain the problem. The classmate then explains a history of heroin addiction in college, which led to a criminal conviction and a period of incarceration; but a successful rehabilitation program enabled the student to beat this addiction and live drug-free throughout law school. The classmate does not want to disclose this on the bar application. The attorney declines to represent the former student, and later receives a call from the bar examiners inquiring about this former classmate's character and fitness. The attorney then recounts everything the classmate said about the past addiction and criminal conviction. Was the attorney's conduct proper in this situation?

D.No, the former classmate here was a prospective client, and the attorney owed a duty of confidentiality, even though no representation occurred.

QUESTION 27. Moonlighting and retiring. Big Firm has a strict rule against moonlighting or its lawyers working cases on the side, even on a pro bono basis. The rule, which it embodied in its employment contract with all the attorneys who work there, is partly to avoid conflicts of interest with these clients who are unknown to Big Firm's managing partners. On the other hand, there is a pure profit motive as well—Big Firm wants the fees from every hour of legal work its lawyers perform for any client whatsoever. Some of the associates question whether this rule might be improper. In addition, Big Firm's employment policies provide that after one of its lawyers retires, at age sixty-five, the firm will pay the former employee five thousand dollars per month for life, so long as the attorney does not re-enter law practice anywhere. This stipend would be in addition to whatever income the retiring lawyer has from a pension or other retirement investments. Which of the following is true regarding Big Firm's internal policies?

D.The policies are proper, for it is a universally recognized exception to the rule against restrictions on lawyer's right to practice law that legal employers can require lawyers to devote their entire practice to the firm's clients.

QUESTION 19. One time thing. In anticipation of a hearing before a federal agency in Washington, D.C., an attorney travels to a Washington suburb in Virginia from her own state to meet with her client (from her home state), interview witnesses, and review relevant documents. The attorney makes weekly trips there over the course of a year and spends most of her workweek there each time (four or five days), as the agency hearing pertains to a complex antitrust matter. The attorney solicits no new clients there. She works only on the matter for the client from her home state but is nonetheless unlicensed in Virginia. Is the attorney's conduct proper?

D.Yes, because a lawyer rendering services in a foreign jurisdiction on a temporary basis does not violate the rules merely by engaging in conduct in anticipation of a proceeding or hearing in a jurisdiction in which the lawyer is authorized to practice law.

QUESTION 23. Licensed in a neighboring state. An attorney has a license to practice in the state in which his firm operates. He hires as an associate, a law school friend who does not have a license in the state, who but holds a license to practice in a neighboring state with similar laws and precedents. The attorney gives the associate attorney only simple cases that require mostly scrivener's work (paperwork) for the clients, but he allows the associate to interview clients and to prepare and file client forms and paperwork. About once a week, the attorney checks with the associate and asks how his work is going, and the associate always says everything is fine, and occasionally asks questions about local laws or rules. Any clients whose matters seem to require actual litigation go to the attorney, and the associate handles only non-litigation forms and filings for clients. Is the attorney subject to discipline for this arrangement?

D.Yes, because the attorney is assisting another person in the unlicensed practice of law in his jurisdiction.

QUESTION 24. Eliminating the plaintiff's lawyer. An attorney is a notorious personal injury lawyer, widely feared by defendant corporations and insurers who must defend claims. The attorney reaches one exceptionally favorable settlement for his client, a structured settlement paying several hundred million dollars over the period of five years. The defendant has lost cases to the attorney on several occasions and wants to avoid dealing with him in the future. The defendant demands, as a condition of settlement, that the attorney will not represent any other clients in the future in tort actions related to this defendant or even to similar businesses in that jurisdiction. The attorney's contingent fee will be large enough for him to retire comfortably to a private tropical island and never need to work again, so he is amenable to this condition of the settlement. Is the attorney subject to discipline for this agreement?

D.Yes, the agreement violates the rules, but the attorney may not care about being subject to discipline if he plans to leave the practice of law.

387. An attorney has expertise in launching new businesses. His undergraduate major was entrepreneurship, and he has numerous connections among investment bankers, and venture capitalists in the area. Entrepreneurs seek him out to incorporate their new businesses and help them find loans and equity investors for startup. The attorney drafts articles of incorporation and bylaws. He handles name registration with the Secretary of State, arranges meetings with local commercial bankers and investors, and helps write business plans and market analysis in anticipation of these meetings. Which of the following is true regarding the attorney's activities?

a) Both the legal services (incorporating) and the law-related related services (writing business plans and arranging investor meetings) would be subject to the requirements of the Rules of Professional Conduct.

275. Two clients, an entrepreneur and a venture capitalist, jointly consulted an attorney about establishing a business. The two clients had not yet agreed on the confidentiality of their separate communications with the attorney. The entrepreneur later sent the attorney a confidential memorandum outlining his own proposed business arrangement. The venture capitalist knew that the entrepreneur had sent the memorandum but did not know its contents. Eventually, the joint representation ended. Two years later, the venture capitalist filed suit against the entrepreneur to recover damages arising out of the failed business venture. Each hired a new lawyer for the litigation. The venture capitalist then requested a copy of the memorandum during discovery, and the entrepreneur responded that this was a privileged communication. The entrepreneur asserted that the venture capitalist never knew the contents of the letter during the joint representation, so he had waived his right to this item. How should the court rule on this discovery issue?

a) In this litigation, the memorandum from the entrepreneur to his previous attorney is not privileged and is therefore discoverable.

350. An attorney could not find a full-time job after law school, so instead he works on a contract basis for other firms. The attorney also signs up with a legal temp-work agency, a company owned by nonlawyers that places lawyers in temporary assignments at law firms that need an extra associate on a short-term basis. Law firms contact the legal temp-work agency when they need lawyers for a special project or assignment, and the agency sends them several resumes from which to choose the temporary associates they want. Through this temp-work agency, the attorney receives a three-month assignment at Big Firm conducting document review as part of litigation discovery. The firm pays the attorney $75 per hour, and it pays the temp-work agency a placement fee of 7% on whatever the attorney earns. Big Firm, in turn, passes the attorney's $100/hour fees and the 7% placement fee through to its clients as an item on the client's bill. Is this arrangement proper?

a) It is proper for Big Firm to pay the placement fee to the agency, to pass the fees through to the clients, and to pay the attorney's hourly rate out of the fees it receives from clients.

386. An attorney practices commercial real estate law in the state capitol, but also provides legislative lobbying services for some clients, especially for firms seeking lucrative government contracts. For example, working on a retainer, the attorney successfully lobbied his state legislature to privatize most of its prison system, and to give his client the contract to operate the private prisons. His client continues to pay the retainer and the attorney continues to lobby for longer statutory minimum sentences for crimes, so that the private prisons remain full. The attorney uses a separate retainer agreement for lobbying work, which specifies that he is not representing the client as their lawyer, but only as a lobbyist, and is not providing legal advice or legal services under their agreement. Meanwhile, one of the attorney's other clients faces charges of securities fraud and hires the attorney to handle his appeal, which includes arguing that the mandatory minimum sentences are unconstitutional. The criminal defendant signs a written waiver of the potential conflict of interest the attorney has over the mandatory sentencing issue, but the attorney fails to obtain a similar waiver from the private prison client on whose behalf he lobbied for the mandatory sentencing laws. If the attorney is successful in having mandatory sentencing laws declared unconstitutional on behalf of his criminal client, will he be subject to discipline for the conflict of interest with his lobbying client?

a) No, because lobbying is a law-related service that a nonlawyer could do, and is distinct from the lawyer's legal services, according to the retainer, so the conflict of interest rules do not apply.

493. An experienced litigator became a judge. In her previous litigation practice, she would regularly search online to learn more about the opposing party, opposing counsel, and even jurors. She sometimes found useful information on opposing parties' websites or on social media. Now serving as a judge, she visits the website of the corporate defendant in one of her cases, to learn more background about the company and its products and pricing. In this instance, the judge did not find any information on the company's website that seemed useful in understanding the issues in the pending case. The judge did not do any online research about the other party (the plaintiff) in the case. Was the judge's research proper?

a) No, because the Code of Judicial Conduct prohibits judges from conducting online research to gather information about a party or juror in a pending case, even if the research yields no useful information.

482. An experienced attorney regularly represented Conglomerate Corporation as its outside litigation counsel. One of Conglomerate's employees filed a lawsuit against the company as her employer. Conglomerate instructed its information technologies staff to copy the contents of her workplace computer for useful information in defending the lawsuit, and then Conglomerate's management provided copies to its outside counsel. Upon review, the attorney for Conglomerate saw that some of the employee's e-mails have the heading "Attorney-Client Confidential Communication." Under the Model Rules of Professional Conduct, does the attorney for Conglomerate have an ethical duty to notify the employee's lawyer that the employer has accessed this correspondence?

a) Notwithstanding a local court order to the contrary, the Model Rules do not independently impose an ethical duty to notify opposing counsel of the receipt of private, potentially privileged e-mail communications between the opposing party and his or her counsel.

454. A certain client applied to for a loan from a Big Bank based on a security interest in farm machinery that the client claims to own. Big Bank required that all borrowers provide an opinion letter at the time of closing from the borrower's lawyer. The legal opinion letter was to verify, based on a check of courthouse records, that the borrower's title to the machinery carried no encumbrances, such as recorded liens. The client asked his attorney to provide the required opinion, and the attorney produced the letter addressed to Big Bank. Unfortunately, the attorney had in fact made no effort to verify the facts stated by checking courthouse records, instead relying entirely on his client's statements concerning the state of title of the property. If the attorney had conducted the investigation described in the opinion letter, he would have seen that the public records indicated several liens superior to Big Bank's security interest. Which of the following is correct, given these facts?

a) The attorney failed to conduct the investigation described in the opinion letter and therefore violated his duty of care.

489. A natural disaster struck a certain attorney's city and destroyed his office, including his records of his client trust accounts. What must the attorney do in response, to satisfy his ethical obligations to keep records of such accounts? a) The attorney must attempt to reconstruct the records from available sources. b) The attorney must self-report the loss to the state disciplinary authority and accept whatever sanction it imposes. c) The attorney must notify current and former clients about the loss of the records, even if the attorney can fully reconstruct the records from available sources. d) The attorney must compensate the clients by returning a portion of the fees that the clients have paid.

a) The attorney must attempt to reconstruct the records from available sources.

272. A wealthy client invited his attorney to visit the client's lavish home, so that they could update the client's will and other estate planning instruments. They updated these documents every year. On this occasion, a few others were present during their conversation about the client's estate planning issues: the client's longtime business partner, the client's new girlfriend, a housekeeper, one of the client's grown children, and the client's personal physician, who had stopped by for a social visit. Two individuals would be necessary to witness the execution of an updated will, so the attorney was glad to have others present. During the conversation, as an aside, the business partner mentioned some upcoming litigation that was in the news, a lawsuit between a major insurance carrier and a pharmaceutical company related to the current epidemic in opioid abuse. No one present was a party to the anticipated litigation, but many investors were following it with great interest. Afterward, the attorney wrote personal notes about the meeting, including who was present and what each person had said. A year later, the client died, and litigation ensued over the client's estate. Which of the following would apply to the attorney's notes and mental recollections about the conversation with the client and others that were present?

a) The attorney's ethical duty of confidentiality to the client.

329. An attorney had a series of private meetings with a client about incorporating the client's new business venture as an LLC. The attorney kept careful notes of these discussions. Which of the following is true regarding these notes about the conversations between the attorney and the client? a) The attorney's notes would come under the protection of the attorney's duty of confidentiality but not the work product doctrine. b) The attorney's notes would come under the protection of attorney-client privilege and the work product doctrine. c) The attorney's notes would come under the protection of the attorney's duty of confidentiality but not attorney-client privilege. d) The attorney's notes would not come under the protection of the work product doctrine, nor attorney-client privilege.

a) The attorney's notes would come under the protection of the attorney's duty of confidentiality but not the work product doctrine.

497. A President appointed an experienced circuit court judge to fill a vacancy on the U.S. Supreme Court. During the Senate confirmation hearings, a committee member asked the candidate if he would overturn Roe v. Wade if he had an opportunity to do so, or if he would uphold Roe v. Wade due to stare decisis. How should the candidate respond?

a) The candidate should refuse to commit beforehand to ruling a specific way on any given case.

307. An attorney sometimes recorded his interviews with clients, after obtaining permission from the client, especially when the client was recounting a long narrative about events that transpired, which had given rise to litigation. The opposing party in one lawsuit sought discovery of the recording of the client's narrative of the events to the attorney. Which of the following is most likely to result in the recording being discoverable?

a) The client played the recording at home for his friend to get his advice and input.

279. The police arrested Professor Stevenson and would not permit him to communicate directly with his attorney. Professor Stevenson asked his longtime friend and confidant, Sisyphus, to convey to his attorney that the attorney should not permit the police to search Professor Stevenson's home. Later, the prosecution calls the friend to testify about the contents of the message he related from Stevenson to his attorney. The attorney claims this information is privileged. How should the court rule?

a) The contents of the message transmitted through the friend are privileged and therefore both undiscoverable and inadmissible at trial, because the friend was acting as an agent of the client.

328. An attorney agreed to represent a plaintiff who sustained serious injuries three months earlier when she fell through a defective staircase on the defendant's premises. Her hospitalization after the incident prevented the plaintiff from securing legal representation for twelve weeks. The attorney filed a personal injury lawsuit immediately, and the defendant retained counsel for the litigation in response. The defendant's lawyer, however, had visited the accident scene immediately after the accident and took photographs. Two weeks later, the defendant completely rebuilt the staircase, adding additional handrails, bannisters, and other safeguards. Later, as the litigation proceeded through the discovery phase, the plaintiff's attorney sought production of defense counsel's photographs of the scene, and defense counsel objected that the photographs were non-discoverable attorney work product. The attorney for the plaintiff explained in a motion to the court that the lapse of time since the accident prevented the attorney from viewing the accident scene as it was at the time, invoking the need-and-hardship doctrine. Moreover, the plaintiff's delay in securing counsel was due to her injuries and hospitalization, which were not her fault. How should the court rule?

a) The court should compel production of the photographs because there is no other way for the plaintiff to establish the condition of the staircase at the time of the accident.

326. An attorney agreed to represent a plaintiff who sustained serious injuries three months earlier when she fell through a defective staircase on the defendant's premises. Her hospitalization after the incident prevented the plaintiff from securing legal representation for twelve weeks. The attorney filed a personal injury lawsuit immediately, and the defendant retained counsel for the litigation in response. The defendant's lawyer visited the accident scene immediately and took photographs. By that time, the defendant had completely rebuilt the staircase, adding additional handrails, bannisters, and other safeguards. Later, as the litigation proceeded through the discovery phase, the plaintiff's attorney sought production of defense counsel's photographs of the scene, and defense counsel objected that the photographs were non-discoverable attorney work product. The attorney for the plaintiff explained in a motion to the court that the lapse of time since the accident prevented the attorney from viewing the accident scene as it was at the time, invoking the need-and-hardship doctrine. Moreover, the plaintiff's delay in securing counsel was due to her injuries and hospitalization, which were not her fault. How should the court rule?

a) The court should deny the motion because the photos depict a completely different staircase than the one that caused the accident, so they are no more helpful than photos the plaintiff could take now.

401. Which of the following is true regarding Model Rule 8.2? a) Unlike defamation cases, which use a subjective test for intent, disciplinary cases for violations of Rule 8.2 use an objective test to assess the lawyer's mental state as to whether the lawyer knew the statement was false or recklessly disregarded its falsity. b) Reckless disregard as to falsity therefore means essentially the same thing in discipline as it does in public-official libel and slander cases. c) A lawyer's subjective belief that the statements are true could be a defense in the context of disciplinary proceedings for violations of Rule 8.2. d) "Reckless disregard as to falsity or truthfulness" does not mean the lawyer has a duty to verify suspicions before making allegations against a judge.

a) Unlike defamation cases, which use a subjective test for intent, disciplinary cases for violations of Rule 8.2 use an objective test to assess the lawyer's mental state as to whether the lawyer knew the statement was false or recklessly disregarded its falsity.

432. A certain attorney made an informal agreement with Physician that they would refer clients to each other when the situation seemed appropriate. They did not pay each other any money for referrals, but the relationship was explicitly reciprocal - the attorney referred patients who needed medical examinations to Physician, and when Physician had patients needing legal representation, he referred them to the attorney. The relationship was not explicitly exclusive - each was free to refer clients to others - but it happened that neither had similar reciprocal relationships with anyone else. They always inform their clients when making such referrals that they have a reciprocal relationship. Is such an arrangement proper?

a) Yes, a lawyer may agree to refer clients to another lawyer or a nonlawyer professional, in return for the undertaking of that person to refer clients or customers to the lawyer, assuming clients are aware, and the relationship is not exclusive.

490. An attorney worked for Big Firm for three years, and thereafter he took several of Big Firm's clients with him to start his own firm. Big Firm had an official unwritten policy that its lawyers should not take Big Firm clients with them when the lawyers left the firm, but the attorney in this case simply ignored this policy. Big Firm threatened litigation over the attorney's actions, but it did not follow through on the threats. The attorney then advertised on local billboards that he was a former Big Firm lawyer who would provide affordable legal services to working-class clients. A certain new client hired an attorney to represent him in a divorce proceeding and gave the attorney several thousand dollars to cover all legal fees and expenses in the case. The attorney deposited the money in his client trust account, and he explained to the client that he would withdraw money periodically as he earned fees or incurred expenses. Was this arrangement proper?

a) Yes, a lawyer may deposit funds from the client into a trust account and withdraw funds only as billable fees or expenses accrue.

461. An attorney had struggled all through law school with the volume of reading and memorization, and then she had struggled to establish a successful law practice because everything took so much time and acumen. She complained constantly about her workload and maintained a blog where she espoused strong views about foreign policy, national security, wine, and exercise. Conglomerate Corporation hired the attorney to represent it in a lawsuit in which it was the defendant. During early settlement negotiations, the attorney told the plaintiff that the Board of Directors for her client, Conglomerate, had formally disapproved any settlement for more than fifty thousand dollars, even though Conglomerate's Board of Directors had in fact authorized a much higher settlement amount. Was it improper for the attorney to make such untruthful statements during settlement negotiations?

a) Yes, a lawyer must take care not to convey communications regarding the client's position, which otherwise would not count as statements of fact, in language that converts them, even inadvertently, into false factual representations.

394. A criminal defense attorney was angry at the local prosecutor for pushing forward with a certain matter against one of the attorney's clients. In a state of frustration, the attorney penned a letter to state officials responsible for overseeing the local prosecutors, in which he accused the prosecutor in his case of specific instances of witness tampering, destruction of evidence, and framing innocent victims for crimes they did not commit. The attorney based these allegations solely on inferences that she had drawn from the unfavorable situation with her own case, and some rumors circulating among inmates in the county jail. Could the attorney be subject to discipline for sending this letter?

a) Yes, a lawyer shall not make a statement that the lawyer knows to be false, or with reckless disregard as to its truth or falsity, concerning the integrity of a public legal officer.

351. After a long, distinguished career as a solo practitioner in a major city, an elderly attorney agrees to join a newer law firm on the condition that the firm would pay $1000 per month after the attorney's death to his sister, who is 74 years old, until her death. The attorney's sister is not a lawyer. The firm agrees to this arrangement, in addition to making the attorney a partner with a 15% share in the firm. Is this arrangement proper?

a) Yes, because it is the payment of money over a reasonable period after the lawyer's death to a specified person.

447. An attorney solicits campaign contributions on behalf of an elected judge who is running for reelection. The judge wins reelection and shows his gratitude to the attorney by frequently appointing him to represent indigent defendants at the state's expense. The attorney engaged in the solicitation of contributions for the judge's reelection campaign because he hoped to receive such appointments. The fees from the appointments are disappointing, though, and the attorney later realizes that the fees earned from these appointments were not equal to the time the attorney spent soliciting the contributions. Could the attorney be subject to discipline for accepting these appointments?

a) Yes, because a lawyer shall not accept a government legal engagement or an appointment by a judge if the lawyer makes a political contribution or solicits political contributions for the purpose of obtaining that type of legal engagement or appointment.

398. An attorney was upset when he lost a high-stakes bench trial. When friends and acquaintances asked him about it in the following weeks, he would bitterly complain that the judge must have received a bribe from the opposing party, because there was no way that a reasonable judge could have ruled against the attorney's own client, given the evidence in the case. The attorney has no reason to think that the judge accepted a bribe except that he was shocked when he lost the case. Could the attorney be subject to discipline for making such comments?

a) Yes, because a lawyer shall not make a statement that the lawyer knows to be false or with reckless disregard as to its truth or falsity concerning the qualifications or integrity of a judge.

477. A state bar pro bono program arranges for lawyers to volunteer at police stations and county lockups to give limited-scope representation to arrestees who plan to proceed pro se, advising them mostly on their pre-trial rights (the right to remain silent, to have court-appointed counsel at trial, and so on). Sometimes these volunteer attorneys accompany arrestees to their arraignments or bond hearings, but then their assistance ends, and the defendant proceeds pro se (without counsel, either by choice or because they are ineligible for court-appointed counsel). The lawyer volunteer program has been going on for a few years, and it has expanded significantly. A local prosecutor receives a case assignment involving a pro se defendant. At the outset of the plea negotiation session, must the prosecutor ask the defendant if he has already received legal advice on a limited basis from a volunteer lawyer, and if so, refrain from further discussions until he has conferred with the other lawyer?

a) Yes, because any prosecutor in that situation has reason to believe that the unrepresented defendant received limited-scope legal services on some portions of the case.

413. An attorney represented a father at a child support modification hearing before a judge. During the hearing, the attorney made repeated disparaging references to the facts that the mother was indigent and was receiving legal services at no charge. Could the attorney be subject to a public reprimand for these comments?

a) Yes, because discrimination against persons based on their source of income or acceptance of free or low-cost legal services would be examples of discrimination based on socioeconomic status.

421. Big Firm handles employee litigation, including workplace harassment suits. Nevertheless, the managing partners at Big Firm have decided they will not take on clients with claims based on same-sex harassment, because they believe the law is still developing and juries return unpredictable verdicts in such cases. Has the firm violated the MRPC?

d) No, the rules that prohibit harassment and discrimination by lawyers do not limit the ability of a lawyer to accept, decline or withdraw from a representation.

491. An attorney represented a female client, the wife in a marriage-dissolution action. The husband had retained a lawyer at Boutique Firm to represent him. Meeting without either lawyer present, the wife and husband negotiated the outlines of an agreement providing for property division and child support. The wife then brought the husband to the attorney's office to have the agreement reduced to writing. The attorney welcomed both the wife and the husband and engaged in a discussion of provisions of the agreement with both the husband and wife. Has the attorney violated the Model Rules?

a) Yes, because he met with a represented opposing party without opposing counsel being present.

345. An attorney hired Receptionist because of her good looks and because her brother was in the attorney's college fraternity, but he did not check into her background at all or ask for references. Receptionist had access to all files, records, and accounts in the firm, and three months later, there arose a problem with funds missing from client trust accounts. Circumstantial evidence pointed to Receptionist as the culprit, and at this point the attorney learns that Receptionist has an arrest record for theft and embezzlement on several occasions in the past. The attorney lectures Receptionist about it but allows her to keep her job because nobody can prove her guilty - the firm does not keep the type of records that would enable anyone to prove where the missing funds went. When additional complaints arise over misappropriated client trust funds, would the attorney be subject to discipline?

a) Yes, because he was negligent in the hiring and supervision of nonlawyer employees.

474. An attorney was renting law-office space from an individual property owner. At one point, the attorney was late with his rent, and he soon received a letter from another lawyer, who was representing the property owner. The letter directed the attorney to vacate by a certain date. The attorney visited his property owner in person, without the prior consent of the property owner's lawyer. During the visit, the attorney insisted to the property owner that the lease prohibits the eviction without a one-month grace period. This argument was entirely plausible. Was it proper for the attorney to do this?

a) Yes, because he was visiting the property owner as a tenant, not as the representative of another party.

391. An attorney faced a disciplinary action over accusations that she had neglected a client matter and had not communicated enough with the client. The state disciplinary authority requested a written account of her version of what happened, and it asked her ten or twelve probing questions during the hearing. At the conclusion of the hearing, the disciplinary tribunal decided that the client complaint was without merit and cleared the attorney of all charges in that regard. At the same time, it also concluded that the attorney had answered one question during the hearing untruthfully, and that she had made a minor misrepresentation regarding dates in her written statement to the board. The tribunal therefore filed a separate grievance against the attorney for these misrepresentations. Could the attorney be subject to discipline for incidental misrepresentations to the grievance committee if the same committee had decided that the underlying case had no merit and issued a dismissal?

a) Yes, because it is a separate professional offense for a lawyer to knowingly make a misrepresentation or omission in connection with a disciplinary investigation of the lawyer's own conduct.

420. While cross-examining a Hispanic witness during a trial, a defense attorney grew frustrated at the witnesses' evasive answers, and finally asked the witness if "his people" or others "in his community" regularly lie under oath on the witness stand. The prosecutor immediately objected, and the judge sustained the objection, so the attorney withdrew the question. The witness then stated that he did not feel offended by the question because he understood that the lawyer was simply ignorant and relying on stereotypes. Three of the jurors were also Hispanic. Could the attorney be subject to discipline for this question?

a) Yes, because it is professional misconduct for a lawyer in the course of representing a client to say things that manifest bias or prejudice based upon race or national origin.

380. An attorney in a small partnership decided it was time to retire. The partnership agreement had clear provisions for the retirement of partners, in which the partnership would buy out the retirement partner's share, including an hourly prorated amount for work on matters that were still pending and had not yet generated divisible fees. The retirement provisions also provided a substantial pension for the retiring partner, purchase of a single-term life insurance policy, and separate payments from an annuity. A condition of these retirement benefits was that the partner permanently leave the practice of law. Is this condition proper?

a) Yes, because restrictions on the right to practice law are permissible as a condition of retirement benefits.

269. An attorney works for a state-operated legal aid clinic, which under a state statute counts as a social service agency. The state has a mandatory reporting law for child abuse, which statutorily requires employees of social service agencies to report any instances of child abuse they discover among their clients or constituents. The attorney met with a prospective client and her child to discuss potential representation at a welfare termination hearing. The prospective client did not meet the agency's guidelines to be eligible for free legal representation, however, so the attorney had to decline the case. Nevertheless, it was evident during the interview that the prospective client's child was the subject of serious physical abuse - a black eye, cigarette burns on her arms and neck, bruises on the backs of her legs, and a demeanor of cowering in fear around adults. The attorney wanted to talk to the mother about it, but the attorney has been unable to reach her since declining to represent her. Must the attorney report the prospective client for child abuse?

a) Yes, because state law requires the disclosure, and a lawyer may reveal information relating to the representation of a client to the extent the lawyer has a reasonable belief that it is necessary to comply with other law.

370. Conglomerate Corporation routinely hires outside counsel for representation on legal matters, and it requires the lawyers to sign an Outside Counsel Agreement (OCG) that contains the following provision: ATTORNEY agrees not to represent any party adverse to CONGLOMERATE CORP., or any entity in the APPENDIX, without prior written consent. In no event may ATTORNEY represent an adverse party against CONGLOMERATE in litigation. The APPENDIX contains a confidential list of entities ATTORNEY must use in screening for conflicts. The APPENDIX includes some entities that may be affiliated with CONGLOMERATE's parent companies, as well as entities that may not be controlled by CONGLOMERATE or its parent companies, but in which they may have an ownership interest. Would it be improper for an attorney to enter into this agreement, if it includes this OCG provision?

a) Yes, because the OCG provision creates an impermissible restraint on the attorney's right to practice law.

486. An attorney has a busy transactional practice and frequently must handle client funds, either for making commercial purchases, sales, leases, dispute settlements, or other transfers. The attorney faithfully deposits client money in a separate trust account and does not commingle the funds with his own, except that he deposits enough of his own money in the account to cover the monthly bank service charges. The attorney keeps complete, accurate records of all deposits and withdrawals for a full year, after which he destroys the records to preserve client confidentiality. Is the attorney acting improperly?

a) Yes, because the attorney did not keep records for a long enough period.

264. Client, a large auto dealer, retains an attorney to represent him in a bankruptcy case. This attorney's firm represents a bank, through which the client has several large loans that covered loans for the dealership. The loans are all contained in the bankruptcy. The attorney is concerned about whether there is a conflict, so he contacts a lawyer friend of his. While explaining his dilemma, the attorney tells Friend the name of the dealer. Is the attorney subject to discipline?

a) Yes, because the attorney disclosed more than what details were necessary to accomplish his purpose.

429. An attorney recently earned her Juris Doctor degree from a prestigious law school and easily passed the state bar exam, gaining admission to the bar in her home state. She worked for three years for a legal aid clinic that provided free legal services for indigent clients. At the end of her third year at the clinic, the attorney decided to start her own firm, representing primarily low-income clients who were ineligible for free services at the legal aid clinic, but who also rarely could afford the fees of most attorneys. As soon as she ended her employment at the legal aid clinic, she sent a certified letter to most of the lawyers in her geographic area describing her experience and explaining that she was starting her own firm and intended to specialize in low-dollar consumer protection cases, simple divorces, adoptions, name changes, and landlord-tenant disputes. The letter concluded by offering to handle such cases for other lawyers if the other lawyers did not want to invest their time on such low-dollar matters. Were the attorney's actions proper?

a) Yes, because the attorney's statements were not false or misleading and the letter was an appropriate announcement of the opening of her new firm and her intent to specialize in certain areas of law.

296. Conglomerate Corporation has several overseas facilities, and a mid-level manager at one of these locations bribed local government officials to obtain lucrative government contracts. The matter came to the attention of Conglomerate's top management and Board, who recognized that the incident was an egregious violation of the Foreign Corrupt Practices Act. An internal corporate investigation of the matter ensued, and the corporation's directors asked their in-house General Counsel to send written inquiries to the wrongdoer's counterparts in each of its overseas branches, asking whether similar payments or bribes were occurring elsewhere. After reviewing the responses to these inquiries and following up with phone calls and meetings, the corporate directors self-reported any questionable transactions to the relevant federal agencies. When one of those agencies brought an enforcement action against Conglomerate Corporation, the Department of Justice lawyers sought discovery of all the original written responses to these internal inquiries. The corporate directors and General Counsel refused, claiming that the information was privileged. Should Conglomerate Corporation be able to resist production of these documents as privileged?

a) Yes, because the corporate directors requested the information from the in-house attorney, and the information was work-related, and was necessary for obtaining legal advice

294. An attorney heard from one of his clients in county jail that the client's cellmate did not have a lawyer, so the attorney sent a message offering to represent him, and the cellmate agreed and hired the attorney. The new client was under investigation for a variety of financial crimes, so the attorney hired a private financial forensics investigator to assess the client's potential criminal liability. This investigation, conducted at the behest of the attorney, involved the investigator interviewing the client alone for over an hour about certain bank transfers and backdated checks. Later, the prosecutor subpoenaed the private financial forensics investigator to testify at the criminal proceedings against the client, and the investigator refused to answer any questions about the conversation with the defendant. Would attorney-client privilege apply to the investigator's private conversation with the attorney's client?

a) Yes, because the investigator was acting as an agent of the attorney, and the conversation was a confidential communication with a client for the purpose of obtaining legal services.

311. Conglomerate Corporation's recent litigation has received unfavorable media attention, so the corporate directors have hired a public relations firm (Afflatus, Inc.) to handle media relations and help boost the company's image. The directors have also asked their attorney, who is handling their litigation, to meet with the Afflatus staff, explaining the company's litigation position and how to answer media inquiries without giving statements that might bind the corporation to a disadvantageous legal position. The attorney opened his presentation with a declaration that the meeting was confidential, and that some of the information shared would be privileged. A few months later, the opposing party learns that this meeting occurred and seeks discovery of the PowerPoint slides the attorney used in the presentation to the public relations firm. Given these facts, would these the slides be discoverable at trial?

a) Yes, because the public relations firm is not the client.

446. Attorney Stevenson's sister is a dentist. Attorney Stevenson telephones his sister and explains that his firm is not doing well, that he needs more cases, and asks his sister to use him as her lawyer for any malpractice actions she faces or any collection actions against patients who do not pay their bills. Attorney Stevenson's sister finds this request annoying and makes no promises, but she agrees to keep it in mind. Was it proper for Attorney Stevenson attorney to make such a telephone solicitation?

a) Yes, because the recipient of the solicitation has a family relationship with the lawyer.

365. An attorney wants to retire from practice due to a chronic illness, and he decides to sell his practice to another lawyer. The sale agreement complies with the Model Rules regarding the sale of a law practice. As part of the sale agreement, however, the attorney stipulates that he will not resume the practice of law in that jurisdiction, even if medical breakthroughs cure his chronic illness and restore him to perfect health. The purchaser of the firm is aware that research for a cure of the attorney's illness is well underway, and he is concerned because it is foreseeable that the attorney would recover and want to return to the practice of law in a few years. Is it proper for the attorney and his buyer to include this provision of the sales agreement for the law firm?

a) Yes, because the rule against restrictions on the right to practice does not apply to the sale of a law practice.

443. After a bizarre accident that received heavy media coverage, the victims took the unusual step of sending written notices to every plaintiff's firm in the area stating that the victims did not want to hear from any lawyers about the matter. The attorney received the notice and promptly forgot about it, because he had not yet seen any of the media coverage about the accident. Two weeks later, the attorney decided to catch up on the latest news, and he read an article online about the bizarre incident. He sent a letter to the victims expressing condolences for their suffering and offering to provide legal services if they decided to file a claim over the incident. The victims read the letter, changed their minds, and agreed to have the attorney represent them. A lawyer at another plaintiff's firm, who had also received the notice from the victims, learned that the attorney was representing the victims. He made some inquiries and discovered how the attorney had found his new clients. The lawyer filed a grievance against the attorney with the state disciplinary authorities. Should the attorney be subject to discipline for the way in which he offered to represent the victims?

a) Yes, because the target of the solicitation has made known to the lawyer a desire not to receive such solicitations.

360. An attorney hired a second-year law student as a clerk. The law student is unlicensed. The attorney has the law student perform a variety of tasks. Which of the following tasks, if performed by the law student, would mean that the attorney is subject to discipline?

d) Reaching settlement agreements with insurance companies before the attorney indeed files any lawsuit in the matter.

310. Walter White conferred confidentially with his attorney, Saul Goodman, about how to resolve a specific legal problem. Attorney Goodman suggested shredding documents and hiring some thugs to beat up the other party in the matter, leaving them with a warning to stay away from Walter White. White, the client, proceeds with this plan. Later, when White faces criminal prosecution for the assault-for-hire, the prosecutor seeks disclosure of any conversations he had with his attorney about hiring thugs to carry out the assault. Predictably, Attorney Goodman argues that the conversation comes under the protection of attorney-client privilege. Is the prosecutor correct to demand disclosure?

a) Yes, given these facts, the crime-fraud exception to attorney-client privilege would potentially apply.

381. Conglomerate Corporation has a rule for in its legal department against "side hustles," that is, its lawyers working cases for private clients on the side, even on a pro bono basis. The rule, which it embodied in its employment contract with all the in-house attorneys who work there, became a policy there when General Counsel was targeting a certain employee in the legal department, for purely personal reasons, and needed to create an excuse to fire the lawyer. Is this rule proper?

a) Yes, it is a universally recognized exception to the rule against restrictions on lawyer's right to practice law that corporate legal departments can require that in-house counsel confine their entire practice of law to the organization's legal affairs.

277. A client kept in his files an old memorandum that the client had prepared for his attorney during an earlier representation by the attorney. After some time, the client takes the memorandum to another lawyer, in confidence, to obtain legal services on a different matter. The memorandum qualified as a privileged communication in the earlier matter. While in the hands of the new lawyer, does the memorandum remain under the protection of privilege?

a) Yes, privilege still applies to the document due to its originally privileged nature.

433. An attorney received a client referral by email from a friend who worked as a nurse in a nearby emergency room. The client called the attorney's office the same day that the attorney received her friend's email about this potential client. The representation of this new client yielded a favorable outcome, with a generous damages award for the client and substantial fees for the attorney. In appreciation for the referral, the attorney sent her friend a fancy fruit basket that cost around $150, with a card thanking the friend for the lucrative referral. Was it proper for the attorney to give such a gift to a nonlawyer for referring a client to her?

a) Yes, the Model Rules permit a lawyer to give nominal gifts, such as an item that might be a holiday gift item, in appreciation to a person for referring a prospective client.

301. An attorney specialized in criminal defense work, and at one point she agreed to represent a client who was multiple charges for gang-related criminal activities. While the client was in county lockup, inmates there from a rival gang assaulted him, necessitating his hospitalization. The attorney visited her client in the hospital to discuss a pending plea offer from the prosecutor. Both the client and the attorney believed, with good reason, that they were having a confidential conversation. Unbeknownst to them, however, a doctor was eavesdropping on their conversation, and the doctor subsequently contacted the prosecutor and repeated the entire conversation. Armed with this new evidence, the prosecutor revoked the pending plea offer, proceeded with the prosecution, and called the doctor to testify at trial about the conversation. The attorney argued that her conversation with her client came under attorney-client privilege and was therefore inadmissible at trial. Is the attorney correct in this assertion?

a) Yes, the attorney and the defendant were reasonable in believing that the conversation was confidential.

361. An experienced attorney has an office in State X, and she is duly licensed to practice law in that state. The attorney's office is in a city on the border of State Y, and the attorney does not have a license to practice there. Over the years, some of the attorney's clients have in fact been residents of State Y, and their legal issues sometimes involve research into the laws or judicial precedents of State Y. For the convenience of these clients, and to attract business of other clients there, the attorney rents a small office space, hires nonlawyer clerical staff, and otherwise prepares premises for the general practice of law at a branch-office location in State Y. Apart from the issues raised by opening the new branch office, was it proper for the attorney to represent residents of State Y in her office in State X?

a) Yes, the clients are coming to the attorney in her office in the state where she has a license to practice law.

315. An attorney's client was a member of a drug cartel that imported and distributed illegal narcotics. The client promised the other cartel members that the client would provide anyone in the cartel with legal representation whenever the need arose. The client then offered the attorney a generous monthly retainer if the attorney would stand ready to provide legal services whenever the client or the cartel associates encountered legal difficulties during the operation of the cartel. In a confidential communication that would normally otherwise qualify as privileged, the client told the attorney the identities of the other cartel members. The client continued the cartel operations for some time after this communication. Would government lawyers, in a subsequent law enforcement action, be able to compel the attorney to disclose the identities of the other cartel members?

a) Yes, the crime-fraud exception renders nonprivileged the communications between the client and the attorney, including identification of the client's confederates.

414. An attorney settled a legal malpractice claim by agreeing to make monthly payments to the former client for five years, which would add up to the full settlement amount. The attorney put forth his car as security for the obligation. After making a few of the monthly payments, the attorney left the jurisdiction with his car, leaving no forwarding address, so the former client (who was now a holder of the security interest in the car) could not locate the attorney or the car for more than one year. The state criminal code provides that it is a class 5 felony to conceal property in which there is a security interest. The attorney never faced criminal charges or arrest, but the state bar received a complaint about the matter and commenced disbarment proceedings against the attorney. Can the attorney face disbarment over a crime for which there were never any charges filed?

a) Yes, the fact that the respondent has not been criminally charged or convicted of this offense is not important for purposes of lawyer discipline.

460. The chief financial officer of Investors' Club, a private investment trust, is under suspicion for converting $100,000 of Investors' Club's assets for personal use. The other responsible corporate officers of Investors' Club, acting on the trust's behalf, retain an attorney to recover the money from the chief financial officer. At the same time, they direct the attorney not to reveal the loss, or file a lawsuit, until she has first exhausted other collection efforts. Although the matter is not yet in litigation, would it be improper for the attorney to proceed with dual representation, of both the organization and the chief financial officer in this matter, if both consent?

a) Yes, the interests of Investors' Club and the chief financial officer are so adverse that even informed consent of both would not permit their common representation by Lawyer in the matter.

283. An attorney agreed to represent a client who suffered from severe mental illness that had resulted in his institutionalization. The client complained that the staff mistreated her and wanted the attorney to litigate. At the end of this litigation, the court appointed a family member as the legal guardian for the client and her assets. Subsequently, a question arose concerning the client's ownership rights in certain intellectual property, and the attorney agreed to represent the interests of the client in the property. The client's legal guardian participates in the conversations between the attorney and the client, and he serves as an intermediary for confidential correspondence or messages between the client and the attorney. Would attorney-client privilege still apply to these communications, if the guardian is present or serves as an intermediary?

a) Yes, the legal guardian is necessary for rendering legal services to the client, and functions as the client's agent in this scenario.

416. An attorney owns his own law practice, and he represents clients if he believes in their cause. He regularly defends racists and hate groups against criminal charges and lawsuits, because he shares their philosophy and identifies with their racist views. Is it permissible for the attorney to advocate on behalf of racists and hate groups in litigation, if he supports their cause on a personal level?

a) Yes, the rules prohibiting discrimination and harassment by lawyers not preclude legitimate advice or advocacy the lawyer provides to clients who are openly engaging in such conduct.

424. An attorney practices law in two adjacent states, as he has a license to practice in each. He lives near the border and can easily serve clients in each jurisdiction. The two states have different rules about attorney disclosures of confidential client information - one state requires disclosures of client confidences whenever necessary to save a third party from death or serious bodily injury, while the other state forbids disclosures even under these circumstances. The attorney did indeed disclose confidential client information to save someone's life (the client was planning a murder and the attorney notified the authorities and warned the potential victim), but this occurred in the state that forbids such disclosures under these circumstances. The client files a grievance against the attorney in both states, and both state bars commence disciplinary proceedings over the same incident. The state bar of the other state, which would have required disclosure in this situation under its own rules, nevertheless reprimands the attorney for making the disclosure in violation of the rules in the state where the incident occurred. The attorney objects that the state cannot impose a sanction on him for conduct that the state's rules would have required. Is the state bar correct?

a) Yes, the state bar should apply the rules of the jurisdiction in which the lawyer's conduct occurred.

322. The law school casebook industry was heavily consolidated. Several witnesses testified before a grand jury investigating this specialized publishing industry. Shortly afterward, an attorney for East Publishing Company debriefed the witnesses and wrote memoranda of those interviews in anticipation of the potential indictment of East Publishing, and the anticipated civil suits that could follow. Five years later, some plaintiffs representing a class of law school casebook consumers filed an antitrust class action against East Publishing and sought discovery of the non-opinion work-product portions of the attorney's debriefing memoranda. The plaintiffs were careful in preparing their case and gathering evidence through other means, and they can show that the witnesses in question were no longer able to remember some of the events to which they testified at the previous grand jury proceeding. Should a court order the attorney to produce the memorandum?

a) Yes, this situation falls under the need-and-hardship exception to the work product doctrine.

274. An attorney was a partner at Big Firm, which represented Conglomerate Corporation in their corporate merger negotiations with Giant Company. Big Firm had state-of-the-art network firewalls, virus protection, password protection, and other data security features in place. Nevertheless, one Friday evening some hackers managed to breach Big Firm's networks and access client information and partner emails, for purposes of engaging in insider trading. The firm detected the breach within a few hours and notified state and federal law enforcement. The stock exchange had closed for the weekend, and law enforcement managed to apprehend the hackers over the weekend, before they had a chance to review the stolen information and share useful data or engage in illegal stock trades. The clients suffered no losses or adverse effects, but they could have. The partners at Big Firm maintain that they should not be subject to discipline for failure to protect their clients' confidential information, because they had all the latest data security measures in place, though technology is constantly changing. Are they correct?

a) Yes, unauthorized access to, or disclosure of, client information does not constitute a violation of the Model Rules if the lawyer has made reasonable efforts to prevent the access or disclosure.

479. A litigation attorney from Big Firm was representing Conglomerate Corporation as the defendant in a civil matter. The attorney learned that the opposing party had hired an expert witness to support their claims, so she decided to initiate an ex parte contact with the expert witness retained to testify for the opposing party, without first obtaining permission from the opposing counsel. In fact, opposing counsel had asked the expert not to discuss the case with the inquiring lawyer. The attorney introduced herself by name, but she did not mention that she was a lawyer or that she had any relationship with the case. Instead, she said she was "researching an issue" and that the expert was a well-known specialist on the topic, which was true. The expert was willing to answer her questions because he was unaware that she was opposing counsel in the case. The attorney pressed the expert for specific examples of the issue they were discussing, which led the expert to mention his research related to the current litigation. The expert revealed information to the attorney that was useful for her representation of her client. Could the attorney be subject to discipline for discussing the matter with the expert without discussing her relationship to the case?

a) Yes, when a lawyer contacts any witness, lay or expert, actual or potential, the lawyer must not knowingly leave the witness in ignorance of the lawyer's relationship to the case that gives occasion to the contact.

320. Which of the following is NOT one of the elements of the work product doctrine?

a) anticipation of litigation applies to almost any legal work performed for a client, because litigation could eventually arise over any contract, will, or property disposition

262. An attorney is representing a corporate client on a variety of litigation matters. The attorney receives a subpoena (compulsory process) for information and a document relating to one of her corporate clients. The attorney promptly produces the information and document required by the subpoena, and then informs the client. Could the attorney be subject to discipline for this action?

a) yes, because she did not consult first with the client before making the disclosure

348. An attorney was part of a partnership before he died. He left his nephew as his sole heir. The partnership agreement, as written, provides that the firm should pay the certain amounts to the nephew. Those amounts are $210,000, for the attorney's share of the firm's assets; a $500,000 death benefit, provided for all shareholders in the partnership; and $17,500 for fees that the attorney earned on recent cases, but had not yet received. Under the Model Rules, which of the following represents the most that the firm may properly pay to the decedent's nephew?

b) $727,500, for the attorney's share of the firm's assets, his of uncollected fees, and the death benefit

403. Which of the following statements, made publicly by an attorney, would violate Model Rule 8.2? a) A lawyer accused a judge of anti-Semitism, for which the lawyer had adequate factual support and documentation.5 b) A lawyer speculated to a reporter that a judge was "not being honest about the reasons why he committed [a defendant] to the Department of Corrections" c) A lawyer referred to a judge as "dishonorable" and a "brainless coward" d) A lawyer criticized a judge's ruling by saying it was "incoherent" and "wrongly decided.

b) A lawyer speculated to a reporter that a judge was "not being honest about the reasons why he committed [a defendant] to the Department of Corrections"

383. An attorney practices corporate securities law in a Wall Street firm. The attorney is also one of three owners of a financial forecasting consulting firm, Trends Tomorrow, which employs several well-known economists and financial analysts. The attorney refers clients to this firm when they need consultants to advise them about the timing of new stock offerings, projections for share price and profit forecasts, and so on. The attorney duly discloses to clients before referring them that she is a part owner of the consulting firm and that they are free to shop around and hire other consultants if they prefer; she also explains that the Trends Tomorrow is not a law firm and provides only financial forecasting services. Trends Tomorrow is in the building next door to the attorney's Wall Street firm, and when clients go there, Trends Tomorrow explains as part of their service contract that they provide no legal services. Eventually, complaints emerge that Trends Tomorrow has been leaking confidential client information to the press, and that the consulting firm has potential conflicts of interest, advising competing clients about strategies to encroach on one another's' market share. The attorney faces disciplinary charges for these violations, but the attorney claims that the complaining clients need to show that the disclosures provided were inadequate to apprise them of the fact that the Rules of Professional Conduct for lawyers would not apply to Trends Tomorrow. Who has the burden of proof on this issue?

b) Attorney has the burden of proof to show that the lawyer has taken reasonable measures under the circumstances to communicate the desired understanding.

362. An experienced attorney has an office in State X, and she is duly licensed to practice law in that state. The attorney's office is in a city on the border of State Y, and the attorney does not have a license to practice there. Over the years, some of the attorney's clients have in fact been residents of State Y, and their legal issues sometimes involve research into the laws or judicial precedents of State Y. For the convenience of these clients, and to attract business of other clients there, the attorney rents a small office space, hires nonlawyer clerical staff, and otherwise prepares premises for the general practice of law at a branch-office location in State Y. Is it permissible for the attorney to open the branch office in State Y?

b) It is impermissible because she does not have a license to practice in State Y and she has established an office or other systematic and continuous presence in this jurisdiction for the practice of law.

346. A certain attorney is a fifth-year associate at a large national law firm. As a senior associate, the attorney can attend business meetings of the firm, but cannot vote on any decisions. The attorney is aware that the firm has no measures in effect that would give reasonable assurance that the paralegals are observing the confidentiality and conflict of interest rules that are part of the professional obligations of lawyers. The attorney mistakenly believes, however, that the rules apply only to the lawyers in the firm, not to the clerical staff of paralegals. When a paralegal in a separate practice group from the attorney violates the rules and the state disciplinary authority investigates the firm's ethical compliance measures, will the attorney be subject to discipline?

b) No, because the attorney is not a partner nor in a comparable managerial position to implement such measures, nor does it appear that the paralegal was under the attorney's direct supervision

499. An attorney practices personal injury law in a small town. One of the judges who regularly presides over the attorney's cases is celebrating his twenty-fifth year on the bench, and the judge's friends and family have planned a banquet honoring the judge for reaching this milestone in his career. The organizers of the event invited many of the lawyers and judges in the area to the event. Many of the invitees are planning to bring a congratulations card or small congratulatory gift to the banquet. The personal injury attorney purchased a $250 silver-encased commemorative watch as a gift for the judge. The attorney presented it at the banquet, and the next day the judge made a public report of the gift. Was it improper for the attorney to give this watch to the judge?

b) No, because the judge publicly reported the gift the very next day.

278. A client confidentially delivered his own business records to his attorney, who specializes in tax matters, to obtain the attorney's legal advice about taxes. The business records were routine bookkeeping files, not prepared for obtaining legal advice. When the IRS eventually brought an enforcement action against the client and sought production of the business records that the client had provided to the attorney, the attorney asserted that attorney-client privileged protected them from disclosure. Is the attorney correct?

b) No, the records gain no privileged status by the fact that the client delivered them to the attorney to obtain legal advice.

458. An attorney worked for a corporation as in-house counsel. The attorney discovered that the Chief Financial Officer falsified the corporation's quarterly earnings report to prop up the firm's share price, as the CFO's compensation is partly in stock options. The attorney knows that these misrepresented earnings appeared in the filings to the Securities and Exchange Commission, and the misrepresentations will eventually result in severe regulatory fines or civil liability for the corporation. The attorney believes, with good reason, that the violation will result in substantial injury to the organization. The Chief Financial Officer hired the attorney, and he directly supervises the attorney in the organizational chain of command. The attorney confronted the Chief Financial Officer, but this proved unfruitful, and then the Chief Financial Officer discharged the attorney. What should the attorney do in this situation?

b) The attorney should proceed as the lawyer deems necessary to assure that the organization's highest authority knows about the circumstances of the lawyer's discharge.

271. An insurance company retained an attorney to defend both the insured employer and one of its employees, whose conduct is at issue and for which the employer might be vicariously liable. During a private consultation with the attorney, the employee recounts some facts about the incident that are self-incriminating. In fact, the confidential information that the employee shared with the attorney suggests that the employee was acting outside the scope of his employment at the time, and his actions were also outside the scope of the employer's insurance coverage. The employee had a reasonable belief that he had client-lawyer relationship with the attorney, and the employee did not understand the legal implications of his admissions. In subsequent interviews with other witnesses, the attorney corroborated this information. It appeared to the attorney that the insurance company could have a contractual right to deny coverage for the employee's conduct, and the employer could invoke scope-of-employment principles to defend against its own liability to the plaintiff. What would the ethical rules require the attorney to do under these circumstances?

b) The attorney cannot disclose the information to anyone, and must withdraw from representing the employer, the employee, and the insurer.

286. Professor Stevenson was walking alone through a high-crime neighborhood late at night, carrying his briefcase, which contained copies of confidential and privileged correspondence between the Professor Stevenson and his attorney. Robbers mugged Professor Stevenson and ran off with his briefcase, which they soon discarded when they discovered that it contained no cash or valuables. The police recovered the briefcase, but to identify its owner, they opened it and read the documents. Some of the documents were very incriminating, so the police turned them over to the district attorney. Professor Stevenson and his attorney claim attorney-client privilege protects the documents from admission as evidence in any criminal proceedings, but the prosecution claims privilege disappeared when the police had a good reason to inspect the contents of a lost briefcase. Which side is correct?

b) The attorney is correct that privilege would still apply, and the documents are inadmissible.

481. A highly experienced attorney represented Big Bank as the financer in a home sale. The buyer, that is, the borrower, did not have legal representation in the transaction. Under the terms of the transaction, the buyer was to pay the legal fees of the attorney. The buyer wrote an email to the attorney stating, "I have several questions about legal issues in the house purchase on which you are representing me." The buyer also had several phone conversations with the attorney in which the buyer made similar statements. What is the attorney's ethical duty in this situation, regarding the buyer?

b) The attorney must inform the buyer that the attorney represents only Big Bank, and that the buyer should not rely on the attorney to protect the buyer's interests in the transaction.

488. A natural disaster struck a certain attorney's city and destroyed his office, including many documents of that had no intrinsic value belonging to clients, that serve no useful purpose to the client or former client, or for which there are electronic copies. Which of the following would be one of the attorney's ethical duties as a result? a) The attorney must self-report the loss to the state disciplinary authority and accept whatever sanction it imposes. b) The attorney need not notify either current or former clients about lost documents that have no intrinsic value. c) The attorney must compensate the clients for the documents. d) The attorney must make reasonable efforts to reconstruct documents of intrinsic value for both current and former clients, or to obtain copies of the documents that come from an external source.

b) The attorney need not notify either current or former clients about lost documents that have no intrinsic value.

456. A recent law school graduate obtained her law license and spent several months searching for a job. Eventually, she went to work for a medium-sized corporation as in-house counsel. The company had only recently grown to the size that it could afford to keep legal counsel on staff, as opposed to hiring outside firms to handle legal matters when they arose. This meant the newly-licensed attorney was the first lawyer to work as in-house counsel at this corporation. After seven months, the attorney discovered that the Chief Financial Officer had falsified the corporation's quarterly earnings report to help boost the firm's share price. Both the attorney and the CFO received stock options every quarter as part of their compensation plan. Realizing that these misrepresented earnings appeared in the filings to the Securities and Exchange Commission, and the attorney feared that the corporation would eventually face severe regulatory fines or civil liability for false earning reports. What should the attorney do in this situation?

b) The attorney should start with the Chief Financial Officer, then take the matter to up the chain of command in the organization if necessary, eventually bringing the matter to the board of directors if nobody in management will address the problem.

496. A President appointed an experienced circuit court judge to fill a vacancy on the U.S. Supreme Court. During the Senate confirmation hearings, a committee member asked the candidate if he supported a textualist or originalist approach to interpreting statutes and the Constitution. How should the candidate respond?

b) The candidate may discuss his jurisprudence or views on statutory or Constitutional interpretations, but he should not indicate how he would rule on a specific upcoming case.

309. An attorney handled the estate planning for an elderly client, which included the creation of a spendthrift trust, with the client's grandchildren as the beneficiaries. The trust document stipulated that disbursements to the beneficiaries were discretionary until they reach the age of 25. The client has now died, and the attorney who drafted the trust document for the client serves as the trustee. The beneficiaries, ages 21-23, have sued, seeking larger and more frequent disbursements from the trust. During discover, the plaintiffs request production of all documents relating to the creation of the trust and the testator's intentions about disbursements - emails and memoranda between the deceased client and the attorney. The attorney, now the trustee, claims that these communications come under the protection of attorney-client privilege. How should the court rule?

b) The court should compel disclosure because attorney-client privilege normally does not apply in disputes between trustees and beneficiaries.

408. An attorney discovers that a partner at his own firm has violated the Rules of Professional Conduct by failing to disclose adverse binding precedent to a tribunal, and by depositing client funds into his own bank account instead of a client trust account. Does the attorney have a duty to report the partner from his own firm to the state bar disciplinary authority?

b) Yes, because a lawyer who knows of a violation of the Rules that raises serious questions about the other attorney's honesty must report it to the state disciplinary authority.

291. A small independent soda company had a delivery truck that collided with a school bus full of children on a field trip. The company's owner and the driver, who were co-defendants in the first lawsuit over the incident, met with their litigation attorney - the owner agreed to pay the fees for representing both. As they were discussing the accident, the attorney called in his own accident scene investigator to join the discussion, and the investigator took notes. As the litigation progressed, the driver eventually filed a crossclaim against the owner for indemnification if the driver has to pay damages to the plaintiff. At that point, the driver sought to depose the attorney's accident investigator to have him testify about the admissions the owner made in the previous conversation. The owner objected. How is the court likely to rule?

b) The deposition can go forward, and the investigator's disclosures will be admissible, because the driver and the owner are now adverse parties in the litigation.

422. A judge asks the two lawyers in a case to help him conduct some first-hand investigation of the facts. At the judge's request, the plaintiff's lawyer and the defendant's lawyer together drive the judge to the location where the accident occurred that became the subject of the litigation and allowed the judge to take measurements and photographs of the scene from different angles. They also accompanied the judge to interview several witnesses at their homes, off the record. Both lawyers felt awkward about this, but they were afraid to contradict or confront the judge, out of respect for the judicial office. Could the lawyers be subject to discipline for this conduct?

b) Yes, because it is professional misconduct for a lawyer to assist a judge or judicial officer in conduct that is a violation of applicable rules of judicial conduct or other law.

417. An attorney faced prosecution for failing to file tax returns over a five-year period. The attorney worked for a legal aid clinic and never charged clients any legal fees, as the clinic provided free representation to the indigent. The attorney received a modest salary from the legal aid clinic, the funds for which came from the state's IOLTA program and from a federal Legal Services Corporation (LSC) grant. Could the attorney face suspension of his license to practice law?

b) Yes, because it is professional misconduct for a lawyer to commit a criminal act that reflects adversely on the lawyer's honesty.

484. Local police obtained photographs from partygoers, and the photographs showed minors from the local high school consuming beer and engaging in sexual activity at a recent drinking party. The police forwarded the photographs to the local prosecutor, who decided not to pursue criminal charges in the matter for several legal and evidentiary reasons. Instead, in hopes of shocking the minors' parents into dealing with underage drinking, the prosecutor showed them photos of their children drinking and engaging in sexual activity at the party. All the parents saw all the photos, including those of other parents' children drinking, some unclothed and others partially clothed, at the party. Could the prosecutor be subject to discipline for his zealous advocacy against underage drinking?

b) Yes, a lawyer shall not use means that have no substantial purpose other than to embarrass or burden a third person.

419. After practicing for two years, an attorney enrolled in an LL.M. program at a local law school, taking night classes. During his second semester, the attorney faced academic discipline for plagiarism in a seminar paper; the school permitted him to graduate, but he received a failing grade in the class and had to make up the credits with another course. As the attorney already has a license to practice law in the jurisdiction, could he be subject to discipline if the state disciplinary authorities learned of the plagiarism?

b) Yes, because it is professional misconduct for an attorney to engage in conduct involving dishonesty, fraud, deceit, or misrepresentation.

393. An attorney faced disciplinary action over a client grievance. The disciplinary tribunal asked the attorney several probing questions about her handling of client funds. The attorney had, in fact, used some client funds to pay off a gambling debt, so she felt less worried about a temporary suspension of her law license than about potential criminal charges for embezzlement. The attorney, therefore, simply refuses to answer the questions, without offering any explanation. The disciplinary tribunal then determines that it lacks substantial evidence that the attorney mishandled client funds, but it commences disciplinary proceedings over the attorney's refusal to answer some of its questions. The attorney now claims she was merely exercising her Fifth Amendment right to refrain from self-incriminating statements. Could the attorney be subject to discipline for refusing to answer the questions in this scenario?

b) Yes, a person relying on such constitution protections in response to a question must do so openly and not use the right of nondisclosure as a justification afterward for failure to comply with the rules requiring disclosures to the disciplinary authorities.

473. An attorney lived in State A, but she had a license to practice in adjacent State B, where she worked for a law firm. Some of her clients also lived in State A, but they had sought legal representation in State B because that is where they worked or owned property. A business owner who lived and worked in State B hired the attorney to help enforce a non-compete agreement against a former employee at their technology firm. According to the client, a rumor started going around just this past week that the former employee had either started his own business nearby or was working for a nearby competitor, either of which, if true, could violate the non-compete agreement. The employee left the client's company on bad terms ten days ago. The former employee lived in State A. The client provided a copy of the non-compete agreement, which the former employee had signed many years before on his first day of work. The human resources director at the client's business told the attorney that she assumed the former employee would have forgotten about the agreement, or that he was unlikely to be aware that he was violating it. The attorney decided that the first step would be to call the former employee and ask whether he has found another job yet or has started his own business. The attorney assumed that the former employee would not have retained counsel yet to challenge the non-compete agreement, given the HR director's comments about him, and how recently the events unfolded. The former employee answered the phone, explained that he was starting his own rival company, and that the non-compete agreement was invalid under state law. When the attorney asked why he thought it would be invalid, the former employee answered that his own lawyer assured him that recent changes in state law made the previous agreement void. They were, in fact, ready to challenge the agreement in court. The attorney asked him to have his own lawyer contact him, so that they could discuss settlement options for the dispute, and then ended the call. Did the attorney act properly?

b) Yes, as the prohibition on communications with a represented person only applies in circumstances where the attorney knows that the person is in fact represented in the matter under discussion.

426. After graduation from law school, an attorney had taken and passed the bar exam in two neighboring states, and she then had a license to practice law in each state. Her primary office was in her home state where he lived, but she also represented a few clients every year in the neighboring state. Seven years into her career, the attorney committed serious professional misconduct in her home state, and she received a public reprimand from the state disciplinary authorities. The actions that led to her disciplinary reprimand occurred entirely in her home state. The lawyer's conduct would have violated the rules in either of the jurisdictions where she had a license to practice law, because it involved commingling client funds with her own money, and the states mostly had identical rules concerning this activity. Several months after she received her reprimand in her home state, the disciplinary authority in the neighboring state commenced disciplinary proceedings against her as well. In the end, the neighboring state bar suspended her from the practice of law for one year in that state, a much more severe sanction than the public reprimand she had received in her home state, where the misconduct in fact occurred. The attorney appealed this suspension, claiming that the neighboring state bar had no jurisdiction over conduct that occurred entirely outside of its own borders. She also contends that the second punishment raises double jeopardy concerns. Did the state bar in the neighboring state indeed have the legal authority to suspend her license there, if the alleged misconduct occurred entirely in the attorney's home state, and she had already received a punishment for it?

b) Yes, attorneys may be subject to the disciplinary authority of two jurisdictions for the same conduct and may receive different sanctions in each state.

399. In Texas, state trial judges are elected by popular vote. A well-known liberal-progressive judge is running for reelection. An attorney who is a staunch conservative is campaigning for the opposing candidate from the other party. At a campaign rally, the attorney declares that the liberal judge (seeking reelection) is completely unqualified and incompetent to serve in the judiciary, and that he is an activist judge who uses his court to push a certain political and social agenda. The judge graduated from a prestigious law school, was formerly a partner at a large law firm, and is active in the state bar. He does, however, give consistently lenient sentences to criminal defendants who are black or Hispanic, and has always ruled in favor of unions when adjudicating cases involving collective bargaining agreements. The judge learns of these remarks by the attorney and files a grievance. Could the attorney be subject to discipline?

b) Yes, because a lawyer shall not make a statement that the lawyer knows to be false or with reckless disregard as to its truth or falsity concerning the qualifications or integrity of a judge.

295. The parents of an autistic child submitted a complaint to a vaccine manufacturer, claiming that its early childhood inoculation for Mumps-Measles-Rubella had caused the child's autism. The vaccine producer referred the complaint to its legal department. Its in-house counsel investigated the complaint, and eventually concluded that the matter posed no legal issues for the company, because of a federal statute that shields vaccine manufacturers from tort liability, which in turn would preempt any lawsuits in state courts. The attorney wrote a legal memorandum to the company's management describing his research and conclusions. He included in the memorandum a section about the alleged facts, and another section presenting the legal analysis. If the parents of the child later file a lawsuit anyway, would the facts that the attorney's memorandum included be discoverable, and admissible at a subsequent trial?

b) Yes, because attorney-client privilege does not apply to underlying facts, even if those facts were under discussion in an otherwise privileged communication.

449. An attorney made substantial financial contributions to the reelection campaign of an elected judge. The judge won reelection and showed his gratitude to the attorney by frequently appointing him to represent indigent defendants at the state's expense. The attorney claims that he made the donations not because he hoped to receive such appointments, but because he honestly believed that the judge was the best candidate for the position, though he could not explain why. In addition, it turned out that taken together, the attorney gave more than every other lawyer or law firm in the judge's district. Could the attorney be subject to discipline for accepting these appointments?

b) Yes, because contributions that in the aggregate are substantial in relation to other contributions by lawyers or law firms, made for the benefit of an official in a position to influence award of a government legal engagement, and followed by an award of the legal engagement to the contributing or soliciting lawyer or the lawyer's firm would support an inference that the purpose of the contributions was to obtain the engagement

382. A criminal defendant received a death sentence after his murder conviction. The defendant's attorney, a court-appointed lawyer representing the defendant at state expense, had already been representing the defendant in an earlier manslaughter (noncapital) case, which he was handling on a pro bono basis. In this other manslaughter case, the attorney filed a motion alleging newly discovered evidence of innocence, with a view toward eliminating one of the aggravating factors that was also a justification for the death sentence in the capital case. The state then moved to disqualify the attorney from representing defendant in the capital case, arguing that state-appointed capital counsel could not represent a capital defendant in more than one proceeding at a time. A state statute prohibited state-appointed capital counsel from representing a capital defendant in a noncapital proceeding at state expense. Can the attorney avoid disqualification because he is handling the noncapital case pro bono?

b) Yes, because extending the statute to pro bono representation of the capital defendant in other cases would constitute an impermissible restriction on the right to practice law.

388. An attorney agreed to write a recommendation letter for admission to the bar on behalf of the law student who had worked for him part-time throughout law school. The student had consistently behaved appropriately during her employment, in compliance with the ethical rules for lawyers and law firms. On one occasion, the student intern had confided in the attorney that she had faced academic discipline for plagiarism on a law school seminar paper, and that she was very ashamed of herself about the incident and had accepted a failing grade in the class. She took an overload of courses the following semester to make up for the lost credits from the course she failed. The attorney did not mention this incident at all in his "character and fitness" recommendation to the state bar, because he felt it was out of character and did not represent the way the student normally behaved at the workplace. He also assumed the student would report it herself or that the bar would inquire about the failing grade on her law school transcript. The bar admissions board eventually learned about the incident only from the law school administration, which turned over the student's disciplinary records. Could the attorney who wrote the favorable recommendation be subject to discipline for filing to mention or address the incident?

b) Yes, because he did not disclose a fact necessary to correct a misapprehension known by the person to have arisen in the matter in connection with an admission to the state bar.

428. An attorney outsources complicated legal research to a firm that exclusively provides background legal research for lawyers. Her newest corporate client is a nationwide business with branches operating in all fifty states, so the corporate client needs information about its legal responsibilities regarding a certain issue in every state - a state-by-state survey. The attorney calls herself a sole practitioner. Could the attorney be subject to discipline for failing to inform the corporate client that she plans to outsource the 50-state survey to a research firm?

b) Yes, because lawyers must not misrepresent their partnership with others or other organizations.

457. An attorney served as general counsel for a municipal auditing and enforcement bureau, which monitored the internal affairs and expenditures of the municipal government. The attorney discovered that the head of the bureau engaged in selective enforcement and self-dealing, and he suspected that bribery had occurred in a few instances. The attorney's confrontation of the bureau head proved futile, so the attorney then needed to proceed up the chain of command. Can the attorney, now serving as general counsel for a government bureau, report wrongdoing to anyone higher within that municipality?

b) Yes, because if the action or failure to act involves the head of a bureau, either the department of which the bureau is a part, or the relevant branch of government may be the client for purposes of the Rules of Professional Conduct.

435. An attorney made and distributed bumper stickers advertising for his firm that simply provided a catchy phone number: 1-800-LAWYER-1. The phone number rolled over to the attorney's office phone. The bumper stickers included no other information. Could the attorney be subject to discipline for such an advertisement?

b) Yes, because it does not include the name and office address of at least one lawyer or law firm responsible for its content.

431. Attorney Stevenson's law firm is simply "The Law Offices of Attorney Stevenson, Esq." Attorney Stevenson specializes in courtroom litigation. His website address is www.mytrialattorney.com. He selected this domain name and registered it so that he could use it for his law firm's website. Is this website address/domain name proper for Attorney Stevenson's law firm?

b) Yes, because it is not misleading, and a lawyer or law firm may also use a distinctive website address or comparable professional designation.

463. A litigation attorney from Big Firm was representing Conglomerate Corporation as the defendant in a civil matter. The attorney learned that the opposing party had hired an expert witness to support their claims, so she decided to initiate an ex parte contact with the expert witness retained to testify for the opposing party, without first obtaining permission from the opposing counsel. The expert witness was hesitant at first to talk to the attorney, because opposing counsel had asked the expert not to discuss the case with the inquiring lawyer. Nevertheless, the attorney persisted and eventually persuaded the witness to tell him some of his ideas and conclusions so far about the case. Was it proper for the attorney to convey to the opposing party's expert witness that he must speak to her?

b) Yes, because the Model Rules do not establish an automatic bar to lawyers initiating contact with the opposing parties' experts.

282. An accountant advised Professor Stevenson to consult a lawyer about a legal problem involving complex questions of tax accounting. Professor Stevenson is easily distractible, and he and does not fully understand the nature of the accounting questions, and he asks his accountant to accompany him to a consultation with his attorney so that the accountant can explain the nature of Professor Stevenson's legal matter to the attorney. The accountant helps to explain the attorney's legal advice in business or accounting terms more understandable to Professor Stevenson. Would attorney-client privilege still protect these consultations against subsequent discovery by government lawyers in a tax enforcement action against Professor Stevenson?

b) Yes, because the accountant is acting as the client's agent in this scenario, just as if her were a foreign language interpreter.

377. Conglomerate Corporation was a defendant in multidistrict litigation, and a plaintiff's attorney represented many different plaintiffs in these related cases against Conglomerate. The attorney and Conglomerate reached a settlement agreement for one group of claimants. The settlement was generous toward those plaintiffs, but it included an agreement by the attorney to withdraw as counsel from representing the other plaintiffs in related cases who had not yet settled their claims. Is the attorney correct in believing it would be improper to sign this agreement with this group of plaintiffs?

b) Yes, because the agreement would be an impermissible restriction on the right to practice law.

347. An attorney employs an experienced legal assistant to manage administrative matters in the firm, including the client trust accounts. The attorney provided the legal assistant with detailed instructions about client trust accounts, including the specific kinds of records to keep, what funds to deposit there, and under what circumstances to withdraw funds. The attorney also sent the legal assistant to attend CLE courses and workshops on IOLTA accounts and managing firm records. Due to the legal assistant's thorough training, competence, and experience, the attorney reviewed the client account books cursorily once a year during the annual review of the employee. Eventually, an audit by the state disciplinary authority revealed numerous discrepancies in the bookkeeping regarding the IOLTA accounts and some prohibited commingling of client funds with the firm's funds. The attorney had no actual knowledge of the discrepancies or problems regarding the client trust accounts. Is the attorney subject to discipline?

b) Yes, because the attorney did not make reasonable efforts to ensure that the legal assistant's conduct was compatible with the professional obligations of a lawyer.

268. Small Firm is considering hiring an attorney, who currently works for Big Firm, in a lateral move. The attorney is a transactional lawyer, so none of the information he possesses is "privileged" in that it was not in anticipation of litigation. To facilitate the checks for conflicts of interest, the attorney discloses to Small Firm the clients he has represented while at Big Firm. This includes the names of persons and issues involved in the matters, as well as names and issues for matters handled by other lawyers in the firm about which the attorney had overheard or otherwise acquired some confidential information. Small Firm uses the information solely for checking about potential conflicts of interest before making an offer of employment to the attorney. The attorney did not ask any of the clients for authorization to disclose the representation or the nature of the issues involved in their matters. Was it proper for the attorney to disclose this confidential information without the consent of the clients?

b) Yes, because the attorney disclosed the information solely to detect and resolve conflicts of interest arising from the lawyer's change of employment.

367. Big Bank hires outside counsel to handle its mortgage foreclosure cases against borrowers who are in default. An attorney agrees to handle a matter for Big Bank, but the engagement contract between the attorney and Big Bank specifies that the attorney may not represent clients in the future who have adversarial claims against Big Bank, and that the attorney agrees to disqualification in any case in which Big Bank would be the opposing party in litigation. The attorney recognized that this term would be unenforceable in court, and he accepted the appointment as outside counsel. Were the attorney's actions improper, under the Model Rules of Professional Conduct?368. An attorney specializes in helping his business clients obtain business loans from commercial lenders. While assisting one client in obtaining an unusually large commercial loan from Big Bank, the attorney noticed a clause in the loan contract by which the borrower promised that its attorney would not seek to obtain similar loans for other parties from Big Bank's primary market competitor in that state. The clause required evidence of a contractual agreement by the attorney - whether with Big Bank or with the client - to this effect. The client desperately needed the loan to survive a temporary downturn in its own industry, and the attorney could easily direct future clients to this same lender, Big Bank, to obtain loans on comparable terms to what the competitor bank offered. In fact, most of the attorney's clients ended up getting their financing through Big Bank, and only rarely had the attorney succeeded in securing loans for clients through the competitor. The contract provision seemed harmless to the attorney, though it would be enforceable. Is it proper for the attorney to sign off on these loan documents for thisterm-5 client, including this clause in the contract?

b) Yes, because the attorney has entered into an employment agreement that restricts his right to represent future clients who sue Big Bank or whom Big Bank sues.

438. A certain attorney is a friend of Blogger, who operates a successful local blog about events, news, and gossip about their city. Blogger includes posts about local judges and well-known lawyers. The attorney has a secret agreement with Blogger. The attorney passes along tips to Blogger in the form of courthouse gossip regarding local lawyers and judges, or even about big cases. Blogger, in turn, covers the attorney's successful cases in glowing terms and recommends the attorney to his readers. Blogger's website is so successful that he earns $50,000 or so in advertising revenue from the site. The attorney occasionally purchases a small, inexpensive advertisement on the site, which merely gives the attorney's name, address, phone number, and areas of practice. Could the attorney be subject to discipline?

b) Yes, because the attorney provides something of value to Blogger in exchange for recommending his services.

298. An attorney met a small business proprietor at a social event, and the proprietor mentioned that he routinely hires lawyers for lease and contract issues. The attorney offered to provide representation for such matters in the future, and gave him his business card, and the proprietor called the next day to engage the attorney to provide these types of legal services. The new client later dropped of boxes of files with documents relating to the matters that the attorney was handling. A few weeks into the representation, the attorney noticed some serious discrepancies and legal issues while reviewing the documents in one of the boxes, and he sent the client an email explaining that he might face regulatory fines and even criminal sanctions if the client did not resolve the matter immediately. The client sent a reply email directing the attorney to shred the entire contents of that box of files, and he did so. A year later, law enforcement officials investigated the client and sought to compel disclosure of the emails between the client and the attorney regarding the boxes of files, including the now-missing files. The attorney claimed attorney-client privilege for the private email communications he had with his client. Should a court compel the production of the emails?

b) Yes, because the communications, though confidential, were in furtherance of committing a crime or fraud.

280. A client who spoke only Spanish hired a local attorney who spoke English and no other languages. The client used an interpreter to communicate an otherwise privileged message to the attorney. The interpreter was an acquaintance of the client. The opposing party later tried to have the interpreter testify at trial about the contents of the conversation he interpreted. The attorney objected that the information falls under the protection of attorney-client privilege. Is the attorney correct?

b) Yes, because the interpreter acted as an agent of the client in facilitating the provision of legal services.

266. An attorney has been practicing for many years, and he is now representing a client who is a notorious celebrity-turned-criminal in a criminal case involving drug charges. The attorney is confused about whether he may publicly disclose information that he learned in confidence from his client if the information is already a matter of public record, and his research indicates there is a split of authority on this question. Seeking clarification, the attorney calls another lawyer who specializes in lawyer malpractice and lawyer disciplinary matters to seek advice about what course of action would comply with the Rules of Professional Conduct. The other lawyer, an expert in legal ethics, agrees to provide an opinion and to keep the conversation a secret. The attorney tries to use a hypothetical to explain the problem, but given the client's national reputation and celebrity status, the other lawyer knows immediately who the client is, and can easily surmise the nature of the confidential information. In addition, the attorney mentions that his client is secretly a bisexual and has been having an affair with both the male and female hosts of a nationally televised morning talk show, though neither of them is aware that the other is having an affair with the same person. Is the attorney subject to discipline for disclosing confidential information about his client?

b) Yes, because the lawyer revealed more client information than was necessary to secure legal advice about the lawyer's compliance with the Rules

308. Attorney Stevenson works in-house as General Counsel for Conglomerate Corporation. Conglomerate's Chief Financial Officer (CFO) resigned suddenly. Due to his background in corporate finance and economics, Conglomerate's Board of Directors asked Attorney Stevenson to serve temporarily as the acting Chief Financial Officer, until they could find a permanent replacement to fill the position. Attorney Stevenson divided his time evenly between corporate financial operations and legal tasks for the company, such as contract review, regulatory compliance, and supervising the outside firms that handle the company's litigation. His financial responsibilities at Conglomerate included reviewing financial reports and forecasts, investment strategy proposals, and various emails or memoranda relating to the firm's financial affairs. An opposing party in antitrust litigation against the corporation seeks to compel production of some of Attorney Stevenson's financial reports and strategy proposals, but he claims these come under attorney-client privilege, as he simultaneously serves as the in-house lawyer for Conglomerate Corporation. Are the documents discoverable at trial?

b) Yes, because these are business communications, not legal advice from the lawyer to the client.

410. An attorney represented criminal defendants, and he received court appointments for indigent defendants. Some of the court appointments he received were female clients. The attorney had a crude sense of humor and progressive views about sexuality, and he often made crude sexual jokes to his female clients, complimented them on their bodies, and half-jokingly made sexual advances or requested sexual favors. The clients normally brushed off these comments, even though they later reported that they felt uncomfortable. None of the clients complained to the court or filed charges with the police for harassment. Could the attorney be subject to disciple and face suspension for these comments and jokes?

b) Yes, because these comments can constitute sexual harassment and could be prejudicial to the administration of justice.

318. A client consults an attorney about the client's indictment for the crimes of theft and unlawful possession of stolen goods. Applicable law treats possession of stolen goods as a continuing offense. The client is still hiding the stolen items in a secret place, and the client asks the attorney about how the client might lawfully return the stolen items. The prosecutor then tries to subpoena the attorney to testify about conversation. Would attorney-client privilege apply to the conversation, if the client's crime is still ongoing?

b) Yes, confidential communications about ways in which Client might lawfully return the stolen goods to their owner are privileged.

342. An attorney works exclusively as a contract lawyer for other firms that need extra help for big cases, whether in pre-trial document review or in background research and writing of briefs. She has no direct contact with the clients of these firms, and she does not participate in important decisions about any of the matters for which she performs legal tasks. Can the attorney avoid being be subject to discipline if a firm uses her contract work in a way that constitutes misconduct, either regarding clients or before a tribunal, assuming she either knows or could have known about the misconduct?

c) No, a contract lawyer has a duty to comply with the requirements of the Rules of Professional Conduct, notwithstanding that the lawyer acted at the direction of another person.

468. A litigation attorney from Big Firm was representing Conglomerate Corporation as the defendant in a civil matter. The attorney learned that the opposing party had hired an expert witness to support their claims, so she decided to initiate an ex parte contact with the expert witness retained to testify for the opposing party, without first obtaining permission from the opposing counsel. The expert witness was hesitant at first to talk to the attorney, because opposing counsel had asked the expert not to discuss the case with the inquiring lawyer. Frustrated, the attorney told the witness that he had to speak to the attorney, under the requirement of law, and that the witness would otherwise face contempt of court charges. She was just bluffing; as with any other witness not under subpoena, an expert witness may choose not to discuss the case with the lawyer. Was it improper for the attorney to convey to the opposing party's expert witness that he must speak to her?

b) Yes, during an ex parte contact, a lawyer may not convey the message, directly or indirectly, that the witness must speak to the lawyer.

339. An attorney worked as an entry-level prosecutor. She did not have a supervisory position or title in her office hierarchy, but merely worked on her assigned cases under the direction and oversight of the higher-ranked lawyers in the office. On one occasion, however, a case arose involving an issue that was important to her, so she asked to be the lead prosecutor on this one case. The District Attorney agreed, and assigned one other lawyer in the office, who was also an entry-level prosecutor, to assist her on the matter. The case had two defendants, and at one point, the attorney leading the prosecution was in one room negotiating a plea arrangement with the first defendant, and the lawyer assisting her was negotiating with the other defendant at the same time in the next room. The state's main witness against the two defendants was a third co-conspirator who had become an informant in exchange for a favorable plea that involved no jail time. The lawyer assisting in the case lied to the second defendant and denied that the state's witness had agreed to a deal. The lawyer had told the lead attorney on the case that he planned to do this beforehand, and she informed him that this would be unethical, but she did not try to stop him from doing so, because she was not his boss. After the negotiations, they met to debrief, and he informed her that he had indeed lied to the defendant and defense counsel about the state's arrangement with their main witness in the case. She reminded him that this violated the ethical rules, but she took no further action, because she was only an entry-level prosecutor, at the same rank as the lawyer assisting on the case. Could the attorney, as lead prosecutor on the case, be subject for the ethical violations in this case?

b) Yes, even if a lawyer is not a partner or other general manager, she directly supervises the work of the other lawyer as lead prosecutor in this proceeding.

338. An attorney was the District Attorney for a local prosecutor's office, and she had several subordinate lawyers working under her authority and oversight. This office had a series of appeals from defendants they prosecuted, and in several cases, the appellate courts reversed the convictions over Brady violations, that is, withholding exculpatory material evidence from defense counsel. Is the District Attorney immune to discipline for these violations?

c) No, because a series of reversed convictions over Brady violations from the same office indicates a lack of training or supervision regarding the ethical duties of prosecutors.

344. An attorney works for a mid-size law firm that employs two or three law students every year as summer associates. The manager of the student associates assigns one of them to work on the attorney's pending antitrust case, in addition to assignments for other lawyers at the firm. While researching a central issue in the case, the summer associate discovered an older Supreme Court decision that was unfavorable to their client. The summer associate decided not to tell anyone about the case, as the opposing party seemed to have overlooked it in their briefs. The attorney was not aware of any of this until they were on a break during their hearing. The hearing was going well for their side, and the associate boasted to the attorney about "burying" that Supreme Court case he had found. The attorney said, "Well, you should have told me about it at the time, but there is no point in bringing it up now, as it appears opposing counsel overlooked it and the hearing is going our way." The judge's clerks, however, found the case, and the judge queried the lawyers about how they could have missed it. Opposing counsel admitted he had been negligent in doing legal research on the matter, and the attorney recounted the story about the summer associate hiding the case from him. Is the attorney now subject to discipline for what the summer associate did?

b) Yes, even though he was unaware of the violation at the time, the attorney ratified the summer associate's conduct after he learned about it.

444. An attorney specializes in criminal defense work. His advertising, signage, and firm brochures offer a service that other lawyers in his city do not provide - the attorney promises to post bail or bond for any client who cannot afford the amount of his bail or bond. Could the attorney be subject to discipline for such an advertisement offer?

b) Yes, given the coercion and duress inherent in the client's incarceration, using the promise of securing the client's release from custody as an inducement to engage the lawyer would be improper.

498. A President appointed an experienced circuit court judge to fill a vacancy on the U.S. Supreme Court. During the Senate confirmation hearings, a committee member asked the candidate about his personal beliefs about abortion. The candidate explained that he thought women had a right to make decisions about their own bodies, and that a fetus is not a person under the law. Was it proper for the candidate to give this answer to the committee?

b) Yes, if he does not say how he would rule in any specific case, the candidate can discuss his views on legal and political issues.

395. A district attorney had a dispute with certain judges in the criminal court in his locale. At one point, the district attorney held a press conference at which he criticized the judges, blaming the large backlog of pending criminal cases on these judges' inefficiency, poor work ethic, and excessive vacations. He went further and mentioned that he would not authorize court funds for DNA testing during police undercover investigations, which hindered the enforcement of vice laws. In conclusion, he said, "All this raises questions about racketeer influences on our lazy judges." The district attorney did not have a reasonable belief that all these statements were true, but at the same time, he was not acting with reckless disregard with the truth. He believed what he said, but he was not entirely reasonable in his belief. Was it permissible for the district attorney to make these statements?

b) Yes, if indeed the district attorney did not make the statements with reckless disregard for their truth or falsity.

336. An associate worked at Big Firm. Even though she had only recently graduated from law school, the associate had earned the respect of the partners at the firm, and she was involved in several projects for multiple lawyers and clients. Overwhelmed with looming deadlines on multiple matters, she realized that she could not devote enough attention to each client's issues - she could not provide competent, diligent representation to so many clients at once. She approached the partner who was her mentor at the firm and explained her concerns, and he responded that she was just experiencing a learning curve, and that her workload was in fact normal, and that she should stop complaining. A few weeks later, the associate was conducting research on a client matter, and she overlooked an important case related to her issue, despite her conscientious work ethic. At the time, she was racing against deadlines on two other projects, was working long hours, and was sleeping only five hours a night on average. Big Firm has a managing partner and a committee of senior partners. Could the partner who was her mentor be subject to disciplinary action for the associate's mistake?

b) Yes, partners and others in a supervisory role at a firm are responsible to monitor the workload of their subordinate attorneys.

316. A client consults an attorney about the client's indictment for the crimes of theft and unlawful possession of stolen goods. Applicable law treats possession of stolen goods as a continuing offense. The client is still hiding the stolen items in a secret place. The prosecutor then tries to subpoena the attorney to testify about the conversations with the client regarding the charges and the legal proceedings. Would attorney-client privilege apply to the conversation, if the client's crime is still ongoing?

b) Yes, privilege covers the confidential communications between the attorney and the client regarding the indictment for theft and possession.

343. An attorney who had only recently graduated from law school, and she received a job offer from a newly elected County Attorney, after volunteering for his campaign. The new attorney did not directly handle cases but assisted trial lawyers with clerical work and non-legal tasks, such as creating public service announcements for websites, social media, and press interviews. The County Attorney soon began a series of highly publicized attacks, including lawsuits and investigations, against political rivals on the County Board and County Courts. At one point, the County Attorney decided to file a federal civil racketeering lawsuit against several of the County Board members. There was no factual support for the allegations. When all the other lawyers in the office refused to be involved in the matter, the County Attorney assigned the case to the new attorney, who had no trial experience, and who was completely unfamiliar with the racketeering statute or case law. She took the case enthusiastically because she was eager to prove herself to the County Attorney; she even tried to amend the complaint to add additional racketeering charges, which were merely duplicative of the existing frivolous charges. She also filed several preemptive pre-trial motions seeking to qualify her expert witnesses and suppress evidence the defendants might try to submit. The court denied the motion to amend the complaint and dismissed the original complaint for having no basis in fact or law. The judge took the additional step of filing a grievance with the state bar against the attorney. In her hearing before the disciplinary committee, the attorney claimed that she was too inexperienced to know that the racketeering charges in her case had no basis in fact or law, and that she merely deferred to the guidance and instructions of the County Attorney. Could she be subject to discipline despite these mitigating factors?

b) Yes, regardless of the directions of her superiors or her inexperience, she had a duty not to bring a frivolous proceeding or assert a frivolous issue in litigation.

341. An attorney had recently graduated from law school and entered the practice of law. After a federal clerkship, he went to work for Big Firm, which paid the highest associates' salaries in the state. A partner at Big Firm gave the attorney an assignment to represent the teenage daughter of one Big Firm's most important clients, a billionaire social media entrepreneur. The daughter had been part of a group of student protesters that the police had arrested the previous week for trespassing. When arrested, the daughter had given the police a friend's driver's license and identified herself as the friend, who had a similar appearance. The police mistakenly charged the daughter under the friend's name, and the district attorney proceeded to prosecute her under the mistaken identity. The friend, whose name and driver's license the daughter had used, was unaware that she was the named defendant in a misdemeanor criminal case, and the billionaire's daughter, who was now the attorney's client, continued with the ruse even as she remained in custody along with the other protestors. During a private consultation with her, the attorney asked about the name discrepancy, as he was expecting to represent the daughter of Big Firm's client, and the girl explained the false identification, and insisted that the attorney not disclose her real identity to the police or the court. Back at the firm's office, the attorney approached the partner who had assigned the case, but before the attorney could finish explaining the name problem, the partner said, "Do not mess this up, her father is an important client of the firm. Convince the court to drop the charges as quickly as possible. Close this matter quickly." The attorney spoke to the prosecutor and convinced her to dismiss the case, but the attorney never told her about the misidentification of his client. After the dismissal of the case, the attorney met with the billionaire's daughter and her mother, together with the friend whose name she had used and the friend's parents, who were upset that their daughter had been a named defendant in the matter in the first place. Despite the attorney's efforts to reassure the friend's parents that the state dropped the charges, the friend's parents contacted the prosecutor's office in hopes of removing the arrest from their daughter's record. When the prosecutor realized what had transpired, he reported the attorney to the state bar disciplinary authorities. Could the attorney, as an inexperienced new associate at Big Firm, be subject to discipline for this matter?

b) Yes, regardless of the directions the attorney received from the partner at Big Firm or from the client, he is subject to discipline for failing to disclose a material fact to a tribunal when disclosure was necessary to avoid assisting a criminal or fraudulent act by a client.

434. An attorney received a client referral by email from a friend who worked as a nurse in a nearby emergency room. The client called the attorney's office the same day that the attorney received her friend's email about this potential client. The representation of this new client yielded a favorable outcome, with a generous damages award for the client and substantial fees for the attorney. In appreciation for the referral, the attorney sent her friend a collectible Star Wars statue (plastic figurine) worth about $20, knowing that the friend avidly collected Star Wars statues. Accompanying the figurine was a thank you card expressing appreciation and promising to send Star Wars collectible figurines every time the friend referred a client to the attorney. Did the attorney act improperly in this instance?

b) Yes, the Model Rules prohibit gifts offered or given in consideration of any promise, agreement or understanding that such a gift would be forthcoming or that referrals would be made or encouraged in the future.

441. In his advertisements and firm brochures, an attorney describes his many years of experience litigating in a specific area of commercial real estate litigation, without claiming to be a specialist or an expert. He does not mention any official certification. Is it permissible for the attorney to boast of his years of experience practicing in a specific area, even though some readers might infer from this that he is an expert or a certified specialist?

b) Yes, the Supreme Court has held that state bars cannot prohibit lawyers form describing their years of experience with certain types of cases, assuming the information is truthful.

405. An attorney worked with a partner who developed a chronic debilitating medical condition. Eventually, the condition materially impaired the partner's ability to practice law, but the partner could not cope with giving up on her career, and she kept practicing. She began to miss court deadlines, to forget to make certain filings to complete transactions, and not to follow through to perform agreed-upon tasks. Under Model Rule 1.16, the partner had a duty to decline or withdraw from representation for clients, at least for the more challenging tasks. On the other hand, up to now no clients had suffered serious prejudice to their legal interests or claims because of these mistakes. Does the attorney who observes these developments have a duty to report her partner for misconduct under Rule 8.3?

b) Yes, the partner's lack of fitness has evidenced itself through a pattern of conduct that makes clear the lawyer is not meeting her obligations under the Model Rules.

281. An attorney agreed to represent an underage client in a legal matter. The client was fifteen years old, and the youth's parents were present at the consultations and other meetings with the attorney. Would the presence of the parents during confidential communications between the attorney and the underage client waive the protection of attorney-client privilege for the conversation?

c) No, because the parents are there to facilitate the representation on behalf of their minor child.

335. A certain attorney worked at Big Firm, and she was supervising a new associate lawyer there. During a negotiation for the sale of a company, in which Big Firm represented the seller, the associate informed the buyer's lawyers that certain assets of the company had no liens or other encumbrances on them, and that she had verified this herself. This was a misrepresentation - the properties had significant encumbrances, which the purchase price should have reflected, but it did not. The supervising attorney, who was part of the conversation when the associate made the misrepresentation, did not correct her, because she did not want to humiliate her in front of the opposing party, or reveal an internal discord among Big Firm's lawyers. Instead, the supervising attorney lectured the associate about the misrepresentation privately the next day, and he told her not to let it happen again. Then they agreed to drop the matter, and the supervising attorney instructed the associate to watch for a good opportunity to bring up the mistake and clarify the matter for the buyer. The associate never did so. Could the supervising attorney be subject to discipline for failing to correct the resulting misapprehension by the buyer?

b) Yes, the supervising attorney had a duty to take affirmative steps to correct the misapprehension of the other party, sometime before the consummation of the purchase.

430. Attorney McLemore grew up in a family that spoke the Witchita language in the home. Her law practice advertisements prominently stated that she spoke Witchita, and that she can represent Witchita-speaking clients. Unfortunately, Attorney McLemore was the last known native speaker of the Witchita language. Was it improper for Attorney McLemore to include this language ability in her advertisements?

c) No, because the statement is true.

404. Which of the following statements, made publicly by an attorney, would be impermissible under Model Rule 8.2? a) A lawyer's motion for new trial claiming judge's gestures and expressions demonstrated bias b) A lawyer's statements that judges in his state were "not learned in the law" and were "laughed at" throughout country c) A lawyer's statement implying the judge must have been thinking primarily about the political ramifications of his ruling d) A lawyer's letter stating that the way in which the legislative ethics commission conducted its proceedings "gave cause for some to speculate that the deck was stacked," when the lawyer had factual evidence to support the accusation.

c) A lawyer's statement implying the judge must have been thinking primarily about the political ramifications of his ruling

321. Prosecutors from the Department of Justice (DOJ) began an antitrust investigation into Conglomerate Corporation, and the DOJ began questioning some of Conglomerate's business customers. Conglomerate's attorney prepared a memorandum analyzing the antitrust implications of Conglomerate's standard contract form with commercial purchasers. Soon thereafter, some Conglomerate employees received subpoenas to testify before a grand jury that was investigating the same antitrust issues in their industry. The attorney worried that the grand jury would indict Conglomerate, so she interviewed the employees herself and prepared a debriefing memorandum. Would the attorney's two memoranda described above come under the protection of the work product doctrine?

c) Both the lawyer's memorandum analyzing the contract form and the lawyer's debriefing memorandum were prepared in anticipation of litigation, because a grand jury proceeding is itself litigation.

363. An experienced attorney has an office in State X, and she is duly licensed to practice law in that state. The attorney's office is in a city on the border of State Y, and the attorney does not have a license to practice there. The attorney represents a regulated utility, which operates a power plant in State X near the border with State Y. The attorney's representation of the utility mostly pertains to environmental issues, obtaining necessary permits, and complying with federal and state regulations of utilities. Occasionally, the utility also has issues relating to compliance with the environmental and permitting laws of State Y because of those same activities. Is it permissible for the attorney to travel to State Y to deal with governmental officials regarding regulatory issues arising out of the utility's activities?

c) It is permissible because the legal issues arise out of or relate closely to the attorney's practice in a jurisdiction in which the lawyer is admitted to practice.

452. An attorney represented a criminal defendant charged with murder. During their consultations, the client informed the attorney that he had committed another murder, but that someone else - an innocent bystander - was standing trial for that crime. The attorney was aware of this other case, as it had received media coverage, and realized that an innocent person would potentially go to jail for many years, or even face the death penalty, for the crime his own client had committed. Which of the following is true, regarding the attorney's ethical obligations in this situation?

c) It would be permissible for the attorney to urge his own client to come forward and confess to this other murder to save the innocent person accused of it, even though such advice would be contrary to his own client's legal interests.

276. An attorney drafted a confidential email to a client offering legal advice on a tax matter. The client had sought the attorney's legal opinion on the question. The attorney's answer relied partly on information that the client had provided, partly on information the attorney himself obtained from third parties, and partly on the attorney's own legal research on Westlaw. When the IRS later brought an enforcement action against the client, the government lawyers sought discover of this email, hoping to find useful evidence about the defendant's financial activities and whether the defendant had knowingly violated the tax code. Can the government lawyers obtain the email through discovery?

c) Neither the attorney nor the client would have to disclose or testify about any of its contents.

436. An attorney advertised his firm's services on the printed paper placemats and drink coasters at a local diner. He also convinced the owner of the diner, for a fee, to let him hand ad posters for his firm over the urinals in the men's restroom. Has the attorney violated the Model Rules with these advertising activities?

c) No, a lawyer may communicate information regarding the lawyer's services through any media.

471. An attorney knows that his opposing counsel has a reputation for refusing to settle cases and forcing lawsuits to go to trial, to impose the full costs of litigation on the opposing party. Cultivating this reputation serves as a deterrent to other would-be litigants against opposing counsel's clients. To avoid a rebuff by opposing counsel, the attorney finds a close friend of the opposing party, and he asks the close friend to communicate an informal settlement offer to the opposing party directly, bypassing the other lawyer. The opposing party is delighted to hear the offer and readily agrees to settle the case. Opposing counsel is furious and reports the attorney for misconduct. The attorney claims that he did not communicate with opposing counsels' client. Instead, the friend did, so the prohibitions on contact with other parties would not apply. Is the attorney correct?

c) No, a lawyer may not make a communication prohibited by the Rules through the acts of another, such as the friend in this case.

466. A prosecutor was conducting a plea negotiation with a defendant and his lawyer. During the plea negotiation, the prosecutor told the defendant and his counsel that there was an eyewitness to the alleged crime, who could identify the defendant as the perpetrator. This was not the case - the prosecutor was just bluffing, and the defense counsel suspected it was not true and decided to wait on deciding anything until he could depose or interview the witness himself. Was it permissible for the prosecutor to bluff like this during a plea negotiation, if no harm resulted?

c) No, a lawyer shall not knowingly make a false statement of material fact or law to a third person.

384. An attorney developed expertise regarding the area of legal ethics and legal malpractice. Another firm hired the attorney to testify as an expert in an adjudication about the reasonableness of the firm's fees. The attorney has testified as an expert regarding legal fees and legal ethics on several prior occasions. During his cross-examination by the lawyer representing the opposing party, the attorney had to answer questions that forced him to disclose some unfavorable information about the client of the firm that had hired him as an expert. The attorney did not object that the information was confidential or attempt to assert privilege; he answered the questions frankly and objectively. If he had been representing the client directly, the disclosures would have clearly violated his duty of confidentiality. The answers were a setback to the interests of the party that had hired him, and the lawyers and their client were upset. Could the attorney be subject to discipline for his actions while testifying as an expert witness?

c) No, a lawyer testifying as an expert is not providing law-related services and does not have a client-lawyer relationship with the party that has hired him.

337. An attorney had supervisory responsibilities for a new lawyer at her firm, but she had her own cases and clients to handle. It was a busy season for the firm, so the attorney did not check on the associate herself, but she would take time to answer questions if the associate approached her. The associate needed more oversight and direction that she received, and she committed several serious ethical violations. The supervising attorney had no way of knowing about these because the associate was always careful to cover up her mistakes or blame others when something went wrong. Could the supervising attorney avoid responsibility for the associate's ethical violations even if she did not direct, ratify, or have knowledge of the associate's misdeeds?

c) No, an attorney having direct supervisory authority over another lawyer must make reasonable efforts to ensure that the other lawyer conforms to the ethical rules, even apart from the supervising attorney directing, ratifying, or even knowing about a specific violation.

478. An attorney sees a friend at a high school reunion. The friend asks the attorney for advice about a potential civil lawsuit that the friend is considering hiring an attorney to file. The attorney gives the friend general information about the area of law and about the specific kind of lawsuit an attorney might potentially file for the friend. The friend lives too far away from the attorney for the attorney to handle the case, and the friend is planning to hire another lawyer near his residence to handle the lawsuit. The attorney later talks to his own wife about the friend's lawsuit. Wife discusses the suit with her own friend. The friend discovers that several people know about his potential suit and is upset, as he believed that the attorney should not have spoken about his potential case to others. Is the attorney subject to discipline?

c) No, an attorney owes no duties or protections, including protections against disclosing information about potential lawsuits, to persons who communicate with attorneys without any expectation of forming a client-attorney relationship.

437. An attorney made an informal agreement with Physician that they would refer clients to each other when the situation seemed appropriate. They did not pay each other any money for referrals, but the relationship was explicitly reciprocal - the attorney referred patients who needed medical examinations to Physician, and when Physician had patients needing legal representation, he referred them to the attorney. The relationship was explicitly exclusive - each agreed not to refer clients to others - but it happened that neither had similar reciprocal relationships with anyone else anyway. They always inform their clients when making such referrals that they have a reciprocal relationship. Is such an arrangement proper?

c) No, because a lawyer may not agree to refer clients to another lawyer or a nonlawyer professional, in return for the undertaking of that person to refer clients or customers to the lawyer, if the relationship is exclusive.

423. An attorney had a license to practice law in two jurisdictions - his home state where he lived and had his primary office, and a neighboring state where he represented several clients each year. The attorney committed serious professional misconduct in his home state and received a public reprimand from the state disciplinary authorities. All the conduct took place in his home state, the client resided in the state, and the representation took place entirely within his home state. The lawyer's conduct would have violated the rules in either of the jurisdictions where he had a license to practice law, because it involved commingling client funds with his own money, and the states mostly had identical rules concerning this activity. After the attorney received a public reprimand in his home state, where the misconduct occurred, the state bar disciplinary authority in the neighboring state (where he also practiced) then commenced disciplinary proceedings against him as well. In the end, the neighboring state bar suspended his license for six months in that state, a much more severe sanction than the public reprimand he received in his home state, where the misconduct in fact occurred. The attorney claims that the neighboring state bar has no jurisdiction over conduct that occurred entirely outside of the state. He also objects that the second punishment raises double jeopardy concerns. Is the attorney correct?

c) No, because a lawyer may be subject to the disciplinary authority of two jurisdictions for the same conduct and may receive different sanctions in each state.

469. An attorney surreptitiously recorded a conversation with a potential witness without the other person's knowledge or consent. State law permits recording of conversations when at least one of the participants consents, which would include the attorney in this case. The potential witness learned about the recording later and was upset, because she would not have consented to the recording of the conversation, or at least would have been more judicious about her comments. Even if the attorney did not violate state or federal laws by recording this conversation, could the attorney be subject to discipline for failing to disclose a material fact to a third person?

c) No, because a lawyer may electronically record a conversation without the knowledge of the other party to the conversation without violating the Model Rules, if the recording is not otherwise illegal.

390. An attorney agreed to represent an applicant to the state bar - a recent law school graduate - in her hearing before the state bar admissions board, which had tentatively denied her application for making false statements on her bar application. The board formally requests the applicant and her attorney make full disclosures about the events in question to help resolve the matter. The client (bar applicant) explains the entire situation to her attorney, including some self-incriminatory information - it turned out that the applicant's misbehavior had been much more serious than the board was aware. The attorney did not disclose this latest information, which would have made it much clearer to the board that the applicant lacked the character and fitness to practice law. Could the attorney be subject to discipline for this action?

c) No, because a lawyer representing an applicant for admission to the bar, or a subject of a disciplinary action, comes under the rules applicable to the client-lawyer relationship, including the duty of confidentiality.

451. An insurance company retained an attorney to represent one of its policyholders (i.e., an insured) against a lawsuit. The insurance company that hired the attorney requires its retained counsel to follow its own litigation management guidelines, designed to monitor the fees and costs of the lawyers the insurer retains. The litigation management guidelines include the requirement of a third-party audit of legal bills. Although the guidelines usually serve the interests of both the insured and the insurer by keeping litigation costs low and expediting the resolution of the case, in this instance the attorney finds that the guidelines require tactical moves that are adverse to the insured's interests. The insurer claims that the insured impliedly consented to the guidelines by agreeing contractually in the insurance policy to "cooperate" during litigation. The insurance company hired the attorney for the case. Should the attorney comply with the insurer's litigation management guidelines?

c) No, because a lawyer shall exercise independent professional judgment, and the insurer's litigation management guidelines in this instance materially impair the lawyer's professional judgment.

439. In his advertisements, an attorney, who practices in California, states, "CERTIFIED SPECIALIST IN CALIFORNIA LAW." The attorney is referring to the fact that he passed the California Bar Exam, not to any other official certification beyond admission to the California bar. According to the Model Rules of Professional Conduct, is such a statement proper in a lawyer's advertisement?

c) No, because a lawyer shall not state or imply that a lawyer is a certified specialist in a specific field of law without being a certified specialist by an official certifying organization in that state, and without including the name of the certifying organization in the advertisement.

368. An attorney specializes in helping his business clients obtain business loans from commercial lenders. While assisting one client in obtaining an unusually large commercial loan from Big Bank, the attorney noticed a clause in the loan contract by which the borrower promised that its attorney would not seek to obtain similar loans for other parties from Big Bank's primary market competitor in that state. The clause required evidence of a contractual agreement by the attorney - whether with Big Bank or with the client - to this effect. The client desperately needed the loan to survive a temporary downturn in its own industry, and the attorney could easily direct future clients to this same lender, Big Bank, to obtain loans on comparable terms to what the competitor bank offered. In fact, most of the attorney's clients ended up getting their financing through Big Bank, and only rarely had the attorney succeeded in securing loans for clients through the competitor. The contract provision seemed harmless to the attorney, though it would be enforceable. Is it proper for the attorney to sign off on these loan documents for this client, including this clause in the contract?

c) No, because an attorney must not make an agreement restricting the attorney's right to practice.

352. An attorney agrees to buy the successful law firm of a fellow lawyer who recently succumbed to terminal cancer. The sale includes the office building, the library and furnishings, and the good will of the firm, and conforms to the provisions of Rule 1.7. The purchasing attorney pays $100,000, the agreed-upon purchase price, to the executor of the deceased lawyer's estate, but the executor is not a lawyer. The funds for the purchase came from the contingent fees in a recent personal injury case won by the purchasing attorney. Was this transaction improper?

c) No, because an attorney purchasing the firm of a deceased lawyer may pay the executor the agreed-upon purchase price.

400. A would-be judge asked his former law school classmate, a practicing lawyer, to write a recommendation letter for him as part of his application and vetting process for a judicial appointment. The attorney obliged and wrote a glowing recommendation, entirely favorable, even though he personally knew that his friend (the one seeking to be a judge) was an alcoholic. Was is proper for the attorney to write such a letter?

c) No, because assessments by lawyers are relied on in evaluating the professional or personal fitness of persons under consideration for appointment to judicial office, so expressing honest and candid opinions on such matters contributes to improving the administration of justice.

450. An attorney made substantial financial contributions to the reelection campaign of an elected judge. The judge won reelection, and he showed his gratitude to the attorney by frequently appointing him to serve as referee or mediator in situations where the attorney received no compensation except reimbursement for travel expenses. The attorney made the donations because he hoped to receive such appointments, but he received no fees as a result. Could the attorney be subject to discipline for accepting these appointments?

d) No, because the term "government legal engagement" does not include mostly uncompensated services.

376. An attorney brought a class action lawsuit against Conglomerate Corporation and was remarkably effective in her efforts, mostly because she was brilliant about forum shopping. After discovery, Conglomerate realized they needed to settle the case before trial. Conglomerate offered a very generous settlement to the plaintiff class, including the full amount sought as recovery in the pleadings, plus reasonable attorney's fees. Conditions of the settlement included a nondisclosure agreement about the terms of the settlement, and an agreement with this attorney limiting venue and forum options in future cases against Conglomerate brought by non-settling plaintiffs. The settlement imposed no other restraints on the attorney. Assume that the attorney did not care anymore about forum and venue, because she had learned enough about Conglomerate Corporation's activities that she thought she could easily win future cases in any court. Is this agreement proper, under the Model Rules?

c) No, because even limiting the attorney's ability to shop for forum or venue in future cases for other plaintiffs would be an impermissible restriction on the attorney's ability to practice law.

374. An attorney represented a plaintiff in a claim against Conglomerate Corporation and was remarkably effective in her efforts, mostly because she hired Professor Stevenson as an expert witness. After the deposition of Stevenson, Conglomerate realized they needed to settle the case before trial. Conglomerate offered a very generous settlement to the plaintiff, including the full amount the plaintiff sought as recovery in its pleadings, plus reasonable attorney's fees, and even some additional stock options in Conglomerate Corporation. Conditions of the settlement included a waiver and release of all the plaintiff's claims, including potential claims not part of this lawsuit, and an agreement by the attorney never to use Professor Stevenson again as an expert witness in a case against Conglomerate. The settlement imposed no other restraints on the attorney, and it did not restrain Professor Stevenson from serving as a fact witness (as opposed to expert) in the future. Assume for this question that Professor Stevenson is not a licensed attorney in this jurisdiction. Is this agreement proper, under the Model Rules?

c) No, because even limiting the attorney's ability to use a specific expert witness against this defendant would be an impermissible restriction on the attorney's ability to practice law.

459. The chief financial officer of Investors' Club, a private investment trust, is under suspicion for converting $100,000 of Investors' Club's assets for personal use. The other responsible corporate officers of Investors' Club, acting on the trust's behalf, retain an attorney to recover the money from the chief financial officer. At the same time, they direct the attorney not to reveal the loss, or file a lawsuit, until she has first exhausted other collection efforts. Given these restrictions, would it be a conflict of interest for the attorney to proceed with the representation?

c) No, it would certainly be proper for the attorney to represent Investors' Club, and in doing so she must proceed in the manner directed.

263. An attorney is a partner in a seven-lawyer firm. The client retained the attorney to handle his workers' compensation matter. Yet the attorney did not discuss with the client that he would normally disclose to the other partners in the firm some of the details about his cases and clients. At the weekly meeting of the partners, as everyone discussed their pending cases, the attorney explained the client's case and solicited input from the partners. One partner had an ingenious suggestion that would have been quite helpful to the client's case. The attorney mentioned to the client in their next phone call that one of his partners had made a brilliant suggestion that could turn the case in the client's favor. The client was upset that the attorney had discussed the case with anyone else. Is the client correct that the attorney should not have discussed the case with the others at the firm?

c) No, because lawyers in a firm may disclose to each other information relating to a client of the firm, unless the client has instructed that certain information be available only to specified lawyers.

418. An attorney was an immigrant from a country that permits polygamy - men can have up to four wives. The attorney had two wives, which his religion permitted, as did the laws of his homeland. Nevertheless, his multiple marriages constituted bigamy in the American jurisdiction where he practiced law, and eventually a court convicted him of bigamy and imposed a fine. Could the attorney be subject to professional discipline for committing this illegal act?

c) No, because offenses concerning personal morality, such as bigamy and comparable offenses, have no specific connection to fitness for the practice of law.

475. A business owner hires an attorney to enforce a non-compete agreement against a former executive at the client's technology firm. According to the client, a rumor started going around just this past week that the former executive had either started his own business nearby or was consulting for a nearby competitor; if true, either scenario could violate the non-compete agreement. The client explains that the former executive has already asserted that the non-compete agreement is invalid under a recent decision from the state Supreme Court and is filing an action for a declaratory judgment to challenge the non-compete agreement preemptively, though the client is unsure whether his company received proper service yet about the lawsuit. The attorney decides that the first step is to call the former employee and ask him whether he has found another job yet or has started his own business. The former employee answers the phone, explains that he has started his own rival company, and that he believes the non-compete agreement is invalid under state law. The attorney asks him to have his own lawyer contact him so that they can discuss potential settlement for the dispute. Has the attorney acted properly?

c) No, because one can easily infer from these facts and circumstances that the attorney indeed knew the former employee had representation.

312. Conglomerate Corporation had an accident occur at one of its chemical manufacturing facilities - a large explosion killed several workers and injured many others. Soon after the incident, at the behest of Conglomerate's corporate managers, the general counsel obtained statements from employees and other witnesses about what happened, memorializing the statements in written form. Later, the family of an employee killed in the accident sued Conglomerate, and the plaintiffs' interrogatories included a demand for the contents of the written statements taken by the corporate general counsel. Must Conglomerate Corporation disclose the statements taken by its attorney after the accident? [pick the best answer]

c) No, because the statements are communications protected by the attorney-client privilege.

305. An attorney represented a personal injury plaintiff in a lawsuit. While trying to find potential witnesses to support the client's litigation claims and personal credibility, the attorney met with several people neighbors and friends of the client, asking about the incident that injured the client, as well as the client's character and past behavior. One of the client's neighbors told the attorney several disturbing stories about wild parties at the client's house, and disreputable character who frequently visited the home. Later, at trial, the defendant sought to compel the attorney to disclose the information conveyed by the client's neighbors. The attorney objected that this information falls under the attorney-client privilege and is therefore inadmissible. Is the attorney correct in this assertion?

c) No, because the information did not come from the client, and therefore attorney client privilege does not apply.

495. A trial judge is going through a divorce, and he hired an attorney to represent him. The attorney's law firm partner is representing another client who is appearing before the same judge in his personal injury lawsuit. The judge and the litigation client both give written informed consent to the representation despite the potential conflicts of interest. Even so, the judge is trying to keep the divorce quiet until after the upcoming elections, because this occurs in a state with elected judges. The judge therefore refuses to disclose to the parties in the personal injury case that counsel for one side is from the same firm as the lawyer representing the judge in his pending divorce. Neither the attorney nor his partner can reveal to opposing counsel in the personal injury case that their firm represents the judge, due to their duty of confidentiality. The judge believes he will be unbiased in the personal injury case, even though he is the client of a partner of one of the lawyers in the case, so the judge does not need to disqualify himself from the case. The Code of Judicial Ethics does require, however, that the judge disclose the representation to the litigants appearing before him, which the judge has refused to do at this time. Can the attorney continue representing the judge in his divorce?

c) No, because the lawyer must withdraw from the representation of the judge under these circumstances.

356. A husband and wife are both attorneys in Puerto Rico, though they attended law school in Florida. They have practiced in Puerto Rico for ten years and have a license to practice there. Last year, they moved to Florida, where the wife took the state bar exam and gained admission to the Florida bar. They have now opened a law office in Florida with both of their names listed on the firm letterhead, followed by the phrase "Attorneys at Law." The husband confines his practice exclusively to Puerto Rican clients who are living in Florida or are visiting there; the wife handles all other legal matters. It is proper for them to use such letterhead?

c) No, because the letterhead reveals that the wife is aiding her husband in the unauthorized practice of law.

302. A former employee is suing Conglomerate Corporation. The employee claims that Conglomerate fired him as retaliation for uncovering internal corruption at the company. While he still worked for Conglomerate, the employee had several email exchanges with Conglomerate's in-house counsel about the problems he had uncovered and the consequences for reporting them. Now that litigation has ensued after his termination, he requests production of all his email exchanges with in-house counsel (he no longer has access to the company's email server). Conglomerate's lawyers assert that these conversations are privileged, because the emails were between a Conglomerate employee and its corporate counsel. Would the emails come under the protection of attorney-client privilege, given these facts?

c) No, because the privilege belongs to the client, and the plaintiff here owns the privilege.

303. Two codefendants stood trial on an arson charge, each represented by separate counsel. The first defendant, through his attorney, offered to tell the prosecutor about some valuable eyewitnesses that would help the prosecution's case against the other defendant, in exchange for a plea agreement that included no jail time for the first defendant. The prosecutor declined the offer and continued with the prosecution of both defendants. The first defendant, who had offered to make the disclosures, died unexpectedly in a violent prison fight. The prosecutor then called the deceased defendant's attorney and asked him to disclose whatever information he had about these additional witnesses that would strengthen the case against the remaining defendant. The attorney was unsure about whether attorney-client privilege applied, but the prosecutor insisted it did not apply after the defendant's death. Is the prosecutor correct?

c) No, because the prosecutor declined the offer of disclosure at the time, and the privilege survives the client's death.

392. An attorney faced disciplinary action over a client grievance. The disciplinary tribunal asked the attorney several probing questions about her handling of client funds. The attorney had, in fact, used some client funds to pay off a gambling debt, so she was less worried about a temporary suspension of her law license than about potential criminal charges for embezzlement. The attorney, therefore, invokes her Fifth Amendment privilege against self-incrimination and refuses to answer the questions. The disciplinary tribunal then determines that it lacks substantial evidence that the attorney mishandled client funds, but it commences disciplinary proceedings over the attorney's refusal to answer some of its questions. Could the attorney be subject to discipline for refusing to answer the questions in this scenario?

c) No, because the rules requiring attorney candor to disciplinary authorities are subject to the provisions of the Fifth Amendment of the United States Constitution and corresponding provisions of state constitutions.

427. Three attorneys open a new firm (a partnership) together. They drafted the partnership agreement themselves, without hiring another lawyer to represent them, and none of them gave informed consent, confirmed in writing, to the conflicts of interest that might arise as a result of drafting their own partnership agreement and trying to represent their own interests at the same time. The partners decided to call the firm "City of Houston Litigation Center," named after the city where they practice. Their advertising, brochures, and signage contain no disclaimers disavowing any connection with the Houston municipal government or with the Houston City Attorney's Office, which is a department of the municipal government. Are the actions of the attorneys described here proper, according to the Model Rules of Professional Conduct?

c) No, because their trade name includes a geographical name without express statements that they are not a public agency or subdivision of government.

369. Big Bank routinely hired lawyers as outside counsel on various matters, and it required each one to sign an Outside Counsel Agreement (OCG) as part of its contract of engagement for legal representation. Big Bank's OCG included the following provision: Notwithstanding the rules and opinions set forth in ABA or state ethical opinions, regulations, or cases applicable to outside counsel, outside counsel agrees to treat Big Bank and all its subsidiaries as one entity for analyzing conflicts of interest. Big Bank will ordinarily give informed consent, confirmed in writing, to waive conflicts in transactional matters, whenever the bank's interests will not be impaired. For conflicts of interest, Big Bank shall include all organizations and entities delineated in the attached APPENDIX, which Big Bank may amend at any time. An attorney has an opportunity to work as outside counsel for Big Bank on a specific matter, but she is concerned about this provision. Would it be proper for the attorney to accept this OCG by contractual agreement?

c) No, because this agreement impermissibly restrains the attorney's right to practice.

425. An attorney was representing a client in a probate matter. The representation mostly occurred within the attorney's home state, where the client also lived. One asset of the probated estate, however, was an account receivable from a debtor in a neighboring state; the matter was already the subject of pending contract litigation in that state. The attorney filed a pro hac vice appearance in the neighboring state, and he traveled there to represent his client in the contract matter, which was ancillary to the probate matter in his home state. During the proceedings, the lawyer committed an act that constituted a violation of the ethical rules in his home state, but not in the neighboring state where he was appearing in a proceeding; the states had different rules in this regard. Could the attorney be subject to discipline in his home state for violating its rules before a tribunal in the neighboring state?

c) No, because whenever a lawyer's conduct relates to a proceeding pending before a tribunal, the lawyer shall be subject only to the rules of the jurisdiction in which the tribunal sits.

396. A criminal defense attorney received a court appointment to represent a defendant, and at the end of the representation, she sought compensation for her legal fees from the appropriate courthouse office. Unfortunately, she did not have some of the receipts and documentation to verify some of her fees, so she received only half of the compensation she expected. Angered by this incident, the attorney sent a letter to the judge's secretary, in which he harshly criticized that local court's administrative system for compensating appointed counsel. The letter declared that he would not submit the additional documentation required for compensation, even if that meant he could no longer accept court appointments from the judges in that courthouse. An objective reader would have thought the letter "exhibited unlawyerlike rudeness," as one of the judges at the courthouse put it. Could the attorney be subject to suspension of his law license for sending this letter?

c) No, even though the bar has a right to place restrictions on lawyer speech, the complaints here would be permissible under the Model Rules and First Amendment jurisprudence.

406. An attorney practiced as in-house counsel within Conglomerate Corporation. She learned of serious ethical misconduct there by a fellow employee who was also a licensed lawyer, but who was employed by the Conglomerate in a nonlegal position as a technical writer. Conglomerate does not have any liability or legal responsibility for the employee's misconduct, so the attorney is not approaching it as a liability concern for her corporate client. Would it be permissible for the attorney to refrain from reporting the employee's misconduct to the bar?

c) No, if a lawyer knows of professional misconduct of another licensed lawyer, even a non-practicing lawyer, must report it where it raises a substantial question as to that lawyer's honesty, trustworthiness, or fitness as a lawyer.

334. An attorney served as the director of the Environmental Enforcement Division of the state Attorney General's office, which brought legal actions against polluters in the state. The Attorney General's Office hired only lawyers with three years' experience or more - they never hired new law school graduates. In the Environmental Enforcement Division, all the lawyers had many years of experience as litigators in that field. The attorney who served as director oversaw the prioritization of cases and implementation of the Attorney General's policy objectives, and assigned cases to the lawyers in her Division, but did not need to monitor their work, train them in legal ethics, or watch for ethical violations, because all the lawyers were competent and experienced. It turned out, however, that one of the lawyers committed some ethical violations, such as testifying as the key witness in a trial in which he was the attorney of record for the state, which was the plaintiff or prosecuting party in the cases. In another instance, the lawyer brought an enforcement action that had no factual basis in retaliation against an entity that had defrauded the lawyer of a substantial amount of money. When these violations received attention in a local new station expose, the lawyer resigned in disgrace, and the Attorney General took the position that the director of the Environmental Enforcement Division is not responsible for the actions of this individual lawyer, whom he described as a "bad apple" in the Division. Is he correct?

c) No, lawyers having comparable managerial authority in a government agency must make reasonable efforts to ensure that the firm has in effect measures giving reasonable assurance that all lawyers in the agency or department conform to the Rules of Professional Conduct.

462. An attorney represented Conglomerate Corporation in negotiating with an office supplies company for a bulk discount on regular monthly purchases. When the supplier refused to go any lower, the attorney said threateningly that he could pick up the phone at any time and get three of their competitors to beat the supplier's current price. The attorney had no reason to think this - he was just bluffing, hoping to leverage the supplier into a lower price. Under the Model Rules, was it impermissible for the attorney to make this false statement to a third party?

c) No, such remarks are merely posturing or puffing, and are not statements upon which parties would justifiably rely, so they are not false statements of material fact.

372. An attorney worked as in-house counsel at Conglomerate Corporation. Her employment agreement with Conglomerate Corporation that she would not, following her employment there, represent any client in litigation against Conglomerate. General Counsel for Conglomerate maintained that this was necessary to prevent lawyers who left there from using confidential information they learned during their time at Conglomerate against the company in litigation thereafter. In other words, the contractual provision merely mirrored the duties a lawyer in that situation would have under the conflicts of interest rules. Would this agreement be enforceable, if the attorney left Conglomerate Corporation and then represented a client who had a contract claim against the company?

c) No, the agreement places an impermissible restriction on the attorney's ability to practice law, and it goes beyond the constraints of the conflict of interest rules.

349. Attorney Barrett was the managing partner at a small law firm. Barrett hired Cooper, an ordained minister who had been unemployed, as a legal assistant at the firm. Cooper's main job at the firm, however, was to bring in new clients. Cooper received a minimum-wage base salary, but also received large bonuses for bringing in clients who generated fees for the firm, and the combined bonuses each year exceeded $100,000. The firm paid for Cooper to complete a certification course to become a hospital chaplain, which gave Cooper chaplain's access to emergency areas of hospitals to visit accident victims and their families. He would offer to pray with them, but he would also give them a business card from Barrett's firm. In this way, Cooper brought several high-payoff personal injury clients to the firm. Cooper also recruited clients from the local church where he served as a "biblical counselor." Is it proper for the firm to pay Cooper bonuses for bringing fee-generating clients to the firm?

c) No, the arrangement constitutes an improper sharing of fees with a nonlawyer.

340. An insurance company routinely hired outside counsel to represent its policyholders in litigation under liability policy. An inexperienced attorney worked for the firm. The firm's partners charged the policyholders fees for the representation even though the insurer was already paying their legal fees; this and other aspects of their fee arrangements violated state insurance laws, as well as the ethical rules about reasonable fees. The inexperienced acted as the partners directed him to do and charged clients these fees that were illegal and unreasonable, but at one point he raised concerns about the practice with one of the partners. The partner said he would check into it. Would the safe harbor provision of Model Rule 5.2(b) absolve the attorney of a duty to research the fee issue?

c) No, the attorney had a duty to research the issue himself and would have discovered that the fees were clearly illegal and unreasonable.

333. For purposes of attorney work product protection, which of the following is NOT likely to create an objectively and subjectively reasonable "anticipation" of litigation:

d) A client who has a history of being extraordinarily litigious

485. An attorney had included false statements on his application for admission to the bar, but the lies went undetected, so the attorney obtained his license and began to practice law. The nonlawyer employees he hired to work at his firm were aware that he had lied on his bar application, but they did not report this to the state disciplinary authorities. The firm had no legal malpractice insurance, but the attorney neglected to disclose this to some of the firm's prospective clients. At one point, the attorney represented a seller in a business transaction involving industrial equipment. To complete the transaction, the purchaser sent the attorney a check for the agreed-upon purchase price, with a letter directing the attorney to forward the money to the seller, whom the attorney represented in the matter. The attorney notified his client immediately that the money had come in. The client was traveling at the time and asked the attorney to hold the funds until he returned from his trip. The attorney had only recently launched the firm and did not yet have a client trust account at any banks in the area, so he deposited the check in his own bank account temporarily. As soon as the check cleared, the attorney wrote a check to the client for the full amount, which the client picked up in person. Did the attorney act properly regarding the funds?

c) No, the attorney had an obligation to hold the funds in a separate account from the attorney's own property.

317. A client consults an attorney about the client's indictment for the crimes of theft and unlawful possession of stolen goods. Applicable law treats possession of stolen goods as a continuing offense. The client is still hiding the stolen items in a secret place, and the client asks the attorney about in which client can continue to hold onto the stolen goods. During the conversation, the client describes the present location of the stolen items. The prosecutor then tries to subpoena the attorney to testify about the location of the stolen goods. Would attorney-client privilege apply to the conversation, if the client's crime is still ongoing?

c) No, the crime-fraud exception defeats attorney-client privilege for this conversation, as the crime is still ongoing.

464. A litigation attorney worked for several years for the Office of the Attorney General in his state, but then left to work for Big Firm. At Big Firm, the attorney exclusively handled litigation for Conglomerate Corporation, one of Big Firm's most important clients. Conglomerate Corporation had no litigation with the state government, so Big Firm made no effort to screen the attorney from any cases, though it would conduct customary conflict checks. In one case, the attorney was defending Conglomerate Corporation in a personal injury lawsuit over an accident with one of its delivery truck drivers. The parties agreed to use caucused mediation. In caucused mediation, the mediator meets privately with the parties and their counsel. These meetings or caucuses are confidential, and the mediator controls the flow of information among the parties and their counsel, as agreed by the parties. The attorney customarily starts negotiations or regular mediations by downplaying Conglomerate's willingness to compromise. In the alternative, the attorney might overstate, or sometimes strategically understate, the strengths or weaknesses of Conglomerate's litigation position. Are such statements, which might otherwise be permissible in regular mediation or direct negotiations, improper during a caucused mediation?

c) No, the same standards that apply to lawyers engaged in negotiations also apply to them in the context of caucused mediation, because parties cannot waive, even by mutual consent, the protections against false statements of material fact during negotiations.

323. A defendant accused of bank robbery hired an attorney. The attorney interviewed a bank teller, who witnessed the robbery. The attorney memorialized the conversation in a written memorandum that qualified as work product. Later, during the trial, the same teller testified for the prosecution, and the attorney cross-examined the bank teller by quoting from the teller's prior statement, as memorialized in the memorandum. The bank teller then denied making the statements. In turn, the prosecutor demanded a copy of the document from which the attorney had read statements during the cross-examination, and the attorney objected that the document was attorney work product and therefore not subject to discovery. Is the attorney correct?

c) No, when the attorney chose to ask the teller questions with direct reference to the memorandum, it waived work-product immunity for the portion of the memorandum discussing the teller's story, and any other parts of the document that are necessary to place all the testimony fairly into context.

455. A certain client applied for a bank loan from Big Bank based on a security interest in farm land and farm machinery. Big Bank required an opinion letter at the time of closing from the client's attorney, vouching for the deed of trust executed by the client that would give Big Bank a mortgage lien on the property, prior to any other recorded liens. The client's attorney provided the opinion letter. It states that the attorney has neither physically inspected the property nor investigated the state of the record title with respect to the mortgaged property, relying instead on the preliminary title report of a title-insurance company that there are no other liens on the property and that the client has clear title to the property. Unknown to the attorney, a third party had already acquired adverse possession rights in the property. The third party has also incurred unpaid bills that resulted in mechanics' liens on the property. All this occurred after the date of the preliminary title report. Which of the following is correct, based on these facts?

c) The attorney did not violate a duty of care to Big Bank by relying as stated in the opinion letter solely on the preliminary title report and not conducting any other investigation.

487. A natural disaster struck a certain attorney's city and destroyed his office, including many documents of intrinsic value belonging to clients. Which of the following would be one of the attorney's ethical duties as a result? a) The attorney must self-report the loss to the state disciplinary authority and accept whatever sanction it imposes. b) The attorney must compensate the clients for the documents, including the return of a portion of the legal fees that the attorney received from the client. c) The attorney must make reasonable efforts to reconstruct documents of intrinsic value for both current and former clients, or to obtain copies of the documents that come from an external source. d) The attorney must promptly notify the opposing party in each clients' matter about the loss of important documents that might be relevant or material to the other party.

c) The attorney must make reasonable efforts to reconstruct documents of intrinsic value for both current and former clients, or to obtain copies of the documents that come from an external source.

288. An attorney represented a client who was a potential defendant in a personal injury lawsuit. The victim of the accident has threatened the client with litigation unless the client can convince the victim's lawyers that the client is not at fault. The victim also gives a deadline for producing such evidence, after which litigation will proceed. The client authorized the attorney produce a large batch of documents. The attorney reviewed the files before sending, but she overlooked one confidential memorandum by the client to the attorney that was in the batch of documents produced. This oversight occurred even though the attorney conducted a more thorough pre-production review than most lawyers would do - the attorney was not negligent, but the mistake still happened. As soon as the attorney discovered her mistake, she reasserted privilege on behalf of the client for that document. The victim's lawyer claims that the attorney waived privilege by disclosing it, even inadvertently. Which side is correct?

c) The attorney who made the inadvertent disclosure, without negligence, can properly reassert privilege.

290. A small independent soda company had a delivery truck that collided with a school bus full of children on a field trip. The company's owner and the driver, who were co-defendants in the first lawsuit over the incident, met with their litigation attorney - the owner agreed to pay the fees for representing them both. As they were discussing the accident, the attorney called in his own accident scene investigator to join the discussion, and the investigator took notes. As the litigation progressed, the driver eventually filed a cross-claim against the owner for indemnification if the driver has to pay damages to the plaintiff. At that point, the plaintiff sought to depose the attorney's accident investigator to discover what admissions the co-defendants made in the previous conversation. The owner objected. How is the court likely to rule?

c) The conversation comes under the protection of attorney client privilege because at the time it occurred, the driver and owner were both clients and the investigator was there to assist the attorney.

293. A client consulted with his attorney privately about how to wire funds to an offshore bank account legally, in a manner that would not violate tax laws or draw the attention of federal regulators. The attorney was not aware at the time that his client was engaged in illegal activity, and thought he merely wanted a secure investment. Later, however, the client became the target of a federal prosecution on corruption charges. The prosecution subpoenaed the attorney to answer questions about the conversation with the client regarding wire transfers to offshore accounts. The attorney objected that this was a privileged communication between the client and the attorney. How is the court likely to rule?

c) The conversation is not privileged because of the client's illegal purpose in seeking the information.

297. An attorney prepared the policy manuals for a corporate client, an insurance company. The manuals guide the client's claims adjusters about claims reporting procedures, such as assigning counsel, closing files, reporting bad-faith claims, maintaining records, settlement authority, and so forth. These attorney-drafted policies served the purpose of facilitating the rendition of competent claims handling by the insurer's employees. When a litigation opponent requests production of these manuals during pre-trial discovery, would they come under attorney-client privilege?

d) No, because the documents were not part of rendering legal advice, but rather for the employees to use in processing claims, and they were not confidential enough to create privilege

284. An attorney represented a defendant in a personal-injury action. The client made a confidential communication to the attorney concerning the circumstances of the accident. Later, in the judicial proceedings, the attorney was conducting direct examination of the client, and the client testified about the occurrence. She did not, however, make any reference in her testimony to what she told the attorney previously about the same matter. When the plaintiff's lawyer began his cross-examination of the client, he asked whether the defendant's testimony was consistent with the account she previously gave to her attorney in confidence. The defendant's attorney objects that privilege applies to this conversation, but the plaintiff's lawyer asserts that the defendant waived privilege by discussing the same things in her court testimony. Which one is correct?

c) The defendant's attorney is correct that his client did not waive attorney-client privilege by testifying regarding the same facts at trial.

375. An attorney made a lateral move to Small Firm. The managing partner had the attorney sign an employment contract on his first day, which included a provision under which the attorney agreed that upon leaving employment, he would pay his former employer ninety-five percent of any attorney fees earned in a contingent-fee settlement from any Small Firm clients who might follow the attorney when he left. The attorney worked for Small Firm for seven years, then left to start his own practice. Before the attorney left Small Firm, however, he had begun representation of a client who was an accident victim, and the client choose to follow the attorney to his new firm, to continue the representation. The attorney eventually obtained a generous settlement for the client; the attorney's contingent fee was one-third of the award, after deducting fees and expenses. The managing partner immediately notified the attorney that he had a contractual obligation to pay Small Firm ninety-five percent of the fee from the settlement, and notified the defendant's insurer, that it should send its check to Small Firm as the loss payee rather than the attorney's new firm. What is the proper result in this case?

c) The insurer should send the check to the attorney's new firm as loss payee, and the attorney should send no money at all to Small Firm.

353. Three law partners have decided to incorporate their firm instead of continuing as a partnership, as their malpractice insurer has offered them a lower rate on their premiums if they incorporate and thereby reduce some of their joint liability. They also want to make a clearer track for associates to become shareholders after reaching certain performance benchmarks. The articles of incorporation provide that when a shareholder dies, a fiduciary representative of the estate may hold stock in the corporation for a reasonable time during administration of the estate before transferring it to the heirs. Which of the following may the partners properly do as they incorporate?

c) They may provide, as stated, that when a shareholder dies, a fiduciary representative of the estate may hold stock in the corporation for a reasonable time during administration of the estate before cashing out the shares and transferring the funds to the heirs.

354. A church retains an attorney to challenge a new zoning regulation that would prohibit the church from constructing a new, expanded sanctuary on its property, attached to the existing church. The church cannot afford to pay the attorney, and it is seeking only a declaratory judgment (that the regulation is invalid) rather than money damages. The attorney agrees to take the case and then split any court-awarded legal fees with the church if they prevail. They win a favorable judgment; the court declares the regulation unconstitutional and awards legal fees, which the attorney shares with the church. Is the fee sharing proper?

c) Yes, because a lawyer may share court-awarded legal fees with a nonprofit organization that retains the lawyer in a matter.

273. An attorney was the managing partner at a firm. The firm had current, up-to-date network security, firewalls, password protection, anti-virus software, and email encryption. As managing partner, the attorney would revisit this issue every year in January, checking with the relevant vendors to see if there were important software updates or new products that the firm needed. One January, a vendor was installing new software and discovered that the firm had suffered a significant data breach the previous summer that went unnoticed. Hackers had used sophisticated methods to bypass conventional firewalls and other mainstream security features, and they had accessed confidential client information. The vendor explained to the managing partner that there was no reason for such events to go unnoticed, because low-cost products and services were available to monitor for data breaches. Could the firm, or at least the managing partner, be subject to discipline for failing to monitor for any breaches in data security?

c) Yes, lawyers must employ reasonable efforts to monitor the technology and office resources connected to the internet, external data sources, and external vendors providing services relating to date and the use of data.

324. The DOJ brought an antitrust suit against Conglomerate Corporation. Giant Company separately sued Conglomerate, mostly alleging the same facts that the DOJ had alleged in its case, and Giant sought parallel relief. An attorney for Giant Company showed the DOJ lawyers some documents that constituted part of the attorney's work product in Gian Company's parallel lawsuit against Conglomerate. Giant Company and the DOJ formally agreed that the DOJ would use documents only in litigation against Conglomerate Corporation. Later, however, in the government's case, Conglomerate Corporation sought discovery of Giant Company's work product, that is, the documents that Giant's attorney had shared with the DOJ. How should the court rule on this discovery request?

d) Both Giant Company and the DOJ (government) may properly assert Giant's work-product protection for the documents, under the common-interest doctrine.

357. An attorney obtained a license to practice law in the state where she attended law school. After a few years, the attorney took a job in a neighboring state, moved there, and obtained a license to practice law in her new state. She kept her original license, in her former state, but went on inactive status there to avoid the burdensome annual bar membership fees in a state where she no longer practiced. Eventually, her new firm loses its anchor clients and recommends that the attorney drum up some new business among her former clients. Then the attorney sends letters to all her former clients in her former state, offering to represent them in any new legal matters they have, or in updating wills or contracts that she previously did for them. She travels about once per week to her home state and meets with clients in a library study room at the law school she attended. A few of her former clients refer her to friends or relatives who become new clients, and the attorney's new employer is thrilled. Which of the following is true?

d) Both the attorney and her supervising lawyer are subject to discipline because she is on inactive status in her home state but is soliciting clients and handling their matters there regularly.

364. An experienced attorney has an office in State X, and she is duly licensed to practice law in that state. The attorney's office is in a city on the border of State Y, and the attorney does not have a license to practice there. The attorney represents a regulated utility, which operates a power plant in State X near the border with State Y. The attorney's original work for the utility in State X related to rate-setting proceedings before a utility commission in that state, and before the Federal Energy Regulatory Commission (FERC). New legislative changes now permit the utility to make retail sales of electricity to consumers in multiple states. Given the attorney's extensive knowledge of the utility's rate-related financial information, the utility asks the attorney to handle its new rate applications in several other states, but in none of these states does the attorney have a license to practice law. The attorney's work in those matters would frequently require her presence for legal activities in each of the other states until the new rate work is complete. Is it permissible for the attorney to conduct those activities in the other states on behalf of the utility?

d) It is permissible because the legal issues arise out of or relate to the attorney's practice in a jurisdiction in which the lawyer has a license to practice.

358. A client retains his attorney, who has represented the client in the past, to represent him in litigation in another state, where the attorney is unlicensed. The matter requires some knowledge of the law of the state where the trial will occur. His attorney files a pro hac vice appearance in the matter, which the local court accepts, and begins preparing for trial there. The attorney and the client never discuss the particulars of filing a pro hac vice appearance; nor did they discuss why it would be necessary. The client never asked if the attorney could practice law in the other jurisdiction, and the attorney never explained the licensing requirement and that he would need permission from the court there to handle the case. Then the attorney prevailed in the matter on behalf of the client, kept his agreed-upon contingent fee, and gave the client the remaining proceeds and unused retainer funds. Which of the following is true?

d) It was improper for the attorney to fail to disclose to the client that he was unlicensed in the other state and would need to file a pro hac vice appearance, especially given that the matter required some knowledge of local laws.

378. An attorney represented a plaintiff in a wrongful death case arising out of a prison riot, which included many claims and crossclaims. The case ended in settlement. The defendant's settlement offer included two conditions: first, the commonplace requirement that the attorney and client not disclose the amount of the settlement; and second, that the attorney give defendant counsel her entire file to keep under seal, meaning the attorney could not keep copies of her own work product in the case. She would have to turn over her own personal notes and internal memoranda in the file from her interns and associates. Would it be proper for the attorney to agree to this as a condition of a large monetary settlement for her client?

d) No because forfeiting the attorney's own work product in the case could restrict her future practice of law in similar cases.

465. An attorney was representing Conglomerate Corporation, a large employer, in labor negotiations with the employee's union. The union had demanded, among other things, better coverage for birth control and abortions under the employee health insurance plan. The attorney told the union's lawyers that adding this benefit would cost the company an additional $142.37 per employee per quarter, but the attorney knew that it would in fact cost only $30 per employee. The attorney had learned over the years that using overly specific numbers instead of round numbers was a more effective strategy for bluffing. Under the Model Rules, was it permissible for the attorney to make this false statement to a third party?

d) No, a lawyer shall not knowingly make a false statement of material fact to a third person.

467. An attorney had struggled all through law school with the sheer amount of reading and memorization, and then she had struggled to establish a successful law practice because everything took so much time and acumen. She advertised heavily as a personal injury plaintiff's lawyer, and she attracted new clients through deal-of-the-day promotions that she ran through a service called PleaseTryThis. Her advertising was so prevalent that when police stopped her vehicle for speeding violations, the officers would immediately recognize her as the lawyer from the advertisements, and sometimes this helped her avoid receiving a ticket. One personal injury client presented an unusually complicated problem. After filing pleadings in the case, while the proceedings were still in the discovery phase, the client died in a car accident, unrelated to the previous injuries that were the basis of the lawsuit. During subsequent settlement negotiations with a corporate defendant, Giant Company, the attorney did not disclose that her client had already died, but continued negotiations as if the client was still alive and had authorized her to accept or reject certain offers. Was it permissible for the attorney to delay disclosure of the client's death during the initial stages of settlement negotiations?

d) No, a lawyer engaged in settlement negotiations of a pending personal injury lawsuit in which the client was the plaintiff cannot conceal the client's death; she must promptly notify opposing counsel and the court of that material fact.

397. An attorney was running for judicial office. On her campaign website, she referred to herself as "Madame Justice," and depicted herself in traditional judicial robes, even though she had never held judicial office before. The statement and photo were impermissible under the state judicial code, but she was not yet a judge, and it did not violate the regular attorney advertising rules, as she was not soliciting or appealing to potential clients for her legal practice through the campaign website. Was it permissible for the attorney to include these statements and photos on her campaign website while running for judicial office?

d) No, a lawyer who is a candidate for judicial office shall comply with the applicable provisions of the Code of Judicial Conduct.

402. An attorney was running for a judicial office, a seat on the county court. She drafted, signed, and mailed a fundraising letter in her own name to local voters announcing her candidacy and asking for campaign contributions. The fundraising letter was typical, would normally have been legal if the attorney were running for the legislature or an executive branch office. The state's code of judicial conduct, however, forbid judges from engaging in direct fundraising. The state bar disciplinary authority brought a grievance against the attorney for violating the judicial code. The attorney objected that she was not yet a judge, but was merely seeking judicial office, and the code itself purports only to regulate the conduct of judges. In other words, she contends the judicial code does not apply to lawyers. Is the attorney correct?

d) No, a lawyer who is a candidate for judicial office shall comply with the applicable provisions of the Code of Judicial Conduct.

492. Attorney Stevenson, from Tiny Firm, brings in a lawyer from Giant Firm to work on a complex litigation matter, and they agree to share fees. The client receives a single billing covering the fees of both lawyers, even though they work for separate firms. To comply with the Rules of Professional Conduct, the fee division between Attorney Stevenson and the other lawyer is proportionate to the services performed by each lawyer, and the client agreed in writing beforehand to the arrangement, including the share each lawyer would receive. The total fee is reasonable. At the end of the representation, Attorney Stevenson receives the earned fee from the client, including the share that he owes to the other lawyer at Giant Firm. He promptly notifies the other lawyer. Attorney Stevenson deposits the total sum in his firm bank account (its operating account, not a client trust account), and after confirming that the funds are available from the bank, he sends a check to the other lawyer with his share of the fees. Are Attorney Stevenson's actions proper, as described here?

d) No, because Attorney Stevenson should have deposited the other lawyer's share of the fees in a trust account, separate from his own funds.

500. An experienced litigator became a judge. In her previous litigation practice, she would regularly search online to learn more about the opposing party, opposing counsel, and even jurors. She sometimes found useful information on opposing parties' websites or on social media. Now serving as a judge, she visits the website of the corporate defendant in one of her cases, to learn more background about the company and its products and pricing. In this instance, the judge did not find any information on the company's website that seemed useful in understanding the issues in the pending case. The judge did not do any online research about the other party (the plaintiff) in the case. Was the judge's research proper?

d) No, because the code of Judicial Conduct prohibits judges from conducting online research to gather information about a party or juror in a pending case, even if the research yields no useful information.

385. An attorney works for a firm that handles residential real estate closings. The firm also provides title insurance, as part of the legal representation it offers to clients, but for an additional fee. Nonlawyers also provide title insurance in that state, for comparable prices. A prospective client met with the attorney for an initial consultation about their anticipated purchase of a home. Another client of the attorney's firm had referred the prospective client to the attorney. When the attorney mentioned that the firm would also provide title insurance for an additional fee, the prospective client asked if the person who had referred her to the attorney had obtained title insurance through the firm, and how much they had paid for it. Would it be permissible for the attorney to share this information with the prospective client without first obtaining the other client's consent?

d) No, because a lawyer is subject to the duty of confidentiality, as well as the other ethical rules, with respect to the provision of law-related services, that are not distinct from the lawyer's provision of legal services to clients.

265. A client hired an attorney to represent her in a burglary charge. During a meeting with the attorney and with the understanding that any information would be confidential, the client advised the attorney about a murder she committed. A wrongfully accused man was presently on trial for that same murder. Eventually, the attorney was able to negotiate a plea deal for the client on her burglary charge. They finalized the plea deal and the attorney's representation ended. Soon thereafter, the attorney discovered that a jury had convicted an innocent man for the murder the client had committed and confessed to the attorney. The wrongfully convicted men received a life sentence, without the possibility of parole. The attorney contacted the District Attorney's office that handled the murder trial and left an anonymous tip stating that the client confessed to committing the murder. Was the attorney's conduct proper?

d) No, because attorneys cannot disclose client representation information and the death had already occurred, therefore, the disclosure would not prevent certain death or substantial bodily injury.

476. An attorney represents a plaintiff in a civil suit. The defendant also has representation, but he contacts the attorney to negotiate a settlement agreement. The attorney advises the defendant that he cannot discuss the case with the defendant because the defendant has representation by counsel. Defendant faxes the attorney a letter stating that he waives the rule restricting the attorney from communicating with the defendant while the defendant has representation. Upon receipt of the fax, the attorney contacts the defendant and discusses a settlement agreement. Are the attorney's actions proper?

d) No, because attorneys may not communicate with represented persons unless the attorney representing that person permits the attorney to communicate with the represented person.

389. An attorney faced a grievance over a client complaint regarding his neglect of the client's matter. The attorney knew that he had never formally agreed to represent the client, but instead had met with the client once, determined that he had a conflict of interest, and he had refused to represent the potential client by both oral and written communication. The client failed to hire another lawyer, and mistakenly (unreasonably) believed that the attorney she had met with was, in fact, representing her. Because he knew the case was without merit, he did not respond to the state bar when the disciplinary authorities requested a formal response from him. In the end, the client withdrew her complaint and the disciplinary authorities dismissed the grievance as frivolous. The board then commenced disciplinary proceedings against the attorney for failing to respond to its requests in the case it had dismissed. Was the attorney's refusal to respond permissible in this case?

d) No, because in connection with a disciplinary matter, a lawyer must not knowingly fail to respond to a lawful demand for information from an admissions or disciplinary authority.

440. An attorney describes his areas of practice in his advertisements as "real estate" and "personal injury," but his state bar requires that lawyers use the less descriptive terms "property law" and "tort law" instead. Could the attorney be subject to discipline for using these more descriptive terms instead of the verbiage prescribed by the state bar?

d) No, because lawyers have a First Amendment right to use verbiage that is accurate and descriptive in their advertisements, assuming the statements are not misleading.

306. A client had a confidential conversation with his attorney seeking legal advice. The client died a few weeks later. The client had pending litigation at the time of his conversation with the attorney, and the opposing party seeks disclosure of the conversation, because opposing counsel believes the client had instructed the attorney to accept the opposing party's settlement offer, up to a certain amount. The attorney is continuing the claim on behalf of the client's estate, and he refuses to settle or to disclose the contents of the conversation. Should the court compel the attorney to reveal whether the client wanted to settle the case before he died?

d) No, because privilege normally survives the death of the client.

300. Conglomerate Corporation has several offices around the state. After receiving a few employee complaints about workplace discrimination from one office, Conglomerate's corporate officers asked the company's attorney to advise them about potential liability in the matter. The attorney conducted a careful investigation and wrote a thorough memorandum summarizing her findings and legal conclusions. Because the matter involved a commonplace scenario, the attorney thought it would be helpful to give all the company's human resources managers, in each of its offices statewide, guidance about the issue, so she sent the memorandum to all sixty-two HR managers in Conglomerate's offices nationwide. When litigation eventually ensued over the alleged discrimination, the plaintiffs sought discovery of the attorney's memorandum, but Conglomerate attorney asserted attorney-client privilege. Is Conglomerate's position correct?

d) No, because sending the memorandum to so many employees who had no connection to the matter waived the privilege.

267. While representing a client, an attorney learned confidential information about the client's previous marriage and divorce, which occurred many years before in another country. Before the attorney could conclude the matter, the client terminated the representation. Over the next three years, the now-former client became a well-known celebrity, and her prior marriage and divorce received widespread public attention in that region. Very recently, the state bar journal interviewed the attorney about his career and his greatest achievements. One question pertained to the representation of the client who became a celebrity. The attorney mentioned that at the time, the client was an unknown figure and her previous marriage were family secrets. The interviewer was not well-informed about this celebrity and was surprised to hear that the individual had been married and divorced in another country. The former client had never authorized the attorney to discuss her legal matters, but the Model Rules provide a "generally known" exception to the duty of confidentiality to former clients. Would that exception apply to the attorney's disclosure of the marriage and divorce during the interview?

d) No, because the "generally known" exception does not apply to disclosures by the attorney about former clients.

445. An attorney spends about one hour per week, on Monday mornings, calling local small business proprietors who routinely hire lawyers for lease and contract issues, and offers over the phone to provide legal services to them for a competitive (that is, low) fee. Does this activity by the attorney violate the Model Rules?

d) No, because the attorney is calling individuals who routinely use for business purposes the type of legal services offered by the lawyer.

325. An attorney had many years of experience in handling personal injury litigation, and in a certain case, the attorney represented a plaintiff in litigation over injuries sustained in a car accident. In preparation for trial, the attorney interviewed each of the eyewitnesses of the accident, and afterward wrote a memorandum summarizing what each witness said. The witnesses themselves agreed to swear and sign the statements, as if they were affidavits. The statements contained no mental impressions of the attorney, only facts communicated by the witnesses. Opposing counsel eventually learned of these interviews and sought discovery of the witness statements that the plaintiff's attorney had drafted. Unsurprisingly, the attorney objected that these documents were attorney work-product doctrine. Should the court compel the production of the witness statements?

d) No, because the attorney prepared the witness statements on behalf of the plaintiff in anticipation of the litigation.

304. The corporate officers of a large hospital were trying to decide whether to provide free HBO and Showtime (and other subscription cable channels) to all the televisions in the patient rooms. Corporate counsel participated in these meetings due to his familiarity with the pricing of these channels and what other hospitals in the area were doing in this regard. Later, the hospital finds itself in contract litigation with its cable provider, and the opposing party requests disclosure of the comments and discussion in this meeting. The hospital's corporate counsel objects that this meeting was privileged communication because of the participation of the attorney in the meeting. Is he correct?

d) No, because the attorney was participating as a business advisor in this meeting, not providing legal services.

448. An attorney made substantial financial contributions to the reelection campaign of an elected judge. The judge won reelection and showed his gratitude to the attorney by frequently appointing him to represent indigent defendants at the state's expense. The attorney made the donations not because he hoped to receive such appointments, but because he honestly believed that the judge was the best candidate for the position. The attorney especially admired the fact that the judge had attended Harvard Law School and that the judge was an active member of the Federalist Society. Could the attorney be subject to discipline for accepting these appointments?

d) No, because the lawyer's motivation was a sincere political or personal support for the judge's candidacy, not a design to receive court appointments.

371. Conglomerate Corporation offered to hire an attorney as outside counsel for a specific legal matter. Conglomerate's OCG (outside counsel agreement) with all outside lawyers it hires includes the following provision: ATTORNEY agrees that it would constitute an impermissible conflict of interest to represent a significant competitor of CONGLOMERATE CORP. or its subsidiaries or affiliates. The APPENDIX attached to this document includes a list of CONGLOMERATE CORP. subsidiaries. Before ATTORNEY'S representation begins, ATTORNEY must disclose in writing the names of any national or regional retailers or any significant competitors of CONGLOMERATE CORP. or its subsidiaries or affiliates that ATTORNEY represents, as well as a general description of the type of representation that ATTORNEY'S firm provides to such client(s). Is it proper for Conglomerate's in-house counsel to require outside counsel to agree to this provision in the OCG?

d) No, because this agreement impermissibly restrains the attorney's right to practice.

299. Howard Hamlin is a partner at the law firm Hamlin, Hamlin, & McGill (HHM). HHM's computer network automatically inserts the firm's "Hamlindigo Blue" logo and letterhead into every email sent from the firm's email accounts, as well as a legal disclaimer at the end of every email that reads, "NOTICE: This email may contain PRIVILEGED and CONFIDENTIAL information and is only for the use of the specific individual(s) to which it is addressed. If you are not an intended recipient, you must not review, copy, or show the message and any attachments to anyone. Please reply to this e-mail and highlight the mistaken transmission to the sender, and then immediately delete the message." Attorney Hamlin believes that every email sent by anyone at the firm to anyone outside the firm should be undiscoverable, under the doctrine of attorney-client privilege, because each email automatically includes this disclaimer under the sender's signature line. Is Hamlin correct?

d) No, blanket privilege inscriptions on law firm correspondence do not guarantee that privilege will apply to the contents of the email, because emails sent to non-clients (or copying non-clients as additional recipients) would not be privileged.

472. In anticipation of a trial over workplace discrimination, a plaintiff's attorney contacts several current managers of the defendant corporation and interviews them about the day-to-day operations of the company and the chain of command for addressing personnel complaints. These managers supervise employees, address interpersonal problems between workers, filed complaints, and consult with the firm's in-house counsel about personnel matters that seem serious. The attorney does this without permission from the defendant's attorney. Was this proper?

d) No, consent of the company's attorney is always necessary for communication with a present constituent of the organization who supervises, directs, or regularly consults with the organization's attorney concerning the matter.

292. Conglomerate Corporation hired outside counsel to represent the organization in a lawsuit, but part way through the representation, Conglomerate's managers decided to fire the attorney and hire someone else with more experience. Conglomerate's former attorney then sued the organization for her unpaid legal fees for the representation up to that point. Conglomerate's new lawyer subpoenaed the attorney's time sheets for the billable hours he claimed to have worked for Conglomerate, because the organization believed the attorney was overbilling. The attorney claimed that the time sheets came under attorney-client privilege and refused to disclose them. Is the attorney correct?

d) No, documents and information about billable hours, scheduling, and so forth are not privileged.

407. A law professor has a tenured faculty position at her institution. She learns of serious ethical misconduct by another law professor on her faculty who is a licensed lawyer in that state, but who engages exclusively in law teaching. The professor who learned of the problem believes she has no duty to report her colleague to the bar, as neither of them are practicing law, though both have law licenses. Is she correct?

d) No, if a lawyer knows of professional misconduct of another licensed lawyer, even a non-practicing lawyer, must report it where it raises a substantial question as to that lawyer's honesty, trustworthiness, or fitness as a lawyer.

483. An attorney represented a powerful but controversial politician. A prosecutor was seeking an indictment of the attorney's client, so the attorney located a young woman who volunteered to befriend the prosecutor at a social event and exchange phone numbers. Then, at the attorney's request, the young woman would call the prosecutor and engage in lurid sexual conversations over the phone, while the attorney was recording the conversations. The prosecutor was unaware that the attorney and the young woman were recording the conversations, but the laws of that state required the knowledge and consent of only one participant to record a conversation. The attorney then sent the prosecutor a copy of the recordings of the phone conversations, which were very embarrassing to the prosecutor. The attorney did not include any communication with the recordings, such as threats or extortionary demands. Nevertheless, the prosecutor did not want the recording to become public, so he stopped pursuing the indictment of the attorney's client. Was it permissible for the attorney to record these phone conversations, under these circumstances?

d) No, if a lawyer records a conversation with no substantial purpose other than to embarrass or burden a third person, the lawyer has violated Rule 4.4.

289. A soda company had a delivery truck that collided with a school bus full of children on a field trip. The soda company's distribution manager wrote a report of the accident and provided it to the company's litigation counsel. The manager did not share the report with anyone except the attorney. When lawsuits from the injured children begin against the company, one of the plaintiffs requests the distribution manager's report. Will a court order the attorney or the company to produce the report during discovery?

d) No, it is privileged communication from a client to a lawyer.

470. An attorney surreptitiously recorded a conversation with a potential witness without the other person's knowledge or consent. The potential witness asked the attorney at the beginning of the conversation if the attorney was recording it, and the attorney assured her that he was not, even though he was in fact recording it. State law permits recording of conversations when at least one of the participants consents, which would include the attorney in this case. The potential witness learned about the recording later and was upset, because she would not have consented to the recording of the conversation, or at least would have been more judicious about her comments. Assuming the attorney did not violate state or federal laws by recording this conversation, were the attorney's actions proper, given these facts?

d) No, lawyer who records a conversation without the consent of a party to that conversation may not represent that he is not recording the conversation.

415. Big Firm engages in aggressive affirmative action in its hiring. It runs ads soliciting applications from minorities and women, and even though they sometimes interview non-minority applicants, they have decided internally to hire only women and minorities for the next five years. Currently, anti-discrimination laws would not require such a practice. Has the firm violated the MRPC?

d) No, lawyers may implement initiatives aimed at recruiting, hiring, retaining, and advancing diverse employees without violating the Model Rules.

379. An attorney represents a large corporate defendant in a tort action over a defective product line. The current action is the first of what may be many such lawsuits, but the problems with its product line have not received any media attention yet, so the company decides to settle the matter quietly. Recognizing that he has a duty to protect the legal interests of his client, the attorney asks for three conditions in the settlement. First, the plaintiff agrees to a waiver and release of this and any other claims arising out of the use of this product, at least up to that time. Second, the plaintiff and the attorney must agree not to disclose the settlement amount to anyone. Third, the plaintiff's lawyer must agree not to use any information learned in the current representation in any future representation against the corporate defendant, whether in litigation or transactional matters. The attorney recognizes that there can be no restrictions placed on the lawyers right to practice law, so he does not ask the lawyer to refrain from representing other plaintiffs against the corporation, but only that the information from this case not carry over into other unrelated cases. The attorney also points out to opposing counsel that the conflict of interest rules would already prohibit the attorney from using any information learned in a representation against the client. Similarly, the confidentiality rule forbids the disclosure (without the client's consent) of confidential information learned from any source during the representation. Thus, the condition in the settlement overlaps with other disclosure restraints that the Model Rules impose on the other lawyer. Opposing counsel is a notorious plaintiff's lawyer in that region, receiving frequent reprimanded for ethical violations from the state bar. The lawyer has a reputation for bringing up irrelevant but inflammatory evidence from other cases in his trials, telling the jury, "You wouldn't believe what this same company did to my other client!" It seemed appropriate, therefore, to the attorney for this defendant to ask for settlement conditions that recognize this lawyer's previous bad behavior. Is the attorney correct?

d) No, prohibiting the lawyer from using any information learned in the representation is an impermissible restriction on the lawyer's right to practice.

480. An inexperienced attorney represented an insurance company in a wrongful-death lawsuit. The plaintiff was the widow of the deceased, acting as the personal representative of the deceased. The widow was a pro se litigant, as she did not have legal representation in the matter. The lawsuit was over the death of her husband. The pro se widow asserted that one of the insurance company's policyholders had negligently caused the husband's death. State law required that settlements of a wrongful death claim by a personal representative must have approval from a tribunal. The widow and the insurance company's claims manager eventually agreed on a settlement amount. The attorney, representing the insurance company, prepared the necessary documents and presented them to the widow for her signature. The widow, knowing that the attorney represented the interests of the insurance company, asked the attorney why the documents were necessary. The attorney responded that to effectuate the settlement, they needed to execute the documents and file them for court approval. This was true. Was the attorney's conduct improper, under the Model Rules?

d) No, so long as the unrepresented person understands that the lawyer represents an adverse party and is not representing the person, the lawyer may prepare documents that require the person's signature and explain the lawyer's own view of the meaning of the document or the underlying legal obligations.

373. An attorney worked as in-house counsel at Conglomerate Corporation. Conglomerate had a problem with lawyers who left its legal department to work for its suppliers - the lawyers would contact their friends who still worked for Conglomerate to solicit additional supply contracts, or to negotiate more favorable terms on existing contracts. Worse, the lawyers could also make strategic use of their knowledge of Conglomerate's internal procurement practices (such as the time of year when certain major supplier contracts were up for renewal). General Counsel for Conglomerate started including in its contracts with all new in-house counsel a prohibition on departing lawyers who work for Conglomerate's corporate vendors, either as in-house counsel or with a law firm representing the vendor, from contacting any of Conglomerate's employees. Is this agreement proper, under the Model Rules?

d) No, the agreement imposes an impermissible restriction on lawyers' ability to practice law.

314. An attorney represented Conglomerate Corporation. An officer of Conglomerate Corporation communicated in confidence with the attorney about deals between Conglomerate and one of its creditors, Big Bank. Conglomerate later declared bankruptcy, and the court appointed a in bankruptcy for Conglomerate. Then the attorney became a necessary witness in the litigation between Big Bank and Conglomerate's bankruptcy trustee. Conglomerate's trustee in bankruptcy waived privilege on behalf of Conglomerate with respect to testimony by the attorney regarding statements by the officer to the attorney. The officer, knowing that the statements would embarrass or even incriminate him, tried to prevent the attorney from testifying, claiming the conversation was a privileged communication to the corporation's attorney. Big Bank's lawyer responded that former officers and directors of a corporation cannot claim privilege after control of the corporation has passed to a bankruptcy trustee. Should the court side with the officer in this situation?

d) No, the officer cannot assert privilege because he was not a client of the attorney in the representation.

287. An attorney represented a client in a license-revocation hearing before an administrative law judge. At one point, the government lawyer asked the client a question about a confidential communication with the client's attorney, and the attorney objected that the conversation clearly came under attorney-client privilege. The administrative law judge overruled the attorney and ordered the client to answer the question, and the client testified about the prior communications with his attorney. On appeal, the attorney claims that the ALJ wrongly overruled his objection and that privilege should in fact apply. The tribunal questioned whether privileged could reattach to a communication after its disclosure, even if the disclosure was the result of an incorrect ruling by a lower tribunal. In subsequent unrelated litigation with another party, opposing counsel seeks to introduce the client's testimony at the administrative hearing that disclosed the information, and the attorney against objects that the original communications were privileged, that he objected to the disclosure at the time, and that the administrative law judge and wrongly overruled his objection. What is the result?

d) The appellate tribunal is incorrect that privilege cannot reattach even if the disclosure was in response to an incorrect ruling by another tribunal, and the disclosure did not waive privilege for subsequent litigation.

270. An employee of Conglomerate Corporation retained an attorney to advise her about a potential claim against her employer. Like most corporate employees, this client has a cubicle workstation with a computer assigned for her exclusive use at work. Conglomerate Corporation's written internal policy states that the company has a right of access to all employees' computers and e-mail files, including those relating to employees' personal matters. Nevertheless, all the employees sometimes use of their computers for personal matters, and most send some personal e-mail messages, whether from their personal or office e-mail accounts. The attorney expects that the employee will sometimes use her computer at work to communicate with the lawyer. Does the attorney have an affirmative ethical duty to warn the employee about the risks this practice entails?

d) The attorney has an ethical obligation to warn the client not to communicate about the matter via her work computer through any email account, and a duty to refrain from emailing the client's workplace email account or responding to emails from the client's workplace email account.

327. An attorney represented a client in litigation. During the discovery phase of the matter, the opposing party sought to discover communications from a meeting that the attorney had previously organized to prepare for the case. The attorney, an accountant, certain interested creditors, and the bankruptcy liquidation committee members had all attended the meeting, as well as a few others. The attorney resisted discovery based on the work product doctrine. The opposing party countered that the presence of other parties besides the attorney, the client, and their necessary agents waived the privilege. How should the court rule?

d) The court should deny discovery because the work product doctrine protects the information from disclosure.

453. An attorney represented a client who had an explosive temper. The representation concerned multimillion-dollar litigation, and the attorney received notice that the judge in the case had refused to qualify the attorney's expert witness to testify at trial. Without the expert, the client's case was unlikely to prevail. Faced with the daunting prospect of delivering this unwelcome news to the client, the attorney emailed the client and explained the setback in highly technical terms, citing the relevant sections of the Federal Rules of Civil Procedure, local court rules, precedential cases, and the Code of Judicial Conduct. He also used archaic legal terms in several places. A nonlawyer would have been unlikely to understand the conclusion - that the disqualification of the expert meant the client would lose the case and should withdraw or settle immediately. Based on the Model Rules, which of the following is true?

d) The attorney violated his ethical duty to the client by providing purely technical legal advice that would be unhelpful to a nonlawyer.

355. An attorney is a licensed lawyer in a New England state, but has an office and represents clients exclusively in a southern state. The attorney confines her practice to immigration law, representing foreign-born clients in immigration hearings. A relevant federal statute permits nonlawyers to appear as representatives for immigrants when they appear before the immigration agency. Many of the attorney's clients have applied for a spousal visa after marrying an American citizen, and some clients had a Notary Public from their home country or an un-ordained lay minister from their home church conduct their wedding ceremony. In addition, some were previously married and divorced in their home country, where such transactions are informal and have no official documentation. There is often some question about whether the marriage is valid under local state law, which is a prerequisite for obtaining certain types of visas. Which of the following is correct?

d) The attorney's conduct is improper if she does not file a pro hac vice appearance in each case.

313. An attorney represented Conglomerate Corporation, and she made a confidential report to Conglomerate's CEO, describing Conglomerate's contractual relationship with Supplier Systems, a large vendor. The attorney advised the CEO that Conglomerate could terminate its contract with Supplier without facing any liability. The CEO then sent a confidential memorandum to Conglomerate's purchasing manager, explaining the parts of the attorney's advice necessary for understanding the issue at hand, and asking whether termination of the contract would nonetheless be inappropriate for business reasons. Months later, Conglomerate finds itself in litigation over a related matter, and the opposing party seeks discovery of what the attorney reported to the Conglomerate CEO regarding Supplier's contract. Conglomerate asserts attorney-client privilege for the report and its contents, but opposing counsel responds that Conglomerate waived privilege by sharing crucial aspects of the report with the purchasing manager, while asking for a business judgment. How is the court likely to rule?

d) The attorney's report to the CEO would remain privileged notwithstanding that CEO shared it with the purchasing manager.

331. An attorney had a series of private meetings with a client about the subject matter of the representation. The attorney kept careful notes of these discussions. Sometime later, an opposing party in litigation moved to compel production of these notes. Which of the following is most likely to be a reason that the attorney would try claiming that they are attorney work product, rather than asserting attorney-client privilege for these notes? a) The representation pertained to anticipated litigation that seemed immediate at the time. b) The notes are written documents rather than the attorney's mental recollections of the meetings. c) The need and hardship exception. d) The client's friend had been present during the conversations.

d) The client's friend had been present during the conversations.

319. A federally recognized tribe of Native Americans, the Jicarilla Apache Nation, brought an action against the Department of the Interior for mismanagement of tribal trust funds, in violation of federal statutes. During discovery, the plaintiffs requested production of certain government documents, but the government had a plausible claim that the documents in question came under the protection of attorney-client privilege. The plaintiffs countered that the fiduciary exception to privilege applied in this case because of the trust relationship between the United States government and the Native American tribes. How should the court rule?

d) The fiduciary exception to the attorney-client privilege does not apply to the general trust relationship between the United States and the Indian tribes.

330. An attorney had a series of private meetings with a client about the subject matter of the representation. The attorney kept careful notes of these discussions, along with the attorney's reflections and concerns. Sometime later, an opposing party in litigation moved to compel production of these notes. Which of the following is most likely to be a reason that the attorney would assert attorney-client privilege for these notes, rather than claim they are attorney work product? a) The client's friend had been present during the conversations. b) The representation pertained to anticipated litigation that seemed immediate at the time. c) The client had recounted the conversations to a friend immediately afterward. d) The need and hardship exception.

d) The need and hardship exception.

285. An attorney represented a defendant in a personal-injury action. The client made a confidential communication to the attorney concerning the circumstances of the accident. Later, in the judicial proceedings, the attorney was conducting direct examination of the client, and the client testified about the occurrence. When the plaintiff's lawyer began his cross-examination of the client, he asked whether the defendant's testimony was consistent with the account she previously gave to her attorney in confidence. The defendant declared, "I have testified exactly as I told attorney two days after this awful accident occurred. I explained to my attorney then that the skid marks made by the plaintiff's car were 200 feet long, and I have said the same things here." The plaintiff's attorney then proceeds to ask questions about the discussions with her attorney, and the defendant's attorney objected that privilege applies to this conversation. The plaintiff's lawyer insisted that the defendant waived privilege by discussing the same things in her court testimony. Which one is correct?

d) The plaintiff's lawyer is correct that the defendant opened the door referencing the previous privileged conversations at trial, thereby waiving privilege for the prior conversations.

332. An attorney had a series of private meetings with a client about the subject matter of the representation. The attorney kept careful notes of these discussions, along with some of the attorney's reflections and ideas. Sometime later, an opposing party in litigation moved to compel production of these notes. Which of the following is most likely to be a reason that the attorney would assert attorney-client privilege for these notes, rather than claim they are attorney work product? a) The client's friend had been present during the conversations. b) The client had recounted the conversations to a group of friends immediately afterward. c) The attorney's firm had an unexpected data breach, despite the firm's updated firewalls and password protection, and the breach allowed an unknown hacker to access the notes before the litigation began. d) The representation pertained to an employee manual that the attorney was drafting for the client's business.

d) The representation pertained to an employee manual that the attorney was drafting for the client's business.

494. A judicial clerk researched alternative methods of toxic mold remediation in homes having mold problems and wrote a memo about her findings for the judge to consider in a case. The defendant alleged, among other things, that the plaintiff in a toxic mold case before that judge had failed to mitigate damages. Has the judge violated the CJC?

d) Yes, this violates Rule 2.9(c) of the Code of Judicial Conduct, because the clerk is conducting the research on behalf of the judge.

261. An attorney represents a client who went through a divorce several years ago in another state, and the divorce resulted in a court order for child support and spousal maintenance. The client then moved to the attorney's state, started a new career in politics, and formed new relationships. She has kept her previous marriage a divorce a secret, except from her closest friend and her attorney, because she is afraid it will affect her new career and public image. Recently, she hired her attorney to handle various legal matters for her, which included issuing a press release about her withdrawal from a political campaign. When news media outlets posted online about the client's withdrawal from the race, the attorney responded to some of the comments that readers posted, to clear up some misunderstandings. In one of the attorney's responses, he mentioned the client's previous marriage and divorce. Did the attorney violate the duty of confidentiality?

d) Yes, because even disclosures of information contained in the public record must have client authorization.

359. An attorney is a joint owner of a collection agency. Whenever the agency's initial efforts to collect prove unsuccessful, the staff at the agency sends the delinquent debtor a demand letter on the attorney's law firm letterhead, threatening to commence litigation if the matter does not reach a resolution within 30 days. The attorney authorized the staff at the agency to send these demand letters, but the attorney is too busy to review all the letters himself. The collection agency staff signs the letters on behalf of the attorney's firm. Will the attorney be subject to discipline for authorizing these letters?

d) Yes, because the attorney is merely facilitating the collection agency in the unauthorized practice of law.

409. An attorney discovers that another lawyer has been stealing clients' funds, but he cannot prove it, as he learned about it from another party who was involved and who has since disappeared. He has some evidence, but not enough to prove that the other lawyer stole the clients' funds. When he confronted the other lawyer, the other lawyer admitted it privately but said he would deny it if there was any attempt to expose the matter. Does the attorney who knows about the violation, but was unlikely to be able to prove it, have a duty to report the violation to the state disciplinary authority?

d) Yes, because the duty to report misconduct depends upon the seriousness of the potential offense and not the quantum of evidence of which the lawyer is aware.

366. An attorney agrees to join a new firm as one of its shareholders, and to merge his practice with that of the new firm. The shareholder agreement includes a provision that if the attorney retires from the firm and begins collecting the firm's retirement benefits, he cannot practice law with another firm, government entity, or as a solo practitioner. Otherwise, the agreement stipulates, the attorney will forfeit the retirement benefits. The firm is concerned that the attorney will want to represent clients occasionally in his retirement, and that he may steal some clients from the firm. Is this agreement proper?

d) Yes, because the rule against restrictions on the right to practice have an exception for agreements concerning benefits upon retirement.

411. A prosecutor was bringing charges against a defendant charged with serious domestic violence. When he met the defendant's victim-girlfriend at the courthouse, she volunteered personal information to the prosecutor in addition to recounting the details of the incident - she explained that she had now had no boyfriend, that she was a struggling single mother, and that she had moved back in with her own parents. The prosecutor and the victim exchanged phone numbers, and he subsequently sent the victim several text messages, the first saying he wished the victim was not a "client" of his office, because "she would be a cool person to know." The next day, he texted her asking, "Are you the kind of girl that likes secret contact with an older married elected DA ... the riskier the better? Or do you want to stop right now before we have issues?" Two days later, he texted again, telling her that she was "pretty" and "beautiful." Then he added: "I'm the attorney. I have the $350,000 house. I have the 6-figure career. You may be the tall, young, hot nymph, but I am the prize! Start convincing! I would not expect you to be the other woman. I would want you to be so hot and treat me so well that you'd be THE woman. R U that good?" Could the prosecutor be subject to suspension of his license for these texts?

d) Yes, because the texts constitute sexual harassment of the victim.

412. An attorney represented a small business owner in litigation against a former employee, who was a Canadian immigrant. During the bench trial, the attorney cross-examined the former employee on the witness stand, and after two of her answers turned to the judge and asked, "Are you going to believe an alien or a U.S. Citizen?" Could the attorney be subject to suspension for these comments?

d) Yes, because this is discrimination based on national origin.

442. An attorney sends a solicitation letter to a prospective client. The recipient of the letter opens it and reads it, but the person does not respond. The attorney then sends a follow-up letter to the prospective client. Could the attorney be subject to discipline for sending the second letter?

d) Yes, if after sending a letter or other communication as permitted by the Rules, the lawyer receives no response, any further effort to communicate with the recipient of the communication may violate the provisions of Rules.


Conjuntos de estudio relacionados

Prep U's - Chapter 31 - Mental Health Disorders of Older Adults

View Set

Exam FX, VA Property & Casualty Insurance

View Set

Theory week 4-Comprehensive Family Assessment

View Set

Chemistry test review Activity 3 & 4

View Set

PSYA2 - Type A behaviour and disease - Friedman and Rosenman

View Set

Chapter 24: Parathyroid and Adrenal Disorders

View Set

PA Accident and Health Insurance

View Set

CompTIA Security+ Exam SY0-501 Common Vulnerabilities

View Set